.
.
Math Central - mathcentral.uregina.ca
Quandaries & Queries
Q & Q
. .
topic card  

Topic:

ne

list of
topics
. .
start over

1803 items are filed under this topic.
 
Page
1/1
A pattern for a truncated cone 2013-12-20
From Josh:
I need to know how to layout a truncated cone with a base diameter of 18" a top Diameter 15.25 and a height of 20". Your help will be greatly appreciated.
Answered by Penny Nom with a spreadsheet by Don.
Solving a cone 2008-01-29
From Marija:
[Note: this problem shows how to solve anything about a cone if you are given two measurements.]
Is there a formula for finding the diameter of the base of the cone?

Answered by Stephen La Rocque.
Pattern for a truncated cone 2007-05-11
From Mike:
I have been trying to get this cone flat so I can build this column. Can you please help me so I can figure this out? Thanks for your help.
Answered by Stephen La Rocque.
The length of a line at 45 degrees 2022-04-09
From Aiden:
Let's say one line is diagonal and one line is vertical. If they eventually meet at a 45 Degree angle, How long will the diagonal line be compared to the vertical line? (For Example: If The Vertical Line is Exactly 1 Inch.
Answered by penny Nom.
The next rational number 2022-01-14
From Sophia:
what comes after 3 1/2?
Answered by Penny Nom.
A graph with 100 vertices and one edge 2022-01-11
From Maftuna:
A graph has 100 vertices and only one edge. How many connected components does it have?
Answered by Penny Nom.
New pool liner 2021-07-14
From Kareena:
I have a rectangular prism shaped pond that are 4ft by 8ft by 2ft. My pond is lined on the sides and bottom with a pool liner so that it doesn't leak (the top of the pool does NOT have a liner). I am redoing my pool and would like to increase the size of the pond. Each dimension will increase by the same amount, and the size of the pool liner will double. How much I am increasing each dimension of the pond? Give an exact answer and than an approx. answer rounded to the nearest tenth.
Answered by Penny Nom.
UFO speed in MPH from nearest galaxy in one year 2021-05-22
From Joe:
If the next closest galaxy is 15 quadrillion miles away and there are 8760 hours in one year; how fast would a UFO have to fly to travel from the next galaxy, in MPH in one year ?
Google calculator gives this result: 1.7123288e+12
I don't understand what the "e+12" means.
How is that pronounced, ex. trillion, quadrillion etc..?

Answered by Harley.
The height of a cone 2021-05-13
From Carmen:
If a cone has a slant height of 25 inches and a radius of 7 inches what is the height in inches of the cone?
Answered by Harley Weston.
A scenario for a linear equation 2021-01-19
From raven:
I have to create a scenario for the following equation y=-1x+45 and then find what each part of the equation represents
Answered by Penny Nom.
10^1,000,000 2020-12-18
From G:
Looking for the name of a really big number. its a 1. with a Million zeroes after it.
Answered by Harley Weston.
0^(a+bi) 2020-12-03
From Douglas:
I realize raising 0^a = 0 if a>0 and undefined if a<=0.

If have read that 0^bi is undefined for all b.

What I don't understand is why 0^(a+bi) = 0 if a and b are not equal to zero.

Is this purely by definition or is there a logical reason why this is the case?
(I have taken Complex Analysis, so have a fairly good understanding of complex numbers.)

Answered by Penny Nom.
Two equations with fractions 2020-10-22
From mia:
I am supposed to look for x and y. my teacher didn't explain very well.
please help : ((
2x - y = 5
x/4 + y/3= 2

Answered by Penny Nom.
Solving an inequality 2020-10-18
From nunya:
3(4 + 2n) > 2n - 16
Answered by Penny Nom.
Simultaneous equations 2020-09-05
From Paa:
a+b=8
a+c=13
b+d=8
c-d=6
find the value of each letter

Answered by Penny Nom.
Filling holes with stone dust 2020-07-30
From Zach:
I’m looking for amount of stone dust needed for thirteen holes with a 3ft depth and 12in diameter with a 4x4 post. I would need the measurement in KG.
Answered by Harley Weston.
A word problem involving linear equations 2020-06-22
From Allie:
The length of a rectangular swimming pool is p m and its width is q m. Its given tht the length of the swimming pool is twice its with. if the perimeter of the swimming pool is 150 m, find the values of p and q this is in the chapter linear equations
Answered by Penny Nom.
The substitution method 2020-06-10
From Hardebolar:
simultaneous equation by substitution method

.a-b=2/3
a+b=1 2/3

Answered by p.
A triangular garden 2020-05-24
From yoonji:
the 3 sides of a triangular garden measure 200 ft., 250 ft., and 300 ft., respectively. the garden is to be divided by a line bisecting the longest side and drawn from the opposite vertex. what is the length of the bisecting line?
Answered by Penny Nom.
A sand castle in the shape of a truncated cone 2020-05-23
From Summer:
A sand castle is in the shape of a truncated cone as shown. Calculate the length of the diameter of the base.
The diameter of the top of the truncated cone is 20cm
The height is 30cm
The slopes are 32cm

Answered by Penny Nom.
Maximizing the volume of a cone 2020-05-18
From Ella:
Hello, this is question - 'If you take a circle with a radius of 42cm and cut a sector from it, the remaining shape can be curled around to form a cone. Find the sector angle that produces the maximum volume for the cone made from your circle.'
Answered by Penny Nom.
An angle i a triangle 2020-05-16
From Ogunjobi:
Two goal post are 8m apart a footballer is 34 m from one post and 38m from the other within what angle must he kick the ball if he is to score
Answered by Penny Nom.
The volume of a stone 2020-05-02
From KHOSI:
A rectangular container is 9cm wide,17cm long and contains water to a depth of 11cm. A stone is placed in the water and the water rises 2,2cm .Find the volume of the stone
Answered by Penny Nom.
The volume under an an uneven surface 2020-05-01
From Solon:
I am trying to fill an area of floor which has an uneven surface. I need to know the volume of fill. If you imagine a rectangular room (4 corners) that measures 189 inches by 240 inches and the depth at each corner is different.
Let us say corner #1 has a depth of 1", corner #2 has a depth of 0", corner #3 has a depth of 2" and corner #4 has a depth of 4".
How would i find the volume of said floor area?

Any assistance would be greatly appreciated!

Answered by Harley Weston.
A system of equations. 2020-04-27
From solomon:
xy + x =28
x + y +4

Answered by Penny Nom.
Power notation 2020-04-26
From Aditi:
The power notation of -1 / 128
Answered by Penny Nom.
The missing diameter of a truncated cone 2020-04-17
From Brandon:
Need a formula to find missing large Dia of a truncated cone where the small dia =.19" Height= 2" and taper =16deg
Answered by Harley Weston.
A rectangular garde 2020-04-10
From Denise:
In a rectangular garden a person says "if I made that bed 2 feet wider and 3 feet longer it Would have been 64 sq,ft bigger. But if it had been 3 feet wider and two feet longer It would have been 68 sq.ft bigger.
What is the length and width of the garden bed?

Answered by Penny Nom.
Setting up a linear equation 2020-04-10
From Jessie:
Michaela and Aleah play on the same basketball team. In one game, Michaela scored one fifth of the teams points and Aleah scored one tenth of the teams points. Together, they scored a total of 42 points. How many points did the team score?

I can't figure out how to put together the equation with the given variables. I have to show the work and I also need to know how to make that kind of equation. Thanks!

Answered by Penny Nom.
A linear system 2020-04-02
From Dadzie:
x+s=8→1
y+x=13→2
y-p=6→3
s+p=8→4

Answered by Penny Nom.
Six-digit telephone numbers 2020-03-12
From Lulamile:
What is the probability that a six-digit telephone number has no repeated digits? The telephone number cannot begin with a zero
Answered by Penny Nom.
A straight line graph 2020-02-26
From Penehafo:
I have a problem of drawing a straight line graph by the equation y=2x-1
Answered by Penny Nom.
A system of inequalities 2020-02-01
From Isaac:
Austin has x nickels and y dimes, having a maximum of 15 coins worth a minimum of $1 combined. No more than 4 of the coins are nickels and no less than 11 of the coins are dimes. Solve this system of inequalities graphically and determine one possible solution.
Answered by Penny Nom.
A point on a line 2020-01-25
From Samved:
The line 3x/5 - 2y/3 + 1 = 0 contains the point (m, 2m-1). Calculate the value of 'm'.
Answered by Penny Nom.
The volume of a coffee cup 2020-01-25
From Amirah:
What is the volume of a polystyrene cup with a height of 8cm, diameter of 5cm at the top and 4cm at the bottom when it is filled with water?
Answered by Penny Nom.
The equation of a line 2020-01-05
From coco:
Good afternoon If relationship between total cost and the number of units made it linear ,and if costs increases by 7$ for each additional unit made ,and if the total cost of 10 units is 180$ .
Find the equation of the relationship b/n total cost (Y) & number of unit made (x).

Answered by Penny Nom.
A circle, a point, and a tangent line 2019-12-31
From Faisal:
A circle has radius 10 units and passes through the point (5,-16). The x-axis is a tangent to the circle. Find the possible equations of circle?
Answered by Penny Nom.
81^5 as a power of 3 2019-12-29
From NA:
How do we turn 81^5 into the power of 3?
Answered by Harley Weston.
Volume of a tree 2019-12-19
From Maria:
I have trees for sale. I need to calculate the volume of a tree from base which is wider than the stem.The base will be used for the making of hurleys, similar to hockey sticks so the wider base is crucial.
Answered by Penny Nom.
The equation of a line 2019-12-10
From Jourdan:
Suppose a mining company will supply 96000 tons of ore per month if the price is 80 dollars per ton but will supply 70500 tons per month if the price is 20 dollars per ton. Assuming the supply function is of the form y = m x + b , find the slope, m and y-intercept, b
Answered by Penny Nom.
Roll two dice and record the difference 2019-12-09
From Barbara:
Suppose you roll two dice 100 times. Each time you record their difference (always subtracting the smaller one from the bigger one to get a positive difference). The possible values you get are 0,1,2,3,4 and 5. You record the frequency of each value in the following table:
Difference of two dice 0 1 2 3 4 5
Observed frequency 12 31 26 13 10 8
Let your null hypothesis be that the dice are fair, and the alternative hypothesis be that they are not fair. Using a confidence level of α = 0.10, test the null hypothesis by a goodness-of-fit test.

Hint: begin by
completing table:
x 0 1 2 3 4 5
f(x)

Answered by Penny Nom.
Simultaneous equations 2019-10-16
From deepak:
This is too complicated please help me
8/x - 10/y = 1 and x+y=9

Answered by Penny Nom.
Fractions and negative powers 2019-09-22
From Will:
-2/3 to the -6 power
Answered by Penny Nom.
A one foot rise in water level 2019-09-15
From Stuart:
If a lake were to rise one foot - how many linear feet would it spread onto the shore line ? What is the correlation - 1 foot of a rise = 10 feet onto the shore ? Does it depend on the lake size - if so Lake Ontario in Canada.
Answered by Penny Nom.
Vectors 2019-09-06
From Kayla:
Hi! Due to time constraint issues, we skipped vectors in Pre-Calc last year. However, I'm attempting to complete my AP Physics summer homework and there are 3 problems with vectors. As I explained, I have no idea even where to begin and as such I need some help.

Problem 1:
A plane flying at 90° at 45 m/s is blown toward 0° at 62 m/s by a strong wind. Find the plane's resultant velocity. I have tried all sorts of online calculators but none are providing me with a formula I can use to solve this problem on my own.

Problem 2:
If you walk 367 m North and 785 m West what is your total displacement from your original location?
What is the problem asking for when it says displacement?

Problem 3:
A plane travels on a heading of 127.0° at a velocity of 25 km/hr. What are the horizontal and vertical components of the plane's velocity? As you know already, I have never done vectors so I don't know what horizontal and vertical components of velocity are.

Thank you So Much for your help!! I am struggling!!

- Kayla

Answered by Penny Nom.
Using the Laws of Exponents 2019-09-06
From Samiya:
i have been struggling with the laws of exponents so im having trouble with this question. the question is to write (b*exponent2*c)(b*exponent5*c*exponent4*)
Answered by Penny Nom.
A negative minus a negative 2019-09-03
From Maggie:
Why is a negative minus a negative a negative?
Answered by Penny Nom.
Can one equation with two variables be solved algebraically? 2019-08-30
From Don:
Can one equation with two variables be solved algebraically?
Answered by Penny Nom.
A flat top cone 2019-08-25
From Mary:
I want to make a paper model of a flat top cone with the radius of the base is 12.5 cm and the radius of the apex is 7 cm.The height is 15 cm and the radius of the apex is 5 cm. The height is 15 cm.
Can you help me with the angle and construction, please?
Thanks.

Answered by Penny Nom.
A cone of maximum volume 2019-08-14
From Refilwe:
The slant height of a cone is 10cm. Determine the radius of the base so that the volume of the cone is a maximum
Answered by Penny Nom.
The maximum volume of a cone 2019-07-14
From A student:
find the maximum volume of a cone if the sum of it height and volume is 10 cm.
Answered by Penny Nom.
One part in 10^x 2019-06-26
From Jim:
What is the meaning of the phrase ‘one part in 10x” where the x is an exponent? I have seen these examples: “...better than one part in 10x” and “...to within one part in 10x.”
Answered by Penny Nom.
The area of a patio 2019-06-23
From Miguel:
Stephanie has 60 feet of metal fencing material to only fence 3 sides of a rectangular patio. A tall metal wall serves as the fourth side of the rectangle. write a function for the area the garden (A) in terms of width in feet (x) What measures of the width would give an area of at least 150 feet?
Answered by Penny Nom.
What are the next three terms? 2019-06-18
From Shriya:
-6, -2 , 2, 6,10 what are the next three terms?
Answered by Penny Nom.
Two power generators 2019-06-07
From A student:
At 100% efficiency two generators would produce 750 MW of power. At efficiencies of 65% and 75%, they produce 530 MW. At 100% efficiency, what power would each produce?
Answered by Harley Weston.
An inline lock with four rows 2019-06-05
From Edward:
I have a inline lock with four rows of numbers 0-9.The numbers cannot be repeated. Can you please print the possible codes so I can go through them one at a time. The lock and the cable cost $300. Don't want to pitch it. You guys are lifesavers, can't wait to hear back. Thanks.
Answered by Penny Nom.
How many lines of symmetry does a circle have? 2019-05-28
From raji:
how many lines of symmetry does a circle have???
Answered by Penny Nom.
Two linear equations with a fraction 2019-04-26
From Asha:
2x=-3y …1
x-2+-4/3(y+1) ...2


note: … is equation

Answered by Harley Weston.
(3d) Whether line segment intersects triangle 2019-04-13
From rj:
Well hello,

Lets say I have a triangle,

Not any ordinary triangle, but a triangle that is represented in 3 dimensions. I also have a line segment.

Lets say that this triangle has points A(0,0,0), B(1,0,0), C(0,1,1)

And the line segment has points D(0,1/2,0), E(1,1/2,1)

In what way can I find if the line segment and triangle intersect? Their intersection point?

I don't prefer matrices, because they don't make any sense, but ... if that's the only way to solve this, then do what's necessary.

Thank you in advance.

Answered by Penny Nom.
4^x=2^x+6 2019-04-05
From Olivia:
How do you find the (b,n) if the solution of the equation 4^x=2^x+6 can be expressed at logb(n) where b and n are both prime numbers?
Answered by Penny Nom.
A dice game 2019-03-17
From Remo:
So, little game my kid came up with.

Dice are standard d6.

I roll a die, he rolls a die. If he gets the same number as me OR a bigger number he wins. So far so good. Used the opportunity to get him to calculate the probabilities of him winning, see if I could get him interested into looking at this game from a more academic perspective. He did fine.

Then we upped the ante. He rolls one die, I roll two dice. I his die has a number that is equal or greater than both of mine, he wins.
We had a bit of trouble solving that one, and had to resort to a table with all the results ( 6x36) to figure out where we stood.

Then again we upped. He rolls two dice, I roll two dice. If any one of his dice has a number equal to or greater than both of mine he wins. I only win if one of my dice is greater than both of his. And we got stuck. Figured I would need to find a formula to resolve this one, as our earlier solution ( table everything ) suddenly is 36x36, giving well over a thousand possible results. And if we ever go bigger than that then we *really* are going to need another way to calculate it.

Answered by Harley Weston.
The top half of a cone 2019-02-27
From danica:
What portion of the height from top that the cone be cut into two equal volume?
Answered by Penny Nom.
A linear equation 2019-02-11
From Nick:
What's a linear equation?
Answered by Penny Nom.
The exponential form of sqrt(13) 2019-02-10
From isaebl:
what is the exponential form of sqrt(13)
Answered by Penny Nom.
An ice cream cone 2019-02-05
From Eyuel:
a scoop of ice cream has a 3-inch radius how tall should the ice cream cone at the same radius be in order to contain all of the ice creams inside the cone
Answered by Penny Nom.
A cylinder with a cone on top 2019-01-16
From Berania:
A water tower in New York City has the shape of a cylinder with a cone on top. The cylinder has a diameter of 12 feet and a height of 15 feet. The roof has an inclination angle of 25 degrees.
(a) Determine the height of the cone to the nearest tenth of a foot.
(b) Determine the overall volume of the tower to the nearest cubic foot.

Answered by Penny Nom.
Simultaneous equations with fractions 2018-12-14
From zaheer:
solve simultaneous equations and give answer in fractional form
3x - 2 = 4y +5/3
y + 7 = 2x + 4
would really appreciate some help on this please

Answered by Penny Nom.
A puzzling equation 2018-12-04
From Manny:
Hello,

Can you explain why this is wrong:
-1 = (-1)^3 = (-1)^{6/2} = ((-1)^6)^{1/2} = (1)^{1/2} = 1 ??

Thank you

Answered by Penny Nom.
The top of a truncated cone 2018-12-02
From Sameer:
Bottom cone radius 1124.height is 200. Top radius 15degree decree to bottom radius. Top radius how much ?
Answered by Penny Nom.
Two cones formed from two circular sectors 2018-11-30
From APARNA:
The central angles of two sector of a circle are 60° and 120° . Using these sectors two cones are made. If the radius of the smaller cone is 5 cm, find the radius and base area of the larger cone.
Answered by Penny Nom.
A line cuts a curve 2018-11-11
From roxanne:
Hello, i need to ask a question, do you mind explaining and writing the formula of how to solve equations such as "find the set values of k for which the line y-2x-5 cuts the curve y=x^2 +kx+11" please
Answered by Penny Nom.
Why 1/3 in the volume of a cone? 2018-10-20
From Keith:
In the formula for the volume of a pyramid, Volume = 1/3 * base * height, where does the 1/3 come from, intuitively?
Answered by Penny Nom.
The point-slope equation for a line 2018-09-23
From Sally:
The equation of the line in point-slope form for the line passing through the point ( 4, -6) and parallel to the line y = -6x + 2. Describe the steps.
Answered by Penny Nom.
An inequality with no solution 2018-09-20
From Alena:
I'm trying to help with a homework problem. Can you please help me?

2kx - 3k < 2x + 4 +3kx

We are solving for "k" with no solution.

Answered by Penny Nom.
Exponential form 2018-09-14
From Saquanna:
What is 25 in exponential form
Answered by Penny Nom.
Exponential form 2018-09-12
From Grace:
hi, I can't figure out how to do 735 in exponential form. note that im only in 5th grade and its not supposed to be in the scientific notation or whatever. okay, thanks!
Answered by Penny Nom.
Two spinners 2018-08-13
From Atina:
A spinner has four equal sectors and a number is written on each sector; 1, 2, 3 and 4. A two-digit number is formed by spinning two times. The number on the first spinning makes the first digit and the number on the second spinning makes the second digit. For example, 2 on the first spinning and 1 on the second spinning make the number 21.
(a) Give the sample space S for the experiment.
(b) Consider the following events : E = odd number; F = number smaller than 35; G = prime number. Give the subset of outcomes in S that defines each of the events E, F, and G.
(c) Describe the following events in terms of E, F, and G and find the probabilities for the events.
• getting an even integer less than 35.
• getting an odd number or an prime.
• getting an even number greater than or equal to 35 that is a prime number.
• an odd number smaller than 35 that is not a prime number.
(d) Are E and F mutually exclusive events? Give a reason for your answer.

Answered by Penny Nom.
Exponents 2018-08-13
From Ella:
What is the radical form of 3xy raised to 5 quantity three- fifths?
Answered by Penny Nom.
Form a cone from a circle sector 2018-08-12
From Tinashe:
A 216 sector of a circle of radius 5cm is bent to form a cone. Find the radius of the base of the cone and its vertical angle.
Answered by Penny Nom.
5 mm of water over 1 hectare 2018-08-10
From Joyce:
Please help me with this question.
How many tonnes of water fell on 1 hectare of land during a rainfall of 5mm?

Answered by Penny Nom.
Truncated cones 2018-08-08
From Belgacem:
Hello, I have a truncated cone: height 500mm , Base radius=147mm and I want to calculate the Top surface radius. How to do that? thank you
Answered by Penny Nom.
Paving stones area 2018-08-08
From Declan:
I am thinking of buying some reclaimed block paving stones. There are 750 pieces in total.
60% are L- 140mm x W -140mm x H-50mm
40% are L-170mm x W-140mm x H-50mm How much area will this cover?

Answered by Penny Nom.
Linear feet and cubic feet 2018-07-16
From Dave:
How do I convert: 1 foot wide x 15 inches high x 19 inches deep

We are trying to figure out how to convert the linear footage into cubic footage the above are the figures represent just 1 linear feet of several thousand

Any thing helps,

Thanks in advance

Dave

Answered by Penny Nom.
The area, radius and slant height of a cone 2018-05-18
From joette:
If you have a cone how to find the slant height when given the area and radius?
Answered by Penny Nom.
The square root 3 in exponential form 2018-05-09
From Paige:
how do you write the square root 3 in exponential form
Answered by Penny Nom.
A rectangular prism with rounded corners 2018-04-25
From Jackie:
Rectangular prism that is 40 by 20 with rounded corners with a radius of 2.5.

What would the surface area be?

Answered by Penny Nom.
More on the curvature of the Earth 2018-04-23
From will:
the formula for figuring the earth's curve goes against logic, looking at a fixed point and backup 1mi. the point drops 8" then 16" in the next mi. and 32" in the third mi. why shouldn't it be 24" why is the 8" per mi. squared can you tell me in laymens terms why this is it goes against logic it would seem the correct wat would be to add up 8" per mile as you back up from the fixed point 8"- 16"- 24"- 32" not 8/16/32/64"
Answered by Harley Weston.
A volume expression for a pyramid 2018-04-23
From Shaheer:
Do a three sided pyramid and a square pyramid have the same formula if you want to calculate the volume?
Answered by Penny Nom.
A cone formed from a circular sector 2018-04-18
From Jessica:
A circle has a radius of 7.5cm. A sector with an angle of 240 degrees is cut out from the sector. If the sector is folded to form a cone. Find the length of the cone.
Answered by Penny Nom.
Graham's money box 2018-04-12
From Tasnim:
Graham's money box consists of sh.10 and sh.5 coins. There are 24 coins and it's total value is 150. Find how many of each coins are there in the box
Answered by Penny Nom.
The volume of a tent with a hexagonal base 2018-03-27
From shohel:
A tent has its base in the shape of a regular hexagon whose sides are 10m . If the height of the tent is 12m , then find its volume.
Answered by Penny Nom.
1/4 acre of land 2018-03-15
From Jean:
I have 1/4 acre of land.it is square.what should the measurement be for each side be in feet
Answered by Penny Nom.
Solve sinX=0.703X for X 2018-03-13
From PARAM:
sinX=0.703X
Answered by Penny Nom.
0.366 x cos square (02 degree 17 mins 27 seconds) 2018-03-12
From michael:
what is 0.366 x cos square (02 degree 17 mins 27 seconds)
what is 0.366 x cos square (88 degree 26 mins 45 seconds)

Answered by Penny Nom.
Parents, grandparents, great grandparents, ... 2018-02-28
From jessica:
every person has 2 birth parents, 4 birth grandparents, and so on. These people are known as ancestors. How many ancestors does a person have in each generation? write in exponential form. 3ed generation, great-grandparents? 4th generation? 10th generation?
Answered by Penny Nom.
Dividing a region in half 2018-02-24
From mandy:
There is a line through the origin that divides the region bounded by the parabola y=4x−5x2y=4x−5x2 and the x-axis into two regions with equal area. What is the slope of that line?
Answered by Penny Nom.
The slope of a line 2018-02-22
From Steve:
my current slope is 6%. I want to know how many inches higher I will be at 18' Thanks
Answered by Penny Nom.
A footballer angle 2018-02-14
From Kim:
Two goal posts are 8m apart. A footballer is 34m from one post and 38m from the other. Within what angle must he kick the ball if he is to score a goal.
Answered by Penny Nom.
Solar cells 2018-02-12
From karla:
Solar (photovoltaic) cells convert sunlight directly into electricity. If solar cells were 100% efficient, they would generate about 1000 watts of power per square meter of surface area when exposed to direct sunlight. With lower efficiency, they generate proportionally less power. For example, 10% efficient cells generate 100 watts of power in direct sunlight. Suppose you want to supply 22 kilowattss of power to a house by putting solar panels on its roof. For solar cells with the average power of 49 watts per square meter of solar panels, how many square meters of solar panels would you need? Assume you can make use of the average power from the solar cells
Answered by Penny Nom.
Water in a cone 2018-02-10
From Shuvo:
The diagram shows a vertical cross-section of a container in the form of an inverted cone of height 60 cm and base radius 20 cm. The circular base is held horizontal and uppermost. Water is pursed into the container at a constant rate of 40 cm3/s.
Show that, when the depth of water in the container is x cm, the volume of the water in the container is (πx^3)/27 cm3.
Find the rate of increase of "x" at the instant when "x" = 2.

Answered by Penny Nom.
Adding two negatives 2018-02-04
From Pinasshakoane:
(-23)+(-3)
Answered by Penny Nom.
The code on a new phone 2018-02-02
From Maria:
Hi -
My husband gave me a new Samsung Galaxy mobile for out anniversaire gift. He set me a challenge which I have to solve before I can use the Phone. He has set the pin code with numbers 4, 7 and 0 and created a four digit code with the said numbers. Is there a way I can somehow list all possible combinations and try them all? Thanks!

Answered by Penny Nom.
Positive and negative values of a function 2018-01-30
From Grayson:
f(x)=x^6-x^4
Interval: ( negative infinity, negative one )
Test Value: negative two
Function Value f(x): positive forty eight
Interval: ( negative one, zero )
Test Value: negative one
Function Value f(x): zero
Interval: ( zero, positive one )
Test Value: positive one
Function Value f(x): zero
Interval: ( positive one, positive infinity )
Test Value: positive two
Function Value f(x): positive forty eight

What is the sign of f(x) for each Interval?

Answered by Penny Nom.
The equation of a line 2018-01-23
From Gloria:
Write the equation of a line in standard form through the point (1, 6) and perpendicular to the line 2x –y = 9.
Answered by Penny Nom.
The product of the coordinates of a point on a line 2018-01-17
From Sophia:
For any point on line l, the product of the x-coordinate and the y-coordinate is less than or equal to zero , what is the answer?
Answered by Penny Nom.
A fraction of a mile 2018-01-06
From Sierra:
There are 5280 feet in a mile. what fraction of a mile is represented by 660 feet?
Answered by Penny Nom.
4444^4444 2017-12-10
From Sashi:
4444 to power of 4444=?
Please share the result with simplification.

Answered by Penny Nom.
Exponential form 2017-11-25
From Sherry:
What is the exponential form of 3sqrt 15?
Answered by Penny Nom.
The graph of graph y= -2x-1 2017-11-11
From Natividad:
How do you graph y= -2x-1?
Answered by Penny Nom.
The volume of a cone without calculus 2017-10-02
From Akash:
How to find the volume of a cone without the knowledge of calculus?
Answered by Penny Nom.
An alphabetic sequence 2017-09-17
From malik:
find the letter of the following series....?

ZOTTFFSSE-

Answered by Penny Nom.
The angle between two planes 2017-09-12
From Arthi:
Hello. hope you are having a good day.

I am stuck on a maths question in which it is asking, how to find the angle between two given planes. These planes are in a 3D shape such as a prism.
How can I do this?
Please reply as soon as possible

Thank You

Answered by Penny Nom.
Simultaneous equations with fractions 2017-09-09
From Farah:
Hi, my name is Farah. I hope you can help me with this question . X/2 + g/5= 3 and 2g - f = 10
Answered by Penny Nom.
Simultaneous equations 2017-09-02
From keto:
x-y=2,x^2+xy=12
Answered by Penny Nom.
A pair of linear inequalities 2017-08-08
From Hempstone:
Solve the inequalities 3x-5<2x+5>4x+9
Answered by Penny Nom.
Catherine, Molly and Tom 2017-07-19
From liezel:
what is the answer of this problem, If Catherine and molly has 320pesos altogether, Molly and tom have 360 in all. Tom and Catherine have 240pesos altogether.How much does each of them have?
Answered by Penny Nom.
Parallel and Perpendicular lines 2017-07-17
From Karen:
Find an equation for the line with the given properties. Perpendicular to the line 7x + 2y = -6; containing the point (-2, -1)

Find an equation for the line with the given properties. Parallel to the line 5x - 3y =-6; x intercept = 3

Answered by Penny Nom.
Exponential form 2017-07-06
From estela:
what is exponential form of 9×1000
Answered by Penny Nom.
Solve the equation completely cos 2x = 1 2017-06-08
From Lava:
Solve the equation completely cos 2x = 1
Answered by Penny Nom.
Simultaneous equations with fractions 2017-06-02
From Jamal:
1/x + 1/y =5 and 1/y - 1/x =-1
Answered by Penny Nom.
Two wires between two buildings 2017-05-11
From Bill:
I need to find the height of the intersection point of two building wires< br /> Building A is 860 feet high and building B is 480 feet high. The wires existent from the top of one building to The bottom of the other < br /> The distance between is 32 feet
Answered by Penny Nom.
A lost iPhone pass code 2017-04-17
From Sam:
I have a locked iPhone and it has all my of my holiday photos on it. I changed the password and then I think I did a incorrect number pattern but anyway I know it ends with 99 and the first 4 digits are 235 and 6 could you please send all of the combinations for this password (without the 99) Thx and plz get back ASAP I need it for school
Answered by Penny Nom.
A function which is onto but not one-to-one 2017-04-16
From Avinash:
Define a function f(x):N-->N which is onto but not one-one.Where N is set of natural numbers.
Answered by Penny Nom.
Forming a cone from a circle 2017-04-15
From Tasha:
A sector of a circle subtends an angle of 216 degrees at the centre, If this sector is used to form a cone of vertical height ,8cm, calculate the base radius of the cone
Answered by Penny Nom.
A rectangular box filled with oil 2017-04-11
From morpal:
we want to calculate volume of rectangular box height 1275mm and lengths 2400mm and breadth 1270mm and oil s. gravity is .950 how much kg. of oil i can store
Answered by Penny Nom.
Successive Differences 2017-04-06
From Twaha:
Please find the equation of the sequence 1 2 4 7 11 16 22
Answered by Penny Nom.
How to find last two or three digits of a large exponent like 17^256? 2017-03-30
From Raginee:
How to find last two or three digits of a large exponent like 17^256?
Answered by Penny Nom.
2^a, 2^b, 1^1 and 1^0 2017-03-17
From Émile:
If you had 2^a = 4 and 2^b = 4 you could assume that a = b right? Yet we can observe that 1^1 and 1^0 both equal 1, yet 1 = 0 isn't true. So if given a log base 1 number 1 the would the answer be 1 AND 0?
Answered by Penny Nom.
What is the grade of the incline? 2017-03-12
From Jon:
If I am climbing 16 feet in elevation over 80 feet horizontal. What is the grade of the incline
Answered by Penny Nom.
Exponential form 2017-03-04
From michael:
I was teaching my student about exponent when we stumbled across this problem.

"write (-125)^-3 in exponent form"

The problem is with the number "-125". I think -125 = (-5)^3. But one of my student thought that it is -125= -5^3.
Both of them is equal to -125 but they totally different in structure. I wonder which one is correct and why it is. I am worried if they get this wrong, it might spell problem in the future when we start substituting the numbers with variable.
Thank you very much for reading my question and answering it.

Answered by Penny Nom.
Five equations 2017-01-16
From Muhammed:
A x 4 = E
B ÷ 4 = E
C + 4 = E
D - 4 = E
___________
& A + B + C + D = 100
what is the value of E

Answered by Penny Nom.
One millionth of one percent of 90,000,000,000 2017-01-16
From Gene:
What absolute number is one millionth of one percent of 90,000,000,000
Answered by Penny Nom.
Plans for dog kennels 2017-01-04
From Pat:
So... I'm trying to draw up a plan for my new kennel buildings. They are going to built out of grain bins. I know, cool,... right. Anyway the kennels will be along the outside of the circle and be shaped like wedges. the center will be a play area. All of the kennel sides are 6 ft in length so the size of the gate will determine how many kennels I can fit in each bin. If I want to put an odd number of kennels in and I cant draw a straight line across to make all of the wedges the same size... how do I figure out the angles...degrees ? maybe i'm making it too hard but i'd like to give the dogs as much space as i can. I did up some ... attached.
Answered by Harley Weston.
Simultaneous equations with fractions 2016-12-21
From Kimi:
I am stuck on this,can someone please help me????

1/2x+1/3y=11
8x+2/5y=102
Can someone please explain how to solve questions like these?? Or else I will never learn...
Thank you

Answered by Penny Nom.
Joining a fitness center 2016-12-18
From Salem:
A fitness center offers a special price for group-class students. The cost of registration is 30 dollars and then you have to pay 25 dollars per month. What is the rule of the equation in the light of
c=---m+----?

Answered by Penny Nom.
How far apart are the boats? 2016-12-13
From Halley:
Two boats leave port at the same time. Boat A travels east at a speed of 12 km/hr. Boat B travels southwest at a speed of 14 km/hr. After two hours, how far apart are the boats? North is 0 degrees. How do I figure this out. Thanks
Answered by Penny Nom.
Acres and miles 2016-11-30
From Carolyn:
If I am running a pipeline of 1,100 miles, how many one acre plots will it take to cover this distance?
Carolyn

Answered by Penny Nom.
2^x= -2x - 11 2016-11-27
From Kathy:
I don't know how to start this problem:
2^x= -2x - 11

Answered by Penny Nom.
Simultaneous equations 2016-11-25
From Rohan:
x-2/3 + y - 1/4 = 13/12
2-x/2 + 3 + y/3 = 11/6

Answered by Penny Nom.
Parallel lines 2016-10-26
From Jordan:
(7,-5) 3x -4y=5
Answered by Penny Nom.
y = mx + c and Ax + By + C = 0 2016-10-18
From Erick:
please explain to me how to convert an equation from the form y=mx+c to Ax+By+C=o form.
Answered by Penny Nom.
A linear relationship 2016-10-16
From Bianca:
x represents the number of hours since 8am. y represents the number of children the school nurse has seen during the school day. The nurse has seen 3 children by 8am and 27 children by noon.
How many children has the nurse seen at 2pm?
If the nurse has seen 18 children, what time is it?

Answered by Penny Nom.
An airplane schedule 2016-10-14
From Stephanie:
Starting at 8:00a.m., a passenger plane leaves from an airport every 6 minutes and a cargo plane leaves every 7 minutes. When is the next time these kinds of planes will leave at the same time?
Answered by Penny Nom.
The equation of a line 2016-10-01
From Miriam:
write the equation of the line that passes through (6,7) and is perpendicular to 3x+5y=0
Answered by Penny Nom.
The length of stock on a roll 2016-09-23
From Ken:
I have a roll that is 13.5 inches wide ( OD) , and is wound on a 3 inch core. the thickness of the material is 3.5 thick.
Do you know of a formula that can calculate the linear feet on that roll??

Answered by Harley Weston.
The equation of a ski slope 2016-08-31
From Hannah:
The information provided is
---A ski slope has a slope of -0.2. You start at an altitude of 10,000ft. Then the question is "Write the equation giving altitude (a) as a function of horizontal distance moved (d).
I know this is probably a very simple question but I didn't take a math class my senior year and now I'm a freshman in college, so some of it is very frustrating to me.
Any help is much appreciated, thank you!

Answered by Penny Nom.
Exponential form 2016-08-31
From Lily:
Hi! I was wondering, I was having a little issue understanding this one problem. It was "write 81 in exponential 2 different ways" and I already got 9(2) but I was wondering what the second one could be! Thanks! -Lily
Answered by Penny Nom.
Two sequences 2016-08-20
From Ting:
The first four terms of a sequence are 4, 20, 56 and 120. By comparing with the nth term of sequence 1,5,14 and 30 which is 1/6n(n+1)(2n+1), find a formula for nth term. Can you precisely show out how you do this questions ( with the steps please)
Answered by Penny Nom.
A cone inscribed in a hemisphere 2016-08-07
From anonymous:
A cone is inscribed in a hemisphere. the slant height of the cone is 20cm. When cut along its slant height, the cone forms a sector of a circle.
find the angle of the sector, to the nearest 1 decimal place.

Answered by Penny Nom.
The dimensions of a rectangle 2016-07-25
From Krunox:
A rectangle is twice as long as it is wide. Its perimeter (the sum of the lengths of its sides) is 60 cm. Find its length and width.
Answered by Penny Nom.
The temperature inside and outside a greenhouse 2016-07-22
From Mehzad:
Every day, the temperature in a greenhouse is at its low temperature, 70 degrees Fahrenheit, at 2 a.m. and at its high temperature, 84 degrees Fahrenheit, at 2pm. Its temperature increases linearly between 2 am and 2pm , and decreases linearly from 2 pm and to 2am. The outside temperature follows the same linear patterns, but has a low temperature of 60 degrees Fahrenheit at 2 am and a high temperature of 78 degrees Fahrenheit at 2 pm. At which of the following times will the temperature inside and outside the greenhouse be the same?

a) 8:00 am
b) 12:00 noon
c) 4:00 pm
d) The two temperatures will never be the same

Answered by Penny Nom.
Two equations with fractions 2016-07-22
From kanesh:
p/2+q/3=3

p/4+2q/3=3

Answered by Penny Nom.
Simultaneous equations 2016-06-25
From sena:
2x/3+3y/2=-1
4x-5y=22

simultaneous equation

Answered by Penny Nom.
The lateral side length of a cone 2016-06-05
From Diane:
Question is find the lateral side length of a right cone with area of 372 sq. cm and base circle radius of 9 cm.
Answered by Penny Nom.
A pair of equations with fractions 2016-05-11
From tiya:
hello, i want to know how to solve this question.

m/6+2n/3=6
-m/10=2n/5=2

Answered by Penny Nom.
Solve 2^2x + 3(2^x) - 4 = 0 2016-04-27
From Lloyd:
Solve the equation 2^2x + 3(2^x) - 4 = 0
Answered by Penny Nom.
600(1+0.05)^7 2016-04-21
From chelsey:
I am unsure on how to solve this problem, 600(1+0.05) 7 ?
Answered by Penny Nom.
A stained glass cone lamp 2016-04-09
From Edwin:
In making a 16" dia. cone lamp (stained glass), how many square feet of glass do I need.
Answered by Penny Nom.
The last two digits of a phone number 2016-04-03
From Joshua:
I want to find the last two digits of a phone number whose first eight numbers of a ten digit phone numbers are known.
Answered by Penny Nom.
The volume of a cone 2016-03-31
From Odum:
Find the volume of a cone with radius 6.5cm and height 12.6cm
Answered by Penny Nom.
Angles 2016-03-12
From Laurynn:
What are angles in general (please include the 'angle of incline')

Thank you
Laurynn

Answered by Penny Nom.
Corrosion inhibitor in a pipeline 2016-03-02
From Edu:
I have a 12 in pipeline and 5 miles long. I need to coat it with corrosion inhibitor how much chemical will I need
Answered by Penny Nom.
Solve for x and y 2016-02-27
From ntshidi:
Y=1/2x+4and1/4x-6
Answered by Penny Nom.
Perpendicular lines 2016-02-14
From Brendon:
points A (-4; 6) , B (1;-2), C (5;-3) and D (4; 1) are given prove that AC and BD are perpendicular to each other
Answered by Penny Nom.
Two equations in two unknowns 2016-02-13
From Anumba:
4x + 2y = 4
7x - y = -11

Answered by Penny Nom.
The equation of a vertical line 2016-02-07
From Kagiso:
The equation for a vertical line that goes through the point (7; -3) is?
Answered by Penny Nom.
Exponential form 2016-01-26
From Jufe:
What is the exponential form of ³√11x²
Answered by Penny Nom.
Exponential growth 2016-01-26
From Paul:
The value of a stamp was 0.03 in 1851. The value in 2011 is $50,000. If the value increased exponentially, what is the annual growth rate?
Answered by Penny Nom.
A system of linear equations 2016-01-24
From kareem:
my name is kareem
and i am a student i have a math puzzle and i tried to solved it but it always have same mistake
x-y=9
x+z=12
z-n=14
y+n=2

Answered by Penny Nom.
The diameter of the top of a truncated cone 2016-01-24
From Peter:
I am trying to calculate the diameter of a truncated cone given one diameter the height of the cone and a 10% taper from one end to the other. For example a butter churn is 18" tall and 9" in diameter at the base. the sides need to slope inward at 10%
What is the diameter at 9" and 18"

Answered by Penny Nom.
The perimeter of a one quarter acre square 2016-01-15
From Kerry:
If you have a perfect square that is one quarter of an acre what are the length of the sides and how do you arrive at the solution? thank you, Kerry.
Answered by Penny Nom.
The range of a function defined piecewise 2016-01-13
From sarah:

f(x)={x-2, x<-2
       {x^2-4, -2_<x_<2
       {x+2, x>2

find the range
Answered by Penny Nom.

Two transport companies 2016-01-06
From Brynleigh:
The Sugar Sweet Company will choose from two companies to transport its sugar to market. The first company charges $5096 to rent trucks plus an additional fee of $200.75 for each ton of sugar. The second company charges $5500 to rent trucks plus an additional fee of $175.50 for each ton of sugar. For what amount of sugar do the two companies charge the same? What is the cost when the two companies charge the same?
Answered by Penny Nom.
A sequence 2016-01-05
From Mia:
the next three terms in each sequence. 0.4, 0.54, 0.68, 0.82,
Answered by Penny Nom.
A graph of the water level of a river 2016-01-05
From emoni:
Suppose that the water level of a river is 34 feet and that it is receding at a rate of 0.5 foot per day.
Answered by Penny Nom.
The height of a parabolic arc 2015-12-30
From Tom:
Is there an algebraic means to determine the highest point of a parabolic arc if the base and perimeter are known?
Answered by Penny Nom.
Constructing simultaneous linear equations 2015-12-28
From Deborah:
Linda thinks of a two-digit number. The sum of the digits is 8. If she reverses the digits, the new number is 36 greater than her original number. What was Linda's original number?

Thank you!

Answered by Penny Nom.
Sierpinski Pyramid 2015-12-14
From victor:
Is a tetrix (a Sirpienski Pyramid/Tetrahedron) its own hyperplane?...alternatively, is it a hyperplane?
Answered by Robert Dawson.
The positive root of sin(x) = x^2 2015-12-13
From Kemboi:
Find the positive root of the equation sin(x) = x^2
Answered by Penny Nom.
Linear equations in two variables 2015-12-13
From priya:
I have problem in solving these equations please help me today itself very urgent:
I)2x+y=y
II)pie*x+y=9

Answered by Penny Nom.
The line of intersection of two planes 2015-12-09
From Rabz:
Determine the vector equation for the intersection between the two given planes 2x+7y equal to 14,x+z equal to 7
Answered by Penny Nom.
The height of a truncated cone 2015-12-04
From Jack:
I need to build a truncated cone that has a top of30 inches and a base of 64 inches . The sides need to be at a 64.5 degree angle. This will determine the height. Can this be calculated?
Answered by Penny Nom.
A tangent line to a parabola 2015-12-02
From pei:
Given that the line y=mx-5 is a tangent to the curve y=2x^2+3 find the positive value of M.
Answered by Penny Nom.
Pouring a foundation 2015-11-27
From Lisa:
Hi im building a house 30ft long 18 ft wide I would like to pour a foundation of 6 inches deep. How much sand, cement , 1/2" gravel do i need.
Answered by Penny Nom.
A concrete container 2015-11-12
From Karstin:
The exterior of a concrete container will be 10 feet by 8 feet by 4 feet tall. The walls and the bottom are 6 inches thick. What will it cost to construct it if concrete is $98.95/cubic yard?
Answered by Penny Nom.
The uniqueness of the lub 2015-10-25
From Aishwarya:
Prove that if the lub and glb of a set exist then they are unique
Answered by Penny Nom.
Exponential form 2015-10-17
From Will:
What is exponential form of -8?
Answered by Penny Nom.
The sequence 1,2,4,7,11,16... 2015-10-15
From HEMANT:
what is the next term in the sequence : 1,2,4,7,11,16,?
Answered by Penny Nom.
How can the probability be zero? 2015-10-13
From anagha:
Hi,
thank you for answering (beforehand)
The other day I was just thinking of circles and I came across this peculiar thing. Most of'em say that there are infinite number of points on the circumference. and, if it is true, if we spin the radius, the probability that it will point a a particular point is 1/infinity, right?
It didn't make any sense to me and so, I thought there might be an explanation or something of the kind...?

Answered by Harley Weston.
Mathematics 2015-10-13
From anagha:
Hi,
thank you for answering (beforehand)
The other day I was just thinking of circles and I came across this peculiar thing. Most of'em say that there are infinite number of points on the circumference. and, if it is true, if we spin the radius, the probability that it will point a a particular point is 1/infinity, right?
It didn't make any sense to me and so, I thought there might be an explanation or something of the kind...?

Answered by Harley Weston.
A tangent line to a circle 2015-09-03
From Jamie:
Use the Greek method to find an equation of the tangent line to the circle x^2+y^2-4x+6y+4=0 at the points (3,2square root 2-3.
Answered by Penny Nom.
A two digit number 2015-08-21
From Cameron:
Write the 2 digit number that matches the clues.

My number has a tens digit that is 8 more than the ones digit. Zero is not one of my digits.

My number is ________.

Answered by Penny Nom.
The nth term 2015-07-04
From ujjal:
What is the nth term in the order 6,8,10....
Answered by Penny Nom.
A tangent to y = x^3 2015-05-31
From Brayden:
Show that a tangent line drawn to the curve y=x^3 at the point (d,f (d)), where d>0, forms a right triangle with the x and y axes in quadrant 4 whose area is (2/3)d^4.
Answered by Penny Nom.
A stone walkway around a pool 2015-05-29
From Jerri:
I have a rectangular pool 15x30 and it will require stone placed around it 4 feet out and 4 inches deep how much will I need? Thanks, jerri
Answered by Penny Nom.
The intersection of a plane and a cone 2015-05-16
From Tom:
Is there a way to derive an equation that describes the perimeter of the intersection of a plane and a cone regardless of the angle of the plane to the cone. Assume that the plane does not cut through the base of the cone, the x, y, z location of the vertex is known, the distance from the vertex to the plane through the axis is know., and that the angle of the cone is known.
Answered by Chris Fisher.
The method of elimination 2015-05-01
From oreanna:

Question from oreanna, a student:

How do u solve 2x+9y=3

7x-4y=-25 in elimination


Answered by Penny Nom.
A fraction with a negative exponent in the denominator 2015-04-30
From ellen:
7/ab to the -4 power
Answered by Penny Nom.
An inequality 2015-04-19
From mr.:
Simplify the inequality √x>x and represent on a number line
Answered by Penny Nom.
A wireless fence 2015-04-18
From Dave:
I'm buying a wireless fence to keep my pet in my yard. It has a half acre range. In a straight line how far would that be?
Answered by Penny Nom.
Two airplanes 2015-04-14
From john:
two planes leave an airport at the same time, one going northwest (N35*W)at 400 mph and the other going east at 332 mph. How far apart are the planes after 4 hours to the nearest mile?
Answered by Penny Nom.
A word problem with fractions 2015-04-09
From Lorraine:
If the numerator of a certain fraction is doubled and the denominator is increased by 1, the fraction becomes 1/2.

If the numerator of the original faction is squared and the denominator is decreased by 2, the fraction becomes equal to 1.

Let x be the numerator and let y be the denominator of the original fraction. Write down two simultaneous equation in x and y.

Solve these equations to find two possible values for the given fraction.

Answered by Penny Nom.
A tangent to y=x^3 passes through (0,2) 2015-04-05
From Kevin:
Given that the curve y=x^3 has a tangent line that passes through point (0,2). Find the equation of the tangent line
Answered by Penny Nom.
n^2 is a multiple of 100 2015-03-30
From Rahul:
I have to prove that n^2 is a multiple of 100 is necessary or Sufficient condition (or both) for n being multiple of 10
Answered by Penny Nom.
A region defined by three lines 2015-03-16
From Lucinda:
find the area of the figure formed by the line y=-5x+9 and the x-axis and y-axis.
Answered by Penny Nom.
A cone of maximum volume 2015-03-16
From Mary:
I have to use a 8 1/2 inch by 11 inch piece of paper to make a cone that will hold the maximum amount of ice cream possible by only filling it to the top of the cone. I am then supposed to write a function for the volume of my cone and use my graphing calculator to determine the radius and height of the circle. I am so confused, and other than being able to cut the paper into the circle, I do not know where to start. Thank you for your help! -Mary
Answered by Robert Dawson.
Constructing a cone 2015-03-12
From Levi:
While the mathematics at the welding shop page was very helpful there is one vital part missing. If I have never built a cone how do I figure out how much bigger the diameter of my circle has to be when laying flat on the floor verses the diameter when it has been pulled into a cone.
Answered by Harley Weston.
Extraneous solutions 2015-03-07
From Emily:
I have a question about Extraneous Solutions, Because I was recently researching to figure out on how to determine that a solution is extraneous and many of the answers talked about how if a answer is negative that it should always be extraneous but now I found out that positive solutions can also be considered as extraneous solutions so that is why I am really confused Extraneous Solution's.
I would really appreciate it if you could clearly explain to me how to determine an extraneous solution from a normal solution.
Thanks,
Emily

Answered by Chris Fisher.
Two equations with fractions 2015-02-26
From Pulane:
Hi math centre I've been trying to solve these equations for days now please help (6/x)-(1/y)=4
(9/x)+1=(-2/y)

Please help me solve them simultaneously
Thank you

Answered by Penny Nom.
A region described by inequalities in cylindrical coordinates 2015-02-21
From Rahul:
I am not able to visualize a solid given by inequalities as under (in cylindrical coordinate system) 0= I know that (can visualize) first 2 inequalities say it is a cylinder of infinite length. It seems that third inequality poses the restriction to the height as well as it makes us to remove a cone.
But I just can not understand it clearly.
Thanks in advance!
Regards,
Rahul

Answered by Penny Nom.
Two equations 2015-02-16
From nigel:
2x+1/2y=1
6x-3/2y=21

Answered by Penny Nom.
The volume of a cone 2015-02-05
From Linda:
How do you calculate the volume of a cone that is 25cm high and has an angle of 20 degrees?
Answered by Penny Nom.
1,4,9,61,52,___ 2015-01-21
From Mubashir:
1,4,9,61,,52,___,84,___,18,1,121,___ please tell me its formula and also that rule that is being applied here like adding 3 or subtracting 5 etc.
Answered by Walter Whiteley.
Largest cone in a sphere 2015-01-15
From Alfredo:
What is the altitude of the largest circular cone that may be cut out from a sphere of radius 6 cm?
Answered by Penny Nom.
A sequence 2015-01-13
From cyaz:
write a rule for each sequence then find next 3 terms for 7/8, 3 1/8, 5 3/8
Answered by Penny Nom.
Missing digits in a phone number 2015-01-07
From Janelle:
I had a friend try and play a game on me, regarding phone numbers, so I only know the area code , and the last four digits of the phone number. Which means, there are 3 digits that I need to figure out all the possibilities to.
Answered by Penny Nom.
A line segment of length root 5 2014-12-15
From angela:
On the dot grid below, draw and label a line segment with length square root 5 the dot grid is 8 by 10
Answered by Penny Nom.
A cone is 2/3 full of sand 2014-12-14
From Janice:
A cone with a radius of 3.5 cm and a height of 12 cm is 2/3 full of sand.
What is the volume of the sand inside?

Answered by Penny Nom.
Planar curves 2014-12-13
From ann:
what does planar curve mean in your definition of a cone?
Answered by Penny Nom.
1999 equations in 1999 unknowns 2014-11-28
From Subrahmanya:
Solve the following system of 1999 equations in 1999 unknowns :

x1+x2+x3=0, x2+x3+x4=0……., x1997+x1998+x1999=0,

x1998+x1999+x1=0, x1999+x1+x2=0.

(in x1,x2,...................,x1999 the numbers 1,2,.............,1999 are subscripts)

Answered by Penny Nom.
Exponential notation 2014-11-27
From Jaqui:
May sound super easy, but how would you SHOW that you simplified 3x3 to the power of 2 (the little 2)?

thank you!

Jaqui

Answered by Penny Nom.
Doubling the dimensions of a cone 2014-11-21
From Hannah:
If the volume of a cone of height 10 cm is 261.8 cm3, show that this volume is increased by a factor of 8 if the dimensions of the cone are doubled.
Answered by Penny Nom.
Two quadratic equations 2014-11-20
From rajesh:
Sir,
I would like to discuss on question-
-Find the value of variable X and Y, equations are
X2+Y2+2XY=25
X2+Y2-2XY=1

Sir, my point is equation looks not so easy.My opinion is there many (more than 4)value possible for both X and Y.
Kindly help me.
Waiting for your response.
regards,
rajesh

Answered by Penny Nom.
The cube root of 729 2014-11-12
From Alexis:
What would be the square root of 729 to the third power and could you explain how to get the answer?
Answered by Penny Nom.
What is -3 squared? 2014-11-12
From Christine:
On a math test, it said "What is -3 squared?" It did NOT say "What's is (-3)squared"

The teacher's explanation is that if there are no brackets or parenthesis, you ALWAYS square the number first then do the negative, so the answer should be -9, but I can't find anywhere that confirms this.

Help please.

Answered by Robert Dawson.
Prime factorization in exponent form 2014-10-31
From Emma:
I need to find out how to make a prime factorization of 120 in exponential form.
Answered by Penny Nom.
Water in a spherical container 2014-10-17
From Natalie:
A spherical container with radius 10cm being filled with water such that the water level, h cm, in the container increase at a rate of 0.5 cm s^(-1). Show that the area of the horizontal surface of water, A cm^(2), in the container is given by A = π(20h - h^(2) ).
Answered by Penny Nom.
A linear system 2014-10-14
From pheter:
4/x - 1/y = 3 .... equation (1)
6/x - 2/y = 5 .... equation (2)

Answered by Penny Nom.
One zero to a quadratic expression 2014-10-06
From Kevin:
Find a value of n such that f(x) = 3^x2 + nx + 6 only has one zero, is this the only solution to the question?
Answered by Penny Nom.
Sieve of Eratosthenes 2014-10-03
From Hope:
using Sieve of Eratosthenes to determine for in instance prime numbers between 1 to 200, to what nth number should i stop?, 4th, which is 11 or what?
Answered by Robert Dawson.
Trig functions and the unit circle 2014-10-02
From Jake:
I was wondering what conclusions can be drawn about the trigonometric functions and how they work about the circle. Can you also please give me an explanation for it? Thank you.
Answered by Penny Nom.
Two equations in x and y 2014-09-25
From seyilogo:
solve y=2x - 3 and (4x - 2y) / x + y = 1 simultaneously
Answered by Harley Weston.
Meetings for 77 students 2014-09-18
From Adam:
We have a new group of students starting, and would like to set up a series of small group meetings so each student meets each other, hopefully without meeting the same people twice.

We have 77 students. In theory I would like to have 11 meeting slots, with 10 groups in each slot, with up to 8 students in each group. Is there a way to do this?

I have wondered if it might be easier to do 9 meeting slots, with 9 groups per slot, with 9 students per group. Thanks!

Answered by Victoria West.
Two equations in two variables 2014-09-18
From Susan:
(28x + 36y) - [20000 - .75(28x + 36y) + 60000] = 5000
x + y = 10000
solving two equations involving variables

Answered by Penny Nom.
A tangent to a curve passing through a point not on the graph 2014-09-15
From Aquilah:
For the curve y = x2 + 3x, find the equations of all tangent lines for this graph that also go through the point (3, 14).
Answered by Penny Nom.
Three collinear points 2014-08-28
From jhanavi:
p,q,r are three collinear points.p and q are(3,4) and (7,7) respectively and PR is equal to 10 units.find coordinates of R
Answered by Penny Nom.
Filling three holes with stones 2014-08-20
From mark:
how many tonnes of hardcore/crushed stone would it take to fill 1 hole 9ft diameter 5ft deep and 2 holes both 3ft diameter and 5ft deep
Answered by Penny Nom.
1/3 of $320 2014-08-13
From Student:
Grade:5
Please help I got tutor for selective and this question is really bugging me a lot what is 1/3 of $320?

plez answer quick before I fail :(

Answered by Tyler Wood.
The method of elimination 2014-07-05
From leo:
please explain how can i solve this problem

3x-6y=-38
6x-9y=44

using elimination and simultaneous method thank you :)

Answered by Penny Nom.
An oval pool 2014-06-21
From steve:
I have a 16' x 28' oval pool that is buried 24" deep inground. The dig site is dug 2' wider all the way around the pool. I need to back fill this area with stone. I want to fill this area with 6 to 8" of stone. How many tons of stone will this take?

Thanks you
Steve

Answered by Penny Nom.
Simultaneous equations 2014-06-20
From rana:
solve the simultaneous equations
a)3x=7y
12y=5x-1

Answered by Penny Nom.
Two regression lines 2014-06-14
From catherine:
Given a set of data values ,we can get two regression lines .Explain.
Answered by Robert Dawson and Penny Nom.
The equation of a line 2014-05-16
From Michael:
Find the equation of a line that passes through (2,-1); the sum of the x- and y-intercepts is 2. (There are two answers)
Answered by Penny Nom.
A piecewise graph 2014-05-10
From Zoe:
Lightning Energy charges residential users for each unit of electrical energy bought from them each quarter, according to the scale below:
0-500 units cost 17 cents per unit
500-1500 units cost $85 plus 10 cents for each unit in the excess of 500
1500 or more units cost $185 plus 9 cents for each unit in excess of 1500
Sketch a graph showing the charge C as a function of U for up to 2000 electricity units

Answered by Penny Nom.
A question concerning two lines 2014-05-09
From Hela:
The equation of the line L is 6x+5y=3, and the equation of line Q is 5x-6y=0, which statement below about the line is true?
A) same y intercept
B)parallel
C) Same x intercept
D)perpendicular

Answered by Penny Nom.
x - 2 Sin[x] = 0 2014-05-08
From chanmy:
please help me to sole this equation x - 2 Sin[x] = 0,thank you
Answered by Penny Nom.
If x+y+z=1 then x^2 + y^2 + z^2 = ? 2014-04-30
From Nitin:
If x+y+z=1 then x^2 +y^2+z^2 = ? ?
Answered by Robert Dawson.
A cone of vision 2014-04-29
From David:
It is known that a fish in water looking up has a 97 degree "cone" of vision that sees "through" the surface of the water. If a fish lies 4 inches below the surface, the cone forms a window (circle) smaller than if a fish lies 8 inches below the surface. What is the ratio of inches of depth to the radius of the circle on the surface that is its visual window?
Answered by Penny Nom.
A triangular chicken pen 2014-04-27
From Cierra:
Margaret has two lengths of fence, 20 meters and 24 meters, for two sides of a triangular chicken pen. The third side will be on the north side of the barn. One fence length makes a 75° angle with the barn. How many different pens can she build if one fence is attached at the corner of the barn? What are all the possible lengths for the barn side of the pen?

Not sure what they are asking here... please show step by step what to do! Thank you so much!

Answered by Penny Nom.
A trig problem 2014-04-25
From srishti:
Consider the points P= (-sin(a-b),-cosa), Q=(cos(a-b),sina), R=(cos(a-b+c),sin(a-c), where 0
Answered by Chris Fisher.
Simultaneous equations with fractions 2014-04-19
From Maryam:
I looked at your example of simultaneous equations with fractions and applied it to my question from an educate exam papers but I couldn't get it to work. The question is:

x/8 - y = -5/2
3x + y/3 = 13

Answered by Penny Nom.
A tangent of the curve (x/a)^n+(y/b)^n =2 2014-04-15
From sudhir:
the equation of tangent of the curve (x/a)^n+(y/b)^n =2. at(a,b) is
Answered by Penny Nom.
Two cones 2014-04-09
From c.j:
what is the length of the radius of the LARGER cone(the LARGER cone has a slant height of 15) when the SMALLER cone has a radius of 8 and a slant height of 12ft ,please help.
Answered by Penny Nom.
The sides of a triangle 2014-04-06
From Michael:
I am supposed to sove for the length of side "b" of an irregular triangle. I am given the following:
Side a: 65'
Side b: Find this length
Side c: 50'
Angle A: unknown
Angle B: unknown
Angle C: 52 degrees
I am supposed to use the law of cosines to solve for side "b" and my teacher says there is no mistake in the "givens" for the problem. I do not see how this can be done using the law of cosines and i have not figured out how to sove for angle B to use the law of cosines.

Answered by Penny Nom.
An inequality 2014-03-27
From James:
a step on a stairway is at least 8 inches high. write and solve an inequality to represent the maximum number of steps between floors that are 10 feet apart
Answered by Penny Nom.
Ms. Werenich's nickels, dimes and quarters 2014-03-19
From London:
Ms. Werenich has twice as many dimes as nickels and 4 more quarters than nickels. If she has $4.50, how many of each coin does she have? What would the let statements be of this problem and can you solve it?
Answered by Penny Nom.
A circle which is tangent to two perpendicular lines 2014-03-09
From MJ:
I'm a College Student taking up Bachelor of Secondary Education on Math Subject. And I'm struggling for my research about Circles. I done solving the said topic particularly on this question:

"What are the possible equations of a circle being tangent to a pair of perpendicular lines, having the origin as the Point of Intersection and the C (h, k), where h, k ∈ℤ"

But I can't get what would be the process that I must do in order to jive to my idea/goal for that problem.
Please check my idea that the numerical coefficients of the equation is equal to the radius of the circle. Thanks in advance! :)

Answered by Penny Nom.
Determine the selling price 2014-03-08
From augustine:
knowing an auctioneer charges a 7% fee and I know I want $74,400 what is the formula to determine selling price.
Answered by Penny Nom.
A cone inscribed in a sphere 2014-02-28
From joel:
how can I find the radius and the height of a cone INSCRIBED in a sphere, given the sphere having a radius of 6? ( note: the diameter of the cone is equal to its slanted height).
Answered by Penny Nom.
Four digit phone numbers 2014-02-25
From Tom:
Hi, I'm an online math teacher, working on lessons for my students.

This question is in their book and I can't do it...! Help!

It has to do with phone numbers, and in this problem, we are only dealing with the last 4 digits of the number.

It asks how many possible numbers we have if at least one digit repeats in the last 4 digits of the number.

Answered by Penny Nom.
The diameter of the base of a cone 2014-02-23
From elwin:
i have a sector of a circle that has 120 degree and 6 cm length. What will be the diameter of the base of the cone.
and what is the diameter of the angle is changed to 180.

Answered by Harley Weston.
Paycheck after deductions 2014-02-23
From Louise:
So I got a paycheck for 89.02 they took out in deductions 15.69. I want to know the percentage they are taking out so that I can figure my take home pay each week before I get it. So I know what to expect for pay each week so I can budget my bills from now on a weekly basis. If I know the percentage than I can just figure out the percentage through how many hours in the week I worked and multiply my gross pay and know what my take home will be
Answered by Penny Nom.
Three investment partners sharing the profit 2014-02-22
From Ayatullah:
Hello;

We are three partners and bought a property at 2050000 and sold at 2300000.
Details of investment of the partners
X= 1500000
Y= 400000
Z = 150000

My questions are
1) How could i calculate percentage of investment of each partner?
2) How could i deduct 30% commission from each partner in profit?
3) How could i distribute the profit amongst the partners?


Thanks in advance

Answered by Penny Nom.
What is the exponential form of 1/square root of 6v? 2014-02-16
From Prince:
What is the exponential form of 1/square root of 6v
Answered by Penny Nom.
Simultaneous fractional equations 2014-02-15
From benjamin:
hi math central. benjamin here. during class, i had problem with this topic. normally i wont have problems with math but this topic i just too hard for me. please help i am having exam and test next week on this topic

here is the question:
using substitution method, solve the simultaneous equation.
(x+1)/(y+2)=0.5

(x-2)/(y-1)=1/3

Answered by Penny Nom.
Water in a conical funnel 2014-02-11
From Marcus:
Water is running out of a conical funnel at the rate of 1 inch^3/sec. If the radius of the base of the funnel is 4 in. and the altitude is 8 in., find the rate at which the water level is dropping when it is 2 in. from the top.
Answered by Penny Nom.
A sequence 2014-02-10
From Joshua:
hello..the question I have today is... A sequence... 1,5,14,30,55,91 find the general terms.
Answered by Penny Nom.
The volume of a cone 2014-02-08
From hibba:
why is the volume of right circular cone divided by 3?
Answered by Penny Nom.
Prime factorization 2014-02-06
From Kadeejah:
Write the prime factorization of 37 in exponential form
Answered by Penny Nom.
A sequence 2014-01-30
From joshua:
hi...really struggle with one question that the teacher gave us...
The sequence is 1,2,1,5,8,1,11,___,___

Answered by Penny Nom.
Two nonlinear equations 2014-01-26
From Naryn:
(1÷x) + (1÷y) = (7÷12)
xy = 12

Answered by Penny Nom.
An inequality 2014-01-25
From LANELL:
this is a problem to solve: 1/3 + 2/7 >=x/21 -- part of the answer is (-oo) not exactly that similar--it is on a calculator as a symbol- sure you know what it is I am talking about- the x will be a number
Answered by Penny Nom.
Mixing water and wine 2014-01-19
From manish:
a man mixes three part of water to one part of wine.When he consumes one part of mixture he realise that the drink is too dilute and hence adds one part of wine to the remaining quantity what is the ratio of water to wine in the new mixture.
Answered by Penny Nom.
An inequality 2014-01-17
From Rosie:
4x/x-2<3

four x divided by x-2 is less than or equal to 3

Answered by Penny Nom.
Fractions and exponents 2014-01-11
From john:
what is the exponent form of 9^4/7^-2 and 6^-3/10^-5
Answered by Penny Nom.
A box-and-whisker plot with one whisker 2014-01-09
From Susan:

Question from Susan, a teacher:

I read your answer to the question about could a box & whisker have NO whiskers.
It made sense to me. However I am preparing for my advanced class and I have a sample graph comparing box plots of data.
Several of the box have only 1 whisker. There is no explanation for this. Can you help me out. I cannot think of a data set that would produce this. Thanks!


Answered by Penny Nom.
When would one flip the inequality sign? 2014-01-09
From Natasha:
Would one flip the inequality symbol in this equation: (explain why) (-9a)/(-9) > 81
And please explain in what circumstances one would flip the inequality sign
THANKS!

Answered by Penny Nom.
A scalene triangle 2014-01-07
From cherry:
Hi
I have been given a scalene triangle of sides -
8m, 6m, and 10m ..
Please help me to find out the area and help me to find out the height ..

Answered by Penny Nom.
25% profit 2014-01-02
From Finn:
Hello,
The question is all about buy-and-sell business.
Problem:
Pencil - $6 for whole sale price
     $8 if I sell the item How do I get the 25% profit? (you can change the whole sale price and the retail price[if i sell the item])
if I buy the pencil at 24 pieces and sell it at 24 pieces.

Answered by Penny Nom.
The slope of the line y=-2x+5 2013-12-29
From victoria:
what is the slope of the line with the equation y=-2x+5
Answered by Penny Nom.
Distance from a point to a line 2013-12-28
From Sue:
I need to find the distances from the west line on 1st Take Point and 2nd Take Point.
Thanks,
Sue

Answered by Penny Nom.
A linear equation 2013-12-26
From khushboo:
A farmer sowed wheat and paddy in two fields of total area 5000 sq.m. Write a linear equation and draw a graph to represent the same?
Answered by Penny Nom.
Find the next 3 terms of the sequence 2013-12-19
From Lanelyn:
Find the next 3 terms of the sequence 2,3,9,23,48,87,__,__,__
Answered by Robert Dawson.
7 digit phone numbers 2013-12-08
From Sean:
Hi I was wondering how you would calculate how may 7 digit phone numbers there are with only odd digits?
Answered by Penny Nom.
How do I graph y=-2x-1? 2013-12-07
From cindy:
How do I graph y=-2x-1?
Answered by Penny Nom.
Exponential form 2013-12-01
From Alina:
My question says to express each number in exponent form in as many different ways as I can; 16, 81 and 64. How do I do this?
Answered by Penny Nom.
What is the slope of this line? 2013-12-01
From Charlene:
What is the slope of this line?
Answered by Penny Nom.
n log n = 36 * 10 ^ 12 2013-11-12
From shihab:
How to find value of n in this equation :

n log n = 36 * 10 ^ 12

Answered by Penny Nom.
Substitution type simultaneous equations 2013-11-03
From Kayla:
I am having problems with substitution type simultaneous equations, when the variable you are substituting is a algebraic one:
y=x^2-3x+4 and 3x-2y=1
I have rearranged 3x-2y=1 to get x=(1+2y)/3 but when I substitute this x value into the other equation, I get the wrong answers!
Would appreciate any help! Thank you.

Answered by Penny Nom.
A plane 2013-10-27
From Thomas:
Why is this 3d plane flat?
if m ≠ n
arcsin(sin(m*arctan(x/y))), arcsin(cos(n*arctan(x/y))),
Pi-arcsin(sin(m*arctan(x/y)))-arcsin(cos(n*arctan(x/y)))
This is using radians.
These are angles of a triangle, when m and n are the same it is a right triangle. All three inputs are not equal to each other. The last input is pi minus the first two inputs, which makes a complete triangle.
I was kind of hoping to find more lissajous curves. It doesn't make sense to me that this is a flat plane, a diagonal one at that.

I'm not a student or anything. I'm just wasting time.

Answered by Chris Fisher.
Extraneous solutions 2013-10-22
From tom:
i need an equation where x=-2 is the correct answer and x=-3 is an extraneous solution. can you provide me with such an equation??
Answered by Harley Weston.
A frustum 2013-10-12
From Lily:
A cone of height 6in. and radius of base 4in. has its top cut off by a plane parallel to its base and 4in from it. Find the volume of the remaining frustum.

I have worked out the volume of the entire cone but I don't know how to work out the radius of the top of the frustum.

Thanks

Answered by Penny Nom.
Proportional rates 2013-10-10
From Varsha:
A province's Ministry of Social services has found that both the number of people needing social assistance and the province's total expenditures on social assistance are proportional to the rate of unemployment. Last August when the provincial unemployment rate was 8.4 %, the province provided assistance to 89,300 individuals at a total cost of 4107.4 million. The forecast unemployment rate for next August is 7.9%. How many people can the province expect to need social assistance next August? What amount should the province budget for social assistance in August?
Answered by Penny Nom.
Maximize the volume of a cone 2013-10-09
From Conlan:
Hi I am dong calculus at school and I'm stumped by this question:

A cone has a slant length of 30cm. Calculate the height, h, of the cone if the volume is to be a maximum.

If anyone can help me it would be greatly appreciated.

thanks.

Answered by Penny Nom.
A line through (5, 1) 2013-10-08
From allison:
find an equation in standard form for line passing through point (5,1) and perpendicular to line x=4
Answered by Penny Nom.
A triangle problem 2013-10-02
From raneem:
ABC is a triangle in which : BC=20cm. M(<B) =29 and m(<C)=73 . D is the midpoint of BC Find the length Of AB and AD approximated to 2 decimal places
Answered by Penny Nom.
A Hole-in-one on all four par 3's 2013-09-04
From Robert:
There are four (4) par 3's at a specific 18 hole course. One member has the honor of making a Hole-in-one on all four par 3's. It has taken him 8 years to do it! How does one go about computing the odds of this TREMENDOUS accomplishment? Respectfully, Robert laffal
Answered by Victoria West.
Ordering crushed stone 2013-09-03
From Prakash:
Dear Sirs, I am working in a Soft Landscaping contracting company. If I need to purchase crushed stone with the size 50-70mm for $53,429 m^2$ area, how many 20feet containers should I need to order to my suppliers? The 20foot container has internal dimensions 5,897 mm by 2,348 mm by 2,285 mm and the $53,429 m^2$ area is to be covered by 10 cm of stone.
Answered by Harley Weston.
Two perpendicular lines 2013-08-15
From Hanna:
The problem says, "Write the equation in standard form for the line that is perpendicular to the graph of y=5x+1 with a y-intercept of 4." I have no clue how to do this. Please help me!
Answered by Penny Nom.
A system of equations 2013-08-13
From Nina:
3x-2y=8
-x+3y=5
What is the solution of the system of equations

Answered by Penny Nom.
An equation in slope-intercept form 2013-08-12
From Brittney:
write an equation in slope-intercept form of the line that is parallel to the grapg of y=-3/4x+6 passes through -8,-5
Answered by Penny Nom.
Differentiate x^x - 2^sinx 2013-08-09
From tarun:
derivative of x^x - 2^sinx
Answered by Penny Nom.
Slicing through a cone to form an ellipse 2013-08-06
From Pulkit:
we get an ellipse on slicing through a cone. Is there a relation between central axis of the cone and this ellipse?
Does it pass through the any of the foci of the ellipse?

Answered by Chris Fisher.
Simultaneous equations 2013-07-10
From Warren:
solve this simultaneous equation:
xy=4
2x+3y=14

Answered by Penny Nom.
A one acre plot of land 2013-06-19
From Carol:
If I have a one acre plot of land, what is the measurement in feet on each of the four sides?
Answered by Penny Nom.
An isosceles tiangle 2013-06-16
From Izzy:
what's the height of an isosceles triangle which has a base of 50 m, and both of the other sides are 25 m?
Answered by Penny Nom.
A kennel for a beagle 2013-06-03
From david:
Hi, I'm building some beagle kennels and I am in need of help with an angle problem. I need to place a roof on my kennel with a drop of 2inches across 3ft 10inches. the posts on the right side will be 5ft and the post on the left will be 4ft 10in. the posts are 4x4 and the space to be covered is 3ft 10in from the outside of the 4x4. Please help, thanks.
Answered by Penny Nom.
Simson line 2013-05-23
From Nazrul:
From any point p lying on the circumcircle of the triangle perpendiculars PD , PE are drawn on BC,CA respectively. The line segment ED intersects at the point 0 ; prove that PO ⊥ AB ( PO is perpendicular to AB ).
Answered by Chris Fisher.
siin (A) and sin (A/2) 2013-05-09
From shanaia:
given that sin A=4/5 and A is obtuse.find sin (A/2)
Answered by Penny Nom.
650*(1+0.005)^96 2013-04-17
From joAnn:
i am trying to figure out how to solve a question like this for example:

650(1+.005)96

I have been doing it this way but takes forever and wanted to know a shorter way

650 x 1.005 x 1.005 x 1.005 x 1.005 96 times

HELP

Answered by Penny Nom.
A cone problem 2013-04-14
From Courtney:
Hello,
I am having difficulty solving this cone problem. The biggest challenge I have is figuring out what angle they are talking about:
The angle at the base of a cone is 34.5 degrees. Find the diameter of the cone at point on the edge of the cone 26cm from the tip.

Answered by Penny Nom.
How do i write log_8(P) =7 in exponential form? 2013-04-14
From nancy:
How do i write this in exponential form log8P =7
Answered by Penny Nom.
4 linear equations with 3 unknowns 2013-04-12
From Marian:
how to solve for 3 unknowns in 4 simultaneous equations
Answered by Penny Nom.
The equation of a line 2013-04-04
From Miranda:
My coordinates provided are (6, 6) and (-8, 9) and i need to put this into ax=bx=c format. I remember the equation to find the slope of y2-y1/x2-x1 to get the slope. However when i put that back into the y=mx+b format it still leaves me with a lot of variables. Please help :)
Answered by Penny Nom.
The distance between 2 lines in 4D 2013-04-01
From samane:
how can i measure the distance between 2 lines in 4D??
Answered by Chris Fisher.
We can't write sinx and cosx as a finite polynomial. 2013-03-31
From rimoshika:
prove that we can't write sinx and cosx as a finite polynomial.
Answered by Walter Whiteley.
Simultaneous equations with fractions 2013-03-31
From Terence:
5/x-6/y=1 17/x+30/y=16 I been spending whole day to solve this question. Would be very grateful if you can help I try The denominator value is a equations term which make is simultaneous equations so hard.
Answered by Penny Nom.
Tangents to the curve y = x^3 2013-03-24
From Ethan:
How many tangent lines to the curve y = x^33 pass through the point (2, 4)? For each such line, and the exact coordinates of the point of tangency on the curve.
Answered by Penny Nom.
A linear programming problem 2013-02-27
From Kelley:
A manufacturer of skis produces two types: downhill and cross-country. Use the following table to determine how many of each kind of ski should be produced to achieve a maximum profit. What is the maximum profit? What would the maximum profit be if the time available for manufacturing is increased to 48 hours.
  Downhill Cross-country time available
manufacturing time per ski 2 hrs 1 hr 40 hr
finishing time per ski 1 hr 1 hr 32 hr
profit per ski $70 $50  

Answered by Penny Nom.
An equation with exponents 2013-02-24
From Chelsea:
(81 to the power of 3n+2 over 243 to the power of -n) is equal to 3 to the power of 4
Find the value of n.
The answer I got was n is equal to negative (4 over 7).
The answer in my text was n is equal to negative (4 over 17)

Answered by Penny.
Write a rule for the sequence 2013-02-20
From Angelia:
Write a rule for the sequence. Then, find the unknown term. 1 3/8, 1 ¾, 2 1/8, _______, 2 7/8 Think: The pattern is increasing. Add 1/6 to find the next term. Rule: _______________
Answered by Penny Nom.
Extraneous solutions 2013-02-18
From Eileen:
(5x+4)^1/2-3x=0
Answered by Penny Nom.
A word problem involving toys 2013-02-14
From sandy:
Each boy gets 5 toys.Each girl gets 3 toys.There are 150 pupils.
The boys had 74 more toys than girls.
How many boys?
How many girls?

Answered by Penny Nom.
Dropping supplies from an airplane 2013-02-14
From Claire:
An airplane flying at an altitude of 3500 feet is dropping supplies to researchers on an island. The path of the plane is parallel to the ground at the time the supplies are released and the plane is traveling at a speed of 300 mph.
a) write the parametric equations that represent the path of the supplies
c)How long will it take for the supplies to reach the ground?
d) how far will the supplies travel horizontally before they land?

Answered by Penny Nom.
Is a line lying in a plane an open or a closed region? 2013-02-13
From pardeep:
please help with this question on the open and closed intervals :: Is a line lying in a plane an open or a closed region? my teacher says it is a closed region reasoning out that it contains all it boundary points. please help how is it so?
Answered by Harley Weston.
Simultaneous equations 2013-02-10
From Michael:
2P + 1/3V =8
3P - 2/V=5

Answered by Penny Nom.
The height of a sheet metal cone 2013-02-09
From Charles:
Sheet metal cone.
I need a cone with a finished base of 38.19719 diameter The cone is to be from a 48 diameter round with the wedge cut out. The best calculation I have is the arc is 286.479. (correct?)

Could you verify this arc angle but more so what is the cone height?

Answered by Harley Weston.
Exponential form 2013-02-05
From Sandra:
What is 37,008 in exponential form? Help me please!!!
Answered by Penny Nom.
The fourth side of an irregular polygon 2013-02-01
From Emran:
I have a irregular polygon. I know 3 of the 4 sides, and 2 of the angles. A-B is 285, B-C is 149, and C-D is 310. Angle B is 135 degrees. and Angle C is 45 degrees. Is there a formula to solve for the final side? Thanks.
Answered by Penny Nom.
How many edges does cone have? 2013-01-28
From Meriem:
how many edges does cone have pls?
Answered by Lorraine Dame, Chris Fisher and Walter Whiteley.
A geometry word problem 2013-01-24
From Matthew:
"G", "A" and "B" are collinear. "A" is between "G" and "B". "GA" Is 14 less than 3 times "AB". "GB"=46 Find "GA"

How would you solve this?

Answered by Penny Nom.
A triangular island 2012-12-29
From Udit:
A long time ago Mr Gibson found an island shaped as a triangle with three straight shores of length 3 km,4 km and 5 km. He declared an 'exclusion zone' around his island and forbade anyone to come within 1 km of his shore. What was the area of his exclusion zone?
Answered by Penny Nom.
Solve 2x^4+x^3-37x>26x^2+12 using a graphical approach. 2012-12-29
From Tehmas:
Solve 2x^4+x^3-37x>26x^2+12 using a graphical approach.
I don't know how to find the x-intercepts or y-intercepts in this form.

Answered by Penny Nom.
A 4 digit phone lock using the digits 4, 2, 3 and 0 2012-12-24
From Ragu:
I would like to know formula for searching a combination of 4 digit. eg.

I knew my Phone Lock combination is my car plate number 4230. But this time, i used another way around instead of 4230.

But I am sure, there is only this 4 digits only. 4 & 2 & 3 & 0

Answered by Penny Nom.
Sinx=logx+x^2 2012-11-28
From yasmin:
Sinx=logx+x^2
Answered by Harley Weston.
-5<2x+1<-1 2012-11-11
From Rahul:
How does -5<2x+1<-1 changes to 1<|2x+1|<5. What is the rule? Can we apply absolute value function throughout the first inequality i.e. can we say |-5|<|2x+1|<|-1|. But then it becomes 5<|2x+1|<1 which is impossible because 5 is not less than 1.
Answered by Penny Nom.
What's your secret? 2012-11-11
From ryan:
Tuscan school has a fair. At the fair, Pia plays "what's your secret? The game starts with 3 players who are told the same secret. Each of those 3 players tells another 3 player, and so on. At the end of the 3rd round, everyone who knows the secret will compare what they have heard. What number in exponent form represents the number of people who know the secret after 3 rounds? How many people is that?

Thank you very much.

P.S. I got 3 cube. which is 27

But when i map it out I end up with 39.

Answered by Penny Nom.
How fast is the distance between the aircraft and the car increasing? 2012-10-24
From Steven:
At a certain instant an aircraft flying due east at 240 miles per hour passes directly over a car traveling due southeast at 60 miles per hour on a straight, level road. If the aircraft is flying at an altitude of .5mile, how fast is the distance between the aircraft and the car increasing 36 seconds after the aircraft passes directly over the car?
Answered by Penny Nom.
A label to cover a plastic cup 2012-10-23
From Kevin:
I'm trying to make a label to cover the entire outer area or a plastic cup. I know there must be a way to figure out the dimensions needed, but I can't seem to figure it out. The circumference of the bottom of the cup is 21.4cm and the circumference at the top of the cup is 29.8cm. The cup is 14.5cm tall. What should the height of the arc from the plane connecting the two ends of the 21.4cm arc. I attached a diagram where x is the value I'm looking for. I'm guessing there is some simple relationship between the length of a line and the arc needed to turn that line into a perfect circle, but I don't know what it is. Can you figure this out and share it with me? Thanks.

-Kevin

Answered by Penny Nom.
A proof by contraposition 2012-10-19
From Rahul:
I am not able to understand the following,
To prove that if for all e>0, |x|0, then |x|>=e. I understand the approach very well but I do not understand why if |x|=e then |x|>=e. If it is so then why not |x|= Thanks in advance!'
Rahul.

Answered by Penny Nom.
The nth term of a sequence 2012-10-19
From Emily:
I don't understand how to do this question:
Nth term for 9, 7, 5, 3, 1
Is there a fast way to do nth term for this question? Or in fact any question?

Answered by Penny Nom.
Composition of functions and one to one 2012-10-17
From Ariana:
If f o g are one to one function,does it follow that g is one to one? Give reasons for your answers
Answered by Penny Nom.
A tank with an inner walled compartment 2012-10-12
From don:
I have a tank 20 feet diameter, 19' 8" tall with an inner walled compartment that has a 7' 6" radius arc with in the tank. I need to figure out the volume of the inner area and the volume of the larger area.
Answered by Harley Weston.
A word problem involving a fraction 2012-10-12
From Derrick:
If the numerator and denominator of a fraction are both decreased by 1 the fraction becomes 2/3. If the numerator and denominator are both increased by 1 the fraction will be 3/4. Find the original fraction. How to do?
Answered by Penny Nom.
GST in New Zealand 2012-10-10
From Colleen:
I work out my claimable gst each month but need to know the total sales or purchases amounts for filling in my six monthly paper work. How do I do that. I tried doing the opposite of calculating the gst i.e. x3 and /23 but it is slightly different to my total.
Answered by Penny Nom.
A problem that yields a linear equation 2012-10-07
From Eshraj:
one fourth of a whole number is three more than one fifth of the next whole number.find the whole number
Answered by Penny Nom.
Changing the sign of an inequality 2012-09-19
From Bryauna:
Why do you change the signs in inequalities?!
Answered by Penny Nom.
Functions 2012-09-18
From nayeem:
I tried with many functions but I am not getting the exact values please help me
A give an example of a function whose domain equals the set of real numbers and whose range equals the set the set {-1,0,1}?
B Give an example of a function whose domain equals (0,1)and whose range equals [0,1] C.Give n example of a function whose is the set of positive integers and whose range is the set of positive even integers D. Give an example of a function whose domain is the set of positive even integers and whose range is the set of positive odd integers E give an example of function whose domain is the set of integers and whose range is the set of positive integers. F. Give an example of function whose domain is the set of positive integers and whose range is the set of integers.
please show me the work
Please give me the trick of finding such functions

Answered by Robert Dawson and Harley Weston.
Functions 2012-09-18
From nayeem:
I tried with many functions but I am not getting the exact values please help me A give an example of a function whose domain equals the set of real numbers and whose range equals the set the set {-1,0,1}?
B Give an example of a function whose domain equals (0,1)and whose range equals [0,1] C.Give n example of a function whose is the set of positive integers and whose range is the set of positive even integers D. Give an example of a function whose domain is the set of positive even integers and whose range is the set of positive odd integers E give an example of function whose domain is the set of integers and whose range is the set of positive integers. F. Give an example of function whose domain is the set of positive integers and whose range is the set of integers.
please show me the work
Please give me the trick of finding such functions

Answered by Robert Dawson and Harley Weston.
Cones, pyramids, cylinders and prisms 2012-09-13
From Roy:
I read on this page that a pyramid is a special kind of cone, but a cone is not a pyramid. Does this apply to cylinders. Is a prism a special kind of cylinder, but a cylinder is not a prism?
Answered by Robert Dawson.
Row echelon form 2012-09-05
From Jennifer:
Hello, my name is Jennifer. I am a 12th grader in Pre-Calculus and I was wondering if you may help me with Row Echelon Form.

x + 2y - 3z = -5
-2x - 4y - 6z = 10
3x + 7y - 2z = -13

Answered by Penny Nom.
A tangent to f(x) = 1/x 2012-09-04
From Steven:
Consider the graph of the function f(x) = 1/x in the first quadrant, and a line tangent to f at a point P where x = k. Find the slop of the line tangent to f at x = k in terms of k and write an equation for the tangent line l in terms of k.
Answered by Penny Nom.
Exponential form 2012-09-04
From angie:
How do i write 9^-5 in exponential form?
Answered by Penny Nom.
Exponntial form 2012-08-30
From Robin:
My math teacher has asked the question:

"Write mmmxxjmyy2 in exponential form"

How would you solve this problem?

Answered by Penny Nom.
Making a wind sock 2012-08-28
From John:
I am trying to build a wind sock and need to be able to lay the shape out on cloth. I need the wind sock front opening (diameter) to be 3 1/2" and the rear opening diameter to be 1". The windsock needs to be 9 1/2" long. I tried using the example of the person trying to make a crayfish trap but got confused and could not figure out my numbers. Any help would be greatly appreciated.

Thanks

John

Answered by Penny Nom.
Problem solving 2012-08-15
From Sugavanas:
If length of a rectangle exceeds its width by 5 m. if the width is increased by 1m and the length is decreased by 2 m, the area of the new rectangle is4 sq m less than the area of the original rectangle. Find the dimensions of the original rectangle.
Answered by Penny Nom.
The exponential function form f(x)=a^x 2012-08-13
From Lucy:
Hi,
Why does the "a" value in the exponential function form f(x)=a^x have to be negative?

Answered by Penny Nom.
A plane travelling with the wind and against the wind 2012-08-03
From Gwen:
The time of a plane trip of 450 miles, with the wind is 3/5 the time of the return trip against the wind. If the plane travels 120 mph in still air, what is the wind speed?
Answered by Penny Nom.
John's electronic store 2012-07-25
From Jora:
Electronic Store John opened an electronic store in December. During his first month, He sold 10 LCD TVs and 20 Plasma TVs. His income during that month was $12800. In January, he sold 25 LCD TVs and 40 Plasma TVs for an income of $27600. In February he projects to sell 30 LCD TVs and 50 Plasma TVs. If John expenses are $28900 in February, how much money will he have after paying the expenses?
Answered by Penny Nom.
A pile of topsoil 2012-07-23
From Perry:
I have a pile of topsoil that is 42ft long and 16ft tall shaped like a cone how many yards of topsoil do I have.
Answered by Penny Nom.
Fence post holes 2012-07-19
From Gerry:
Hello, I'm digging 30 8" dia holes, 5 ft deep for fence posts that are 4"x 4" Can you please help me figure out how much stone dust I should order for all 30 holes. Thanks
Answered by Penny Nom.
A truncated cone: the central angle 2012-07-17
From Tom:
I have researched several sites, including this one and am fairly confident I can do the calculations required to produce the two radii and the slant height for a truncated cone. I a somewhat confused by the central angle. Some sites indicate that it should never exceed 180 degrees while others do not. Different examples on your site seem to use this in two different ways in constructing a truncated cone. Sometimes the angle seems to indicate the section to be removed, while in other examples the angle seems to indicate the section to be saved. Since the two angles will always total 360 I am confused about how to use the info to calculate the minimum rectangle required to contain the pattern. I'm guessing that in some cases I may need to use the chord or sagitta to determine the desired height and width of the material. Any help is greatly appreciated. Thank you. Tom
Answered by Penny Nom.
2x+underroot2=3x-4-3underroot2 2012-07-09
From Eshraj:
2x+underroot2=3x-4-3underroot2
Answered by Penny Nom.
Expanded exponential form? 2012-06-12
From Ramon:
express the numbers 990 614 000 005 in expanded exponential form.
Answered by Harley Weston.
The position of an aircraft relative to the airport 2012-06-08
From Dennis:
"Air traffic controllers usually describe the position of an aircraft relative to the airport by altitude, horizontal distance, and bearing.  Suppose an aircraft is at altitude 500m, distance 15km, and bearing 35 degrees east of north.  What are the x,y, and z components (in meters) of the position vector. The x-axis is east, the y-axis is north and the z-axis is vertical.
Answered by Penny Nom.
A lamp shade 2012-05-15
From Fleur:
I am helping my child make a cone lampshade, the measurements are as follows in "cm's"
21 cm = height
8 cm = top diametre
40 cm = bottom diametre

Please could you give the pattern (cut out) dimensions for final cut.

Answered by Penny Nom.
cos(theta/30) = 1 2012-05-14
From Hope:
cos (theta / 30 = 1
I am very confused as to how to solve it. Can you help?

Answered by Penny Nom.
An equation of the form y=mx 2012-04-17
From Samiya:
what would be an equation of the form y=mx with the points (-2, 6) and (1, -3) for the line?
Answered by Penny Nom.
Using Newton's Method to find a root 2012-04-09
From Nancy:
Use Newton's method to find the real root function, accurate to five decimal places

f(x) = x^5+2x^2+3

Answered by Penny Nom.
The spread of a rumor 2012-04-09
From Roohi:
The function f(t) = a/(1+3e^(-bt)) has also been used to model the spread of a rumor. Suppose that a= 70 and b=3 0.2. Compute f(2), the percentage of the population that has heard the rumor after 2 hours. Compute f'(2) and describe what it represents. Compute lim t approaches infinity and describe what it represents.
Answered by Penny Nom.
Radioactive decay 2012-04-02
From Christy:
Hello, how do I go about answering this question?

Should I be using the formula Ce^(kt)?

Given that the decay constant for Radium is -.000428/year, how long, to the nearest year, does it take a sample to decay to 15% of its present mass?

Answered by Penny Nom.
The equation of a line 2012-03-26
From Danielle:
Hello, could you help me figure out what the equation of the line is in the form of Ax+By=C with the points (-2,0) and (0,3)? Thank You!
Answered by Penny Nom.
A cubic inequality 2012-03-25
From Victoria:
The flight path of a bumblebee above the ground can be modelled by the function f(x)= 2x^3- 17x^2+ 11x + 130. Where x is the time in seconds and f(x) represents the height in inches above the ground. The entrance to the bee's hive is located 100 inches above the ground. Determine when the bumblebee's height is greater than 100 inches. The bee's height was monitored from 0 to 25 seconds inclusive. (over the domain 0
Answered by Penny Nom.
A vertical line passing through (10,4) 2012-03-16
From Isabelle:
Hi! My question is this:

Write the equation of each line described:
A vertical line passing through (10,4)

I would really appreciate your help!
Thanks,
Isabelle

Answered by Penny Nom.
Forming a triangle from 3 line segments 2012-03-15
From rustom:
A point X is selected at random from a line segment AB with midpoint 0. Find the probability that the line segments AX, XB, and A0 can form a triangle.
Answered by Penny Nom.
A water tank is in the shape of a truncated cone 2012-03-08
From Victoria:
Suppose you have a water tank in the shape of a truncated cone. The larger diameter is K, the smaller diameter is K/2, and the height is also K (all measured in meters). The force of gravity on an object of mass, m, is F=9.8m (measured in Newtons). The density of the water is 1000 kilograms per cubic meter. What is the volume of the tank and how much mass will the tank have when it is full?
Answered by Penny Nom.
81+8 to the 3 power 2012-03-08
From KiMesha:
81+8 to the 3 power
Answered by Penny Nom.
2x^3*2x^2/3x^2 2012-03-06
From hope:
2x^3*2x^2/3x^2
Answered by Penny Nom.
The dimensions of a conical tent 2012-03-04
From yash:
a conical tent is to accommodate 11 people.Each person must have 4m square of space on the ground and 20m cube at air to breathe.Find the height and radius of the conical tent.26202
Answered by Penny Nom.
Integral 1/(25-x^2)^3/2 2012-02-22
From John:
Integral 1/(25-x^2)^3/2
Answered by Harley Weston.
A cylindrical container 2012-02-14
From KRIS:
YOU ARE PaCKAGING 4 EQUAL SIZED PAINT CANS WITH A RADIUS 8CM AND A HEIGHT OF 20CM, IN A CIRCULAR CONTAINER.What is the surface area of the packing container?
Answered by Penny Nom.
The curvature of the earth 2012-02-08
From sean:
Question from sean, a student:

Two people 1.8 metres tall walk directly away from each other until they can no longer see each other (due to the curvature of the earth, which has a radius of about 6378 km).
A) Find a function relating the height of two identical objects with the distance between them using the scenario above as an example.
B) Sketch this function (you may use Graphmatica if you wish). Over what domain and range does the function exist?
C) Describe this relation in practical terms.

Answered by Harley Weston.
Building a tipi 2012-01-29
From Lacy:
Hi there! We are building a tipi for our children. We want to build a large one about 15ft tall with a base of about 15 feet diameter. I am trying to figure out how much canvas we need to accomplish this. I graduated about 20 years ago and am struggling. Please help if you can.
Answered by Penny Nom.
Tangent of theta 2012-01-17
From stahl:
explain what the 'tangent of theta' means. Draw and label a diagram to help with your explanation.
Answered by Harley Weston.
Exponential form 2012-01-15
From Helen:
How do you write (1/x) in exponential form?
Answered by Penny Nom.
Four apples and two oranges cost Rs. 30... 2012-01-13
From nasr:
Four apples and two oranges cost Rs. 30, and one apple and 3 oranges costs Rs.15.How much does each apple and each oranges cost?
Answered by Harley Weston.
An equilateral triangle and some circles 2012-01-10
From tushar:
draw an equilateral triangle with side 6cm.draw 3circles with radii 3cm on each angular point of triangle.draw common tangent on each of two circles
Answered by Penny Nom.
A straight line distance 2012-01-06
From Margaret:
If you traveled 300 miles east and 275 miles north, how many miles would you save by going in a straight line?
Answered by Penny Nom.
The volume of a frustum of a cone 2011-12-24
From CV:
If I know the height, taper, and volume of a frustum cone, what are its Radii?

Calculating frustum cone volume is straight forward.
v=Pi/3*h*(R*R+r*r+R*r) where;
v=volume;
h=frustum height;
R=major radius;
r=minor radius;
Pi=3.1415926;
t=taper, ('slant angle' where t=0 is a disk)

Here is the tricky part.
Knowing 'h', 't', & 'v';
Calculate one or both of 'R' & 'r'.

Specifically, what is the formula for 'R=' or 'r='?

Answered by Penny Nom.
Three sides of a triangle 2011-12-24
From saba:
the three sides of a triangular lot have lengths 10,11and 13cm,respectively. find the measure of its largest angle and the area of the lot?
Answered by Penny Nom.
Water is flowing into a cup 2011-12-19
From Tim:
A cup has a radius of 2" at the bottom and 6" on the top. It is 10" high. 4 Minutes ago, water started pouring at 10 cubic " per minute. How fast was the water level rising 4 minutes ago? How fast is the water level rising now? What will the rate be when the glass is full?
Answered by Penny Nom.
Augmented matrix 2011-12-14
From Britney:
Write Augmented matrix and use row reduction to solve.
I need major help with this problem! Tomorrow is my semester test and I have no idea how to do this.
2x+5y=8
y-x=10

Answered by Robert Dawson.
Exponential form 2011-12-12
From Zoey:
What is the exponential form of 1/8??
Answered by Penny Nom.
A railway bridge over a road is in the shape of a parabola 2011-11-23
From Brennen:
A railway bridge over a road is in the shape of a parabola, and the bridge is 3 m high in the middle and 6 m wide at its base. A truck that is 2m wide is approaching the bridge. What is the maximum height that the truck can have and still pass under the bridge? Explain.

Thank you!!

Answered by Penny Nom.
I started with Euler's identity and manipulated it 2011-11-14
From anonymous:
I started with Euler's identity and manipulated it
e^i*pi=-1
e^-i*pi=(-1)^-1
e^-i*pi=-1
e^-i*i*pi=(-1)^i
e^--pi=(-1)^i
e^pi=(-1)^i
type it in in a calculator and you get e^pi=23.1406926... and (-1)^i=0.0432139183... What did I do wrong?

Answered by Robert Dawson.
Lines tangent to y^2=4x 2011-11-11
From Reuchen:
Find equations of the lines tangent to y^2=4x and containing (-2,1).
Answered by Penny Nom.
A log equation in exponential form 2011-11-11
From Sarah:
Express the equation in Exponential form

log2 to the 16th power=4

Answered by Penny Nom.
The slope of a line 2011-11-10
From Sara:
Line k lies in the xy-plane.
The x-intercept of line k is -4.
And line k passes through the midpoint of the line segment whose endpoints are (2,9) and (2,0).
What is the slope of line k?
Give the answer as a fraction.

Answered by Penny Nom.
A spherical ball in a conical wine glass 2011-10-26
From Jules:
A heavy spherical ball is lowered carefully into a full conical wine glass whose depth is h and whose generating angle (between the axis and a generator) is w. Show that the greatest overflow occurs when the radius of the ball is (h*sin(w))/(sin(w)+cos(2w)).
Answered by Claude Tardif.
Linear equations 2011-10-06
From Hannah:
I am confused about a question on my homework, here is an example on what I need to do:

You own a sightseeing boat tour. Kids' admission costs $5, adults' admission costs $8. During that day you get (made-up number) $521 with 71 admissions. How many of each passenger boarded?

I tried something like
5x + 8y = 521
x + y = 71

But, I still don't understand how to solve it! Help?

Answered by Penny Nom.
Exponential form? 2011-08-30
From Emily:
I have a 6th grader that needs to find out what is the exponential form of 461,885,811
Answered by Penny Nom.
A pattern for a cone 2011-08-30
From Izzy:
I need to create a cone as a prop and my math is not good enough to create the pattern. Here are my instructions :
The diameter is 2'5" and it has to be 4 feet tall. It is a giant cone. I want it to be pointy at the end, not truncated.

Answered by Penny Nom.
Exponential form 2011-08-23
From kiara:
The African bush elephant is the largest land animal and weighs about 8 tons . Write the amount in exponential form .
Answered by Robert Dawson.
Two line segments in the plane 2011-08-15
From Tim:
For this problem I actually have tried to visualise the image in my head many times. This question makes my head spin.

Four points lie in a plane. They are partitioned into two pairs so that the sum of the lengths of the segments joining the points of each pair has the minimal possible value. Prove that these segments have no common points.

Answered by Chris Fisher.
A sequence 2011-08-14
From intesar:
how do i figure this out 0,1/6,2/9,3/12
Answered by Penny Nom.
The third term of a sequence 2011-08-13
From sammantha:
Find the 3rd term of the sequence, an=1.5(4)n - 1.
Answered by Penny Nom.
A cone with an oval as a base 2011-08-03
From Emily:
Hi, I was wondering how to calculate the surface area of a cone with an oval as a base (which I think is referred to as an elliptic cone or something like that). I have the both the maximum and minimum radius as well as the height, but I don't have a slant height and I'm not sure how to calculate it and then calculate surface area. I would really appreciate some help. Thanks!
Answered by Robert Dawson.
An equation in the form Ax+By=C 2011-07-28
From Mike:
I need to write an equation in the form Ax+By=C The points I was given are (5,0) and (-3,5)

I know how to find the slope I got -5/8 and wrote it out as y=(-5/8)x+b however I'm not quite sure where to go from here.
Any help would be greatly appreciated.

Answered by Penny Nom.
A stained glass lamp 2011-07-25
From Guy:
Like Kay, I also work in stained glass, but in 3-dimensions. I am frequently asked to replicate lamp shades in stained glass where the diameter of the top is different (narrower) from the diameter of the bottom (which is wider). Some people want 5, 6, 7, 8, 10, 12, 16, ,,, n-sided shades. Is there a formula I can use to determine the width of the sides using the angle, if I remember correctly, I think it's called theta. In other words, is there a formula where I can plug in the angle which describes the arc of the circle. For instance, if someone asks for a 7 sided shade, plugging in 51.43 (360/7). I could then use that to determine the width at the top and bottom rings to create the appropriate trapezoids. I've visited a few sites so the formula looks like its a function of sin & cos but they are presented like proofs for teaching. Your site appears to want to actually answer questions without making the inquirer feel stupid.
Answered by Harley Weston.
A linear equation with fractions 2011-07-13
From Simon:
I would like to ask if you would be able to explain : linear equation: 2 1/2 (X-1) - x+3 /3 = 4 , the first step shows multiplying by 2 then multiplying by 3. etc. What I don't understand is where the 2 in multiplying by 2 comes from? ( the three is pretty obvious being x+3 /3 (to get rid of the divided by) ) .
Thanks for your help.

Answered by Penny Nom.
A line tangent to f(x)=1/x 2011-06-05
From Michael:
A line tangent to f(x)=1/x in the first quadrant creates a right triangle with legs the x-axis and the y-axis. Prove that this triangle is always 2 square units regardless of where the point of tangency is.
Answered by Penny Nom.
Exponential form 2011-05-18
From Kara:
How do I write this in exponential form?

log 1/8 = -3

Answered by Penny Nom.
Numbers with a digit 9 2011-05-13
From Veronica:
How many numbers between 200 and 750 have a 9 for at least one of the digits?
Answered by Penny Nom.
An exclusion zone around a triangle II 2011-05-03
From Aishwarya:
A long time ago Mr gibson found an island shaped as a triangle with three straight shores of length 3km, 4km and 5km. He declared an exclusion zone around his island and forbade anyone to come within 1km of his shore. What was the area of his exclusion zone? This question was answered but did not understand the explanation.
Answered by Robert Dawson.
The nth term 2011-04-30
From Samantha:
predict the general term, or the nth term, An for the sequence.
1/2,2/3,3/4,4/5,5/6

Answered by Penny Nom.
The radius of a cylinder 2011-04-27
From Jazmin:
Hi, I don't understand how to find the radius in a cylinder with only the surface area (143.7) and the height (0.8)? I know that the formula is 2pir2+2pirh, but I don't see how to isolate the r? I appreciate your help.
Answered by Penny Nom.
(27^r^-1)^-2/3 2011-04-24
From Yung-Ju:
(27^r^-1)^-2/3
Answered by Penny Nom.
Cone story problem 2011-04-16
From Chelsea:
There is a cone with a story problem that states only that the radius is the same as the altitude and that the slant height of the cone is 6cm. I have no idea how to figure this problem out.
Answered by Penny Nom.
The volume of a flower pot 2011-04-07
From kp:
I have 2 flower pots with the following dimensions that I am trying to calculate the volume of Soil I will need to fill them. pot #1 29"tall, top of pot 31.5 inches across( radius of 15.5") the bottom of the pot is 21"across (radius 10.5) Pot #2 29"tall, top of pot 26 across (radius 13") the bottom of the pot 17'"a cross (radius 8.5")
thank you
KP

Answered by Penny Nom.
Eliminate y 2011-04-07
From Lynn:
2x + y = 8
y + 3z =5
z + 2w =1
5w + 3x = 9

Form three equations with y eliminated

Answered by Penny Nom.
A conical container and a spherical balloon 2011-04-06
From Steven:
Water is running out of a conical container 12 feet in diameter and 8 feet deep (vertex down) and filling a spherical balloon. At the instant the depth of the water in the cone is 4 feet, the radius of the sphere is approximately 4 feet.

The rate of change of the depth of the water in the cone at the instant is approximately ______________ times the rate of change of the radius of the balloon.

Answered by Penny Nom.
A transversal 2011-03-30
From Jillian:
I don't understand finding angels in a transversal
Answered by Penny Nom.
sin x = -0.25 2011-03-29
From Wayne:
How do you solve for x in the equation sin x = -0.25

the answer is 3.394 and 6.030 but I don't know the steps they used to calculate this

Answered by Penny Nom.
New Brunswick hst 2011-03-25
From andy:
what is the short formula to get the hst 13% out of the total price of an item
Answered by Penny Nom.
Linear inequalities 2011-03-25
From Alice:
OK. I'm in algebra 1 and the linear inequality is
y<2x-1
y>2
I don't completely know how to graph y>2 and I'm getting really frustrated
Please help me in any way possible!
Thank You!

Answered by Penny Nom.
The number of points on a line is equivalent to that of a surface 2011-03-24
From Gary:
I I was reading about how the number of points on a line is equivalent to that of a surface. This was done by taking any point on a line then taking alternating digits and making them as points on an x and y axis therefore points on a surface.The problem is as i see it if you just take a line then hold it over a surface you have just put the points on the line in a one to one correspondence with the points directly under it on the surface.Now you have all the rest of the surface which cannot be mapped onto the line since it is already used up.What am i missing?
Answered by Penny Nom.
What is x to the power of 0? 2011-03-23
From Jason:
What is x to the power of 0?
Answered by Robert Dawson.
6 faces and 8 corners 2011-03-04
From noemi:
jeff made a figure with 6 faces the figure has 8 corners all of the faces are the same size and shape what figure did jeff make
Answered by Penny Nom.
A camera's line of sight 2011-02-26
From MJ:
A rocket that is rising vertically is being tracked by a ground level camera located 3 mi from the point of blast off when the rocket is 2 mi high its speed is 400mph At what rate is the (acute) angle between the horizontal and the camera's line of sight changing
Answered by Penny Nom.
At what rate is the grain pouring from the chute? 2011-02-26
From MJ:
Suppose that grain pouring from a chute forms a conical heap in such a way that the height is always 2/3 the radius of the base. At the moment when the conical heap is 3 m high, its height is rising at the rate of 1/2 m/min. At what rate (in m^3/min) is the grain pouring from the chute?
Answered by Penny Nom.
Cutting the top off a conical tent 2011-02-22
From tom:
how far from the top must you cut a conical tent in order to cut the cloth in half...
Answered by Penny Nom.
Making a truncated cone 2011-02-18
From lisa:
We need to make a cone that has the following dimensions.
14-3/8" diameter on large end
13-3/8" diameter on small end
4" tall

What are the dimensions I need to cut in order to make a cone with one seam?

Answered by Stephen La Rocque.
Two conical tanks 2011-02-17
From rustom:
Two vertical conical tanks (both inverted) have their vertices connected by a short horizontal pipe. One tank, initially full of water, has an altitude of 6 ft. and a diameter of base 7 ft. The other tank, initially empty, has an altitude of 9 ft., and a diameter of base 8 ft. If the water is allowed to flow through the connecting pipe, find the level to which the water will ultimately rise in the empty tank (Neglect the water in the pipe.)
Answered by Penny Nom.
Points on a graph 2011-02-15
From Maggie:
Which set of ordered pairs can be used to graph y = -2x-1.

A.(-5,9),(1,-3),(4,9)
B. (-2,-5),(1,-1),(5,9)
C.(-2,-5),(0,-1),(4,-3)
D. (-5,9),(-2,3),(4,-9)

I cannot figure it out. Could you help me?

Answered by Penny Nom.
The equation of a circle 2011-02-14
From Cristela:
find the equation and all the information in General Form and Standard Form of the Circle that will passed trough the point (2,3) (6,1) (4,-3)
Answered by Stephen La Rocque.
Calibrating a conical tank 2011-02-05
From Bill:
Hi, I have a round tank with tapered sides where I know the diameter at the top and bottom. Is there a formula I can use to calculate the volume by measuring from the bottom up the side (at the angle of the side) to any given point? Thanks, Bill
Answered by Stephen La Rocque and Penny Nom.
The nearest one and a half and the nearest inch 2011-02-02
From Miranda:
REASON:sarah gave the same answer when asked to round 4 seven eighth inches to the nearest one and a half and the nearest inch. explain why sarah is correct?
Answered by Penny Nom.
A cone with a specific angle 2011-02-01
From John:
Hi my daughter came with a seemingly easy question (which to me it was not) How to make/calculate a cone of a specific angle from top to bottom radius.
Answered by Penny Nom.
(7cubed * 8(exponent6))exponent 6 2011-01-29
From Shalaine:
What is the exponential form of (7cubed•8(exponent6))exponent 6?
Answered by Penny Nom.
The sequence 2,3,5,5,8,7,11 2011-01-19
From taylor:
number relationship in number sequence 2,3,5,5,8,7,11
Answered by Chris Fisher and Penny Nom.
Points with distance 5 from the point (-2, -1) 2011-01-19
From Alexa:
Find all points having a x-coordinate of 2 whose distance from the point (-2, -1) is 5.
Answered by Penny Nom.
Flipping a cone over 2011-01-17
From Fionna:
The height of your cone is "x". Holding the cone so that the vertex is at the bottom, fill it half full with water. The depth of the water is "1/2x". Put a lid over the top, and flip it over. What is the new depth of the water, now that the cone is flipped?
Answered by Penny Nom.
What is next in the sequence? 2011-01-13
From jodi:
what is next in the sequence 1, 1/4, 1/9, 1/16, 1/25
Answered by Penny Nom.
The angle between a line and a plane 2011-01-12
From tom:
what are the angles of the diagonal of a rectangular parallelepiped 2in by 3 in by 4 in makes with the faces...
You know this is a problem that I can't figure out ...I don't know where the angle and the diagonals where? can you help me with this one?

Answered by Penny Nom.
Line segments 2011-01-04
From Izsak:
What is the formula to find the number of segments that can be named by a given number of points on a line?
Answered by Penny Nom.
A tangent line 2011-01-03
From Amanda:

Question from Amanda, a student:

an equation of the line tangent to y=x^3+3x^2+2 at its point of inflection is
(A) y=-6x-6
(B) y= -3x+1
(C) y= 2x+10
(D) y=3x-1
(E) y=4x+1


Answered by Penny Nom.
Exponential form 2011-01-02
From jumana:
find any five examples of exponential form from real life situation
Answered by Penny Nom.
A truncated cone 2010-12-31
From jagjeet:
dear suppose i have cone of 50cm radius at base and 100cm radius at top at a height of 80cm. now keeping base radius of 50cm and height of 80cm i want to increase top radius to 150cm how will i do that. plz height m talking abt is vertical height not slant height
Answered by Penny Nom.
An inequality 2010-12-13
From jimmy:
1.7b-1.1 <2.3
Answered by Stephen La Rocque.
A water tank has the shape of a right circular cone 2010-12-07
From mike:
A water tank has the shape of a right circular cone with height 12 feet and radius 8 feet. Water is running into the tank so that the radius r (in feet) of the surface of the water is given by r=0.75t where t is the time (in minutes) that the water has been running. the volume V of the water is given by V=1/3 pi r^2h. Find V(t) and use it to determine the volume of the water when t=5 minutes.
Answered by Penny Nom.
Simultaneous equations 2010-12-05
From ryan:

Question from ryan, a student:

3         4
--   -    --      =     1      (1)
x         y

7       2             11
--   -  --        =   --       (2)
x       y              12


Answered by Chris Fisher and Stephen La Rocque.
Can determine if it is scalene, isosceles, or equilateral 2010-12-01
From Jessie:
find the measures of the sides of triangle KPL and classify each triangle by its sides. my first problem would be K(-3,2) P(2,1) L(-2,-3) ...The three points they give you are the vertices of the triangle and you need to match them up. Draw the triangle and write in the vertices and the related point with the vertex. You will then do the distance formula three times to find the distance of all three sides. Once you have the three sides you can determine if it is scalene, isosceles, or equilateral...using the distance formula how do i solve this?
Answered by Penny Nom.
Linear inequalities with absolute values 2010-12-01
From ryan:
Good day!
We we're given this assignment in algebra concerning linear inequalities with absolute value. In this number I'm confused how to isolate the absolute value, considering there is a variable in the absolute value and there is a variable in the outside.

2+5 |4-3t|+t ≤ -3-4t

Answered by Penny Nom.
What is its speed in still air? 2010-11-28
From Shelby:
An Airplane flies a certain distance in 3 hours going WITH a 40km/hr wind. It travels the same distance in 5 hours when going against the wind. What is it's speed and how long would it take to travel the same distance in still air? Distance = speed x time
Answered by Penny Nom.
How long will you and your friend be able to talk? 2010-11-23
From beket:
you and your friend leave your home town at the same time your friend travels by train 45 mph directly east while you travel by train 50 mph going directly south you are talking on cell phone but you plan charges extra if the phones are 100 miles apart. How long will you and your friend be able to talk before the charges increase?
Answered by Penny Nom.
How far must the pitcher travel to get to the ball? 2010-11-04
From ken:
A baseball player bunts a ball down the first base line. It rolls 35ft at an angle of 26 degrees with the first base path. The pitchers mound is 60.5 ft from the plate. How far must he travel to get to the ball.
Answered by Penny Nom.
What is the speed of each car? 2010-11-04
From Jennifer:
two cars are 420 miles apart and traveling towards each other along the same road. They meet in 3.5 hours. One car is traveling 15mph slower than the other. What is the speed of each car? This must be solved using a system of equation, I have no idea?
Answered by Stephen La Rocque.
Two algebraic expressions 2010-10-29
From chaptay:
what is the difference between x(n^2) and x(n)^2
Answered by Penny Nom.
A rectangle with rounded corners 2010-10-21
From Connie:
How do you find surface area if the rectangle is 100ft. by 200ft. with quarter circles in the corners with a radius of 20ft. ?
Answered by Penny Nom.
Powers 2010-10-20
From dylan:
how do you write 20736 in exponential form .same for 1728 and 50625.

is there a formula to figure out how to express large know numbers in exponential form.

Answered by Penny Nom.
An exponential equation 2010-10-19
From Sandi:

Question from sandi, a student:

I have several questions similar to this one and was wondering if you could walk me through this one. I'm totally lost on how to do it.
Paramecia reproduce by splitting in two. In a laboratory flask, a colony of paramecia had an initial population of 500. Each day, the population of the paramecia was counted. The results are as listed.
Time (in days)------Population
0-----------------------500
1-----------------------600
2-----------------------720
3-----------------------864
4----------------------1037
5---------------------1244
6---------------------1493
7----------------------1792
8---------------------2150
1.)Using graphing calculator make a scatter plot of the data in table.
I think I did this part right I set my window at Xmin=0 Xmax=10 Xscl=1 Ymin=0 Ymax=2500 Yscl=100 Xres=1
2.) Determine an exponential equation to represent the population as a function of time without using a graphing calculator.I have no clue how to do this.
3.)Suppose the flask and food supply is large enough to support the trend of the population growth. Estimate the population of the colony when the time is 10 days.


Answered by Penny Nom.
The equation of a line 2010-10-19
From Sava:
I have to write an equation and sketch a graph for the line that meets the given conditions:

A line with slope -15/5 that passes through the point (-2.5,4.5)

Answered by Penny Nom.
The angles in an m-gon and genrealizations 2010-10-16
From Michael:
Hello: In answer to a student's question, someone named Penny from your organization provided a proof that the sum of the interior angles of a triangle in the plane is pi radians (or 180 degrees).

I am interested (and I'm sure many other people would be too) in 3 potential generalizations of this basic fact in plane geometry:

Answered by Walter Whiteley.
A linear system 2010-10-15
From Sandy:
HARD!! algebra (Linear Systems) question? An Airplane cruises at 120km/h in still air. One day, when the wind was blowing steady from the west, the airplane travelled west (flying into the wind) and landed at its desination after 1.5h. The airplane's return trip (flying with the wind) was only 1.0 h long. On both legs, the airplane travelled at its cruising speed. (meausred with respect to the air surrounding the airplane).

Part A
Write a System of equations that would allow you to determine the speed of the wind blowing that day and the distance travelled each way

Part B
When solving the system of equations graphically, what equations would you enter into the equation editior (to make graphs on your calculator)

Part C
Indicate what the X and the Y represent

Answered by Stephen La Rocque.
Two jets leave denver at 9 am. 2010-10-15
From Carl:
two jets leave denver at 9 am. One flying east at a speed of 50 km per hour greater than the other which is traveling west. At 11 am the planes are 2500 km apart. Find their speeds.
Answered by Stephen La Rocque.
Mathematics and medicine 2010-10-13
From Dixie:
need equations used in medicine for medicine and intravenous administration
Answered by Penny Nom.
Can a function be both even and one to one? 2010-10-09
From Tracy:
Can a function be both even and one to one?
Answered by Harley Weston.
Exponential form 2010-10-06
From celina:
what is the exponential form of 4x*4x
Answered by Penny Nom.
rat sequence 2010-09-24
From Vishal:
this is known as rat sequence. I want to know the logic pf this :-

1,2,4,8,16,77,145

Answered by Chris Fisher.
A Squared Number That's Negative 2010-09-22
From David:
What is the only number that when it's squared becomes negative?
Answered by Stephen La Rocque.
Limits 2010-09-20
From rodrigo:
what's the limit of x+1/x-1, when x tends to 1?
Answered by Janice Cotcher.
Six nines 2010-09-16
From Steph:
It's sort of one question. We have to use six nines to get the numbers 1-30. I got all but 22, 23, 24, and 30. Like 15 is 9+9-(9+9+9)/9.
Answered by Penny Nom.
A strip of land 2010-09-15
From Walter:
How many linear feet of strip of land 15 feet wide does it take to make 1 acre?
Answered by Penny Nom.
La racine carrée et l'exposant une demie 2010-09-14
From Alain:
Bonjour. Je cherche une explication sur l'équivalence entre les exposants fractionnaires et les racines nième. Par exemple, comment prouve-t'on que la racine carrée correspond à  l'exposant une demie? merci
Answered by Pierre-Louis Gagnon et Claude Tardif.
(5xy)^5 2010-09-10
From ana:
(5xy)^5
Answered by Penny Nom.
A building and a flag pole 2010-09-09
From paul:
A flag pole and a building stand on the same horizontal level. From the point p at the bottom of the building,the angle of elevation of the top t of the flag pole is 65 degrees. From the top q of the building the angle of elevation of the point t is 25 degrees.If the building is20 meters high. Calculate the distance pt
Answered by Penny Nom.
A logarithmic equation 2010-09-08
From Rohit:
x^2 + k*ln(x) - c - k = 0

Where k and c are constants.

Answered by Penny Nom.
What is -2^2? 2010-08-25
From alex:
I know that (-2)^2 is 4 but what is -2^2?
Answered by Robert Dawson and Penny Nom.
Exponential form 2010-08-24
From Courtney:
What the Exponential form? For Ex. X2*X5*X4??
Answered by Penny Nom.
Remainders 2010-07-17
From GARY:
WHAT NUMBER DIVIDED BY 11 LEAVES 1, DIVIDED BY 13 LEAVES 8, AND DIVIDED BY 17 LEAVES 3.
Answered by Chris Fisher.
A play tent 2010-06-28
From Susan:
Hi!!! I am making a play tent as seen at the link below and need to figure out how to get the dimensions for the cone shape. The one shown has 4 different seams, but I guess I can get away with just one seam to sew it together (?) I need it to go over a hula hoop as that is what I am using for the round support at the top. My hula hoop is 35" in diameter from outside edge to outside edge. I would like the height of the teepee to be around 30" from the center to the peak. Thanks so much, oh my gosh, I have been fretting for 2 days about this and my hair is about to fall out!!!! Please contact me if you need any additional info.
~Susan

http://www.landofnod.com/family.aspx?c=52&f=4100

Answered by Penny Nom.
The intersection of two graphs 2010-06-27
From Austin:
In depth explanation of how to find the intersection points of functions y=-4t+300 and y=-4.9(t-5)^2+300
Answered by Stephen La Rocque.
Volume of a styrofoam cup 2010-06-23
From Stacy:
how do you find the volume of a truncated cup with height of 3, top diameter of 2.5, and a bottom diameter of 2
Answered by Penny Nom.
A linear relationship 2010-06-22
From Laura:
The monthly cost of driving a car depends on the number of miles driven. Lynn found that in May it cost her $300 to drive 480 miles and in June it cost her $460 to drive 800 miles.

Question: Express the monthly cost C as a function of the distance driven d, assuming that a linear relationship gives a suitable model.

Answered by Robert Dawson.
Exponential form 2010-06-16
From charlotte:
what is the exponential form of 6x6x6x5x5x5x5=
Answered by Robert Dawson.
My phone is "sim locked" 2010-06-10
From Daniel:
Question from daniel:

got a phone that says sim locked, it requires 0-9 digit code to get in, how many combinations could there be if you could please list the possibilities, the number can be repeated.

Answered by Penny Nom.
Graphical Representation of Complex Numbers 2010-06-08
From Anas:
why do we write complex number a+ib as (a,b)?
Answered by Janice Cotcher.
4 > -3m-7 >= 2 2010-06-05
From pamela:
The question is
4>-3m-7>=2

Answered by Penny Nom.
graph y=(2x^2-3x)e^ax 2010-05-31
From James:
graph y=(2x^2-3x)e^ax
Answered by Robert Dawson.
x/a +y /b =a+b : x/a^2+ y/b^2 =2 2010-05-30
From smithu:
x/a +y /b =a+b : x/a2+ y/b2 =2 solve by using elimination method , cross multiplication, substitution method
Answered by Penny Nom.
More on a truncated cone 2010-05-28
From Mike:

Question from Mike, a parent:

I was reviewing this question and answer:
http://mathcentral.uregina.ca/QQ/database/QQ.02.06/phil1.html

But I have trouble with this part:
Now if we express the radius of the inside circle as r and the outside circle's radius is R, then this means r/R is 911/1728. But earlier we said that the outside radius R is simply w more than the inside radius r, so R = r + 282. That means that r/R = r/(r + 282). Now we can simply solve the equation for r:
r/(r+282) = 911/1728
This means r = 314 mm (with rounding).

Can I get more detail on the method to solve for r?

Thank you,
Mike


Answered by Penny Nom.
Graphing a linear inequality 2010-05-26
From Valeie:
When graphing a linear inequality, how do you know if the inequality represents the area above the line.
Answered by Penny Nom.
Which values of x satisfy (x-2) / (x+4) <7 2010-05-23
From Rocco:
Which values of x satisfy (x-2) / (x+4) <7
Answered by Harley Weston.
Extraneous solutions 2010-05-22
From Joe:

Question from Joe, a parent:

w+3    2w
----- - ----- = 1
w2-1   w-1

W2 is = w squared

                            -4
The answer is w= ------
                             3

but have no idea how this was solved. any help is appreciated. Thanks.


Answered by Penny Nom.
Line of sight 2010-05-06
From David:
I live in St. Joseph, Michigan and there is an ongoing argument regarding line-of-sight over the horizon.

Standing on a 200 foot high bluff here, people swear they can see the top of the Willis (nee Sears) Tower in Chicago, which is about 1653 feet high.

It is my contention that this is actually a "refracted reflection" and not direct-line-of-sight.

So, to settle the argument, I'd sure like some simple explanation for this, even if-and I hope not-I am incorrect.

David

Answered by Harley Weston.
The "vertex" of a cone 2010-04-27
From Tom:
By definition a vertex is a point where three edges meet in a 3 dimensional object. My ten year old son argues that the point at the top of a cone is not a vertex since it does not fit the definition. He got the answer wrong on a test recently but insists that he is right. I need a mathematician to answer this for him.
Answered by Robert Dawson and Chris Fisher.
A champagne tower 2010-04-16
From gabriel:
thanks for having something like this up online. My question is how do i build a Champagne Tower that has 290 glasses and 15 stories high.
Please help me out. this is for a clarity function.

Answered by Robert Dawson.
A tangent line to a circle 2010-04-15
From Rhonda:
The Greek method for finding the equation of the tangent line to a circle used the fact that at any point on a circle the line containing the reauis and the tangent line are perpendicular. Use this method to find an equation of the tangent line to the circle x^2+y^2=9 at the point (1,2 square root of 2).
Answered by Penny Nom.
A sequence 2010-04-13
From Essence:
hello, I am having trouble doing my math homework. It asks me to write the first five terms of each sequence .I can not figure it out because it is a fraction.

The Problem Is:

{2n+1/2n}

Answered by Penny Nom.
A conical pile of gravel 2010-04-13
From Chassity:
The gravel pile is 120' around at the base and goes up 20' high at the peak. How many tons or yards of gravel in that pile?
Answered by Penny Nom.
Exactly two lines of symmetry 2010-04-11
From debbie:
i am looking for a quadrilateral with exactly two lines of symmetry. please help! thank you.
Answered by Tyler Wood.
If a third is a fourth of a number, what is the number? 2010-04-08
From larry:
If a third is a fourth of a number, what is the number?
Answered by Penny Nom.
A max min problem 2010-04-06
From Terry:
The vertex of a right circular cone and the circular edge of its base lie on the surface of a sphere with a radius of 2m. Find the dimensions of the cone of maximum volume that can be inscribed in the sphere.
Answered by Harley Weston.
A system of equations 2010-03-30
From samantha:
the equations of two lines are 6x-y=4 and y=4x +2.what is the value of x in the solution for this system of equations?
Answered by Penny Nom.
The sides of a 30-60-90 triangle 2010-03-29
From maris:
Find the sides of a 30-60-90 triangle whose perimeter is 6. The solution must use systems of equations using 3 variables.
Answered by Penny Nom.
A radio tower 2010-03-26
From Alex:
The height of a radio tower is 450 feet, and the ground on one side of the tower slopes upward at an angle of 10 degrees. How long should a guy wire be if it is to connect to the top of the tower and be secured at a point on the sloped side 110 feet from the base of the tower?
Answered by Harley Weston.
A negative times a negative 2010-03-25
From priya:
why is minus into minus plus?
Answered by Harley Weston.
The angle of intersection between two lines 2010-03-25
From Madhumitha:
How to find the angle of intersection between a line x=249 and another line which is of the form y=m*x+c; Line1 has a slope of infinity so what does the angle of intersection turn out to be? Or how can i find it?
Answered by Chris Fisher.
The line passing through (2,6) and (2,9) 2010-03-24
From antoinette:
the equation of a line passing through the points (2,6) and (2,9)?
Answered by Penny Nom.
A champagne pyramid 2010-03-22
From Kathy:
I have 680 champagne glasses and my pyramid can only be 15 levels high. How many glasses need to be on each level? Is there a formula I can use?
Answered by Robert Dawson.
A quadrilateral with 4 known sides and 1 known angle 2010-03-19
From samuel:
Name: Samuel
Status: Student

I have a quadrilateral with 4 known sides and 1 known angle, and I'm trying to evaluate the other angles of my quadrilateral.

By the law of cosines, I can easily find my opposite angle (using the diagonal as a basis for the equation).

However, to find the two remaining angles, I have found no other way so far than to use the other diagonal, which can be found with the equation attached (from geometry atlas).

Is there any simpler way?

Answered by Robert Dawson and Harley Weston.
A quadratic equation 2010-03-15
From rachelle:
under certain condition, it is found that the cost of operating an automobile as a function of speed is approximated by a quadratic equation. use the data shown below to find the function. then use the function to determine the cost of operating the automobile at 60mph.
speed
(in miles per hour)
operating cost
per mile(in cents)
10 22
20 20
50 20

Answered by Penny Nom.
Arc length and Chord length 2010-03-13
From Darryl:
Is there a formula to determine the chord length of an arc knowing only the arc length and the arc depth (sagitta)? I know you can't find the radius with only these two inputs, but can you find the chord length?
Answered by Harley Weston.
Cooling 2010-03-07
From Lori:
If a house is always at 20 degrees celsius. Water (tea) boils at 100 degrees celsius. 5 minutes later the tea was 70 degrees celsius. Use an equation to predict the temperature after 20 minutes.
Answered by Tyler Wood.
The line joining (3,4) and (-5,-6) 2010-03-06
From Patric:
Find the condition that the point (x,y) may lie on the line joining the points (3,4) and (-5,-6)
Answered by Penny Nom.
The test for some disease is 99% accurate 2010-02-24
From baaba:
Assume that the test for some disease is 99% accurate. If somebody tests positive for that disease, is there a 99% chance that they have the disease?
Answered by Chris Fisher.
A linear relationship 2010-02-24
From Mel:
This table shows the linear relationship between an aeroplane's height (above sea level) and the outside air temperature.
h (cm) 120 340 480 600 1040

T (degrees C)

14.28 12.96 12.12 11.4 8.76
Find the gradient and vertical intercept of the function. Write the formula for T.

Answered by Penny Nom.
A geometric sequence 2010-02-22
From Kelsey:
_, 2, _, _, 250, _

i don't know how to fill in the missing terms

Answered by Penny Nom.
Frank makes picture frames 2010-02-20
From Carol:
Frank makes picture frames his revenue from sales is R=$13.60x for frames sold. The cost making frames is C=$5.80x+$120 for frames made. What is the minum number of frames Frank must make and sell in order for his revenue to be greater than his costs.
Answered by Penny Nom and Melanie Tyrer.
Substitution method 2010-02-17
From Melodie:
7x - 4y = -10
4y= x-2

Answered by Tyler Wood.
Can a line segment curve over two planes? 2010-02-14
From Graham:
I am working on a math fair project. Can a line segment curve over two planes? Such as, if I had a three dimensional L bracket and I drew a line segment on it with a marker starting on the bottom of the L and had it curve around the corner and up the top, would it still be considered one line segment? Or is that two line segments? Is there a rule that a line segment can only occupy one plane? Thank you. Graham
Answered by Chris Fisher.
Substitution 2010-02-14
From Izayah:
what is 3 ways to find the meeting point of y=2x+4 and 3x+y=7 by making 2 table and using a graph
Answered by Stephen La Rocque.
The angle at which the road is inclined 2010-02-10
From abeth:
a mountain road drops 5 m for every 22 m of road. Calculate the angle at which the road is inclined to the horizontal to the nearest degree.
Answered by Penny Nom.
1/a + 1/b + 1/c < 1 2010-02-01
From Mrityun:
suppose a,b and c are natural numbers such that 1/a + 1/b + 1/c < 1. Prove that

1/a + 1/b + 1/c < = 41/42.

Answered by Penny Nom and Claude Tardif.
A 3D cardboard cupcake 2010-01-30
From Margaret:
Hi,
I'm an art student and I'm attempting to build a 3D cupcake out of cardboard. I want it to have a circular base of 8.5 inches and sides that are 7'' tall and slope outward so the top of the base is 29'. The top with the frosting would be a detachable lid made from a cone with a base circumfrance of 29". My problem is how to cut the side so they will slope out, I'm pretty sure there needs to be a curve, however I don't how to calculate the degree of it. Ideally I want the sides to be a single piece of cardboard. I also don't know how to calculate the arc needed to make a cone who's circular base's circumfrence is 29''.
Thanks,
Margaret

Answered by Stephen La Rocque.
Linear growth 2010-01-27
From S.:
I. Linear Growth:
Suppose that the town has a fixed increase in population growth number of population increase each year.

Choose the amount of population growth each year = 200 (Hint: Choose a whole number for your growth rate, rather than a percent.)

a) Fill in the following chart:

Year (t) Population (P)
t = 0
(2010) 1000

t = 1
(2011) 1000+200=1200

t = 2
(2012) 1000+400=1400

t = 3
(2013) 1000+600=1600

b) Find a linear equation in the form P = mt + b (y = mx + b), which gives the population, P, t years from 2010.

.
.
.

Answered by Melanie Tyrer.
A sequence 2010-01-27
From Haku:
if there are 8 dots in the first shape, 13 in the second, 19 in the third, and 26 in the fourth, then what is the formula?
Answered by Penny Nom.
Growth rates 2010-01-26
From Bhavya:
Dear Sir/Ma'am,

I read in the text book that the growth rates of these 3 functions are as below- n2/3 < n/lg n < n0.99

I tried the substitution method to check the correctness of this. But it gets really tough as n increase.

Is there any simpler way to understand the correctness of these growth rates?

Thanks and Regards,
Bhavya

Answered by Robert Dawson.
A tunnel from Toronto to Montreal 2010-01-25
From Dave:
I want to make a tunnel from Toronto to Montreal (for example)

Something like this http://mathcentral.uregina.ca/QQ/database/QQ.09.09/h/grant1.html
-------------
My coordinates are 45.442455,-73.861340 (Montreal) and 43.442455, -79.861340 (Toronto)

I need to know how to find arc distance, chord distance and radius What websites can i find for this subject Google has many but they are useless (blah blah) websites
LOL
Thanks

Answered by Chris Fisher and Robert Dawson.
Solving a triangle 2010-01-25
From Paige:
how do i solve a triangle with one angle of 73 degrees, one angle of 32 degrees, and one side of 23cm?
Answered by Harley Weston.
A sequence 2010-01-22
From brahmaji:
0 7 26 63 _ 215 342 fill the blank? find the missing number
Answered by Robert Dawson.
A 4-digit phone code 2010-01-21
From pat:
my daughter can`t remember her 4-digit code to unlock her phone. she is pretty sure it starts with a 4. can you help us? my brain is locked up
Answered by Robert Dawson.
The cube of an integer 2010-01-20
From Shankar:
Can the cube of an integer end with 1985 ones ?
Answered by Robert Dawson.
Angle of incline 2010-01-20
From Alan:
how do I fnd the angle of an incline with a measurement of 0.042 with an adjacent of 1.2mtrs?. Thank You
Answered by Penny Nom.
A cone circumscribed about a given hemisphere 2010-01-19
From Neven:
The cone of smallest possible volume is circumscribed about a given hemisphere. What is the ratio of its height to the diameter of its base?
(G.F.Simmons, Calculus with Analytic Geometry, CH4 Applications of Derivatives)

Answered by Chris Fisher.
A linear equation with the variable on both sides 2010-01-16
From dog:
4x-9=7x+12
Answered by Penny Nom.
Percentage increase from a negative to a positive 2010-01-12
From Greg:
If a company had a net lost in 2005 of $11,234 and net income of $3231 in 2006, what is the percentage annual growth in income between the two years?
Answered by Robert Dawson.
The triangle inequality 2010-01-11
From Nazrul:
If A and B are any two vectors, then how can I prove that magnitude of (A+B)<=magnitude of A+magnitude of B. Please show me the process in details. Thank you.
Answered by Robert Dawson.
The intervals where the function is positive and negative 2010-01-10
From Ron:
Hello

I'm trying to find out the intervals where the function is positive and negative. It's for a polynomial function y= -(x+2)^2 (x-2) and y= (x+1)(x+4)(x-3)

I have tried the right and left side of each x-intercepts, but I still don't understand the results

thank you for your help

Answered by Penny Nom.
A pair of simultaneous equations 2010-01-09
From Yumiko:
Solve the following pair of simultaneous equations.

x^2 -4x = y^2-4
3y=2x - 3

Answered by Penny Nom.
A networking event 2010-01-07
From Tricia:
I have 70 people to go around to 10 tables with 7 at each table. only 6 from each table will rotate each time. There is 1 table monitor that always stays at same table. How do I make the rotation so everyone moves and gets to see all 70 people at the networking event?
Answered by Robert Dawson.
A linear equation with fractions 2009-12-31
From Michelle:
How would I solve: (-2)(-1/2x + 5) = (-2)(8)?
Answered by Penny Nom.
The volume of a frustum 2009-12-29
From dave:
I have a frustum top 1.7r bottom .55r and 2.14 h I have to calculate cement in a silo every week. I am not very good at maths but i have been adding the top and bottom to get an average so as to turn it into a cylinder and i come up with 8.5m3 I know that the correct volume is 9.24 m3. Can you tell me why getting an average width on the top and bottom of a frustum doesn't work.
Thank you.

Answered by Chris Fisher.
Linear programming using the Simplex Method 2009-12-28
From William:
A gold processor has two sources of gold ore, source A and source B. In order to keep his plant running, at least three tons of ore must be processed each day. Ore from source A costs $20 per ton to process, and ore from source B costs $10 per ton to process. Costs must be kept to less than $80 per day. Moreover, Federal Regulations require that the amount of ore from source B cannot exceed twice the amount of ore from source A. If ore from source A yields 2 oz. of gold per ton, and ore from source B yields 3 oz. of gold per ton, how many tons of ore from both sources must be processed each day to maximize the amount of gold extracted subject to the above constraints? I need a linear programming solution or algorithm of the simplex method solution. Not a graphical solution. Thanks.
Answered by Janice Cotcher.
The acute angle formed by intersecting lines 2009-12-17
From Katie:
To the nearest tenth of a degree, find the size of the acute angle formed by the intersecting lines 3x + 2y = 12 and x - 2y = -2. Was is necessary to find the intersection point?
Answered by Robert Dawson.
A pile of sand 2009-12-16
From Malik:
Sand is leaking out of a hole at the bottom of a container at a rate of 90cm3/min. As it leaks out, it forms a pile in the shape of a right circular cone whose base is 30cm below the bottom of the container. The base radius is increasing at a rate of 6mm/min. If, at the instant that 600cm3 have leaked out, the radius is 12cm, find the amount of leakage when the pile touches the bottom of the container.
Answered by Harley Weston.
A trig question 2009-12-15
From A trig question:
Hey, my name is Candle
I'm in academic math10 and am stuck on my trig... one question I thought I had right because i used the cosine law I got wrong and can't figure out why... here's a copy of the question. (i guessed it was D... but my teacher said it's B)
Thanks
Candle

Answered by Robert Dawson.
Energy in calories 2009-12-15
From Josephine:
A soft drink manufacturer claims that a new diet soft drink is now "low Joule". The label indicates that the available energy per serving is 6300 J. What is the equivalent of this energy in calories? (1 Calorie=1000 cal)
Answered by Robert Dawson.
A telephone pole on a slope 2009-12-14
From Marissa:
A 10 meter telephone pole casts a 17 meter shadow directly down a slope when the angle of elevation of the sun is 42 degrees. Find the angle of elevation of the ground. Its a law of sines problem.
Answered by Penny Nom.
A number base that's not a positive integer 2009-12-11
From Nick:
Is it possible for a number to have a base that's not a positive integer? Base 1 gives a result that's meaningless but possible. I have no concept of a base 0, a negative base, or a fractional base, never mind any other base. I think there isn't any base that's not a positive integer, but, knowing that math keeps jumping ahead and sometimes has inventions before anyone knows how to exploit them, I think I'd better ask.

Thank you.

Nick

Answered by Robert Dawson and Claude Tardif.
e = m c^2 2009-12-09
From Jim:
Please explain the following formula: E=MC2 (squared)
Answered by Robert Dawson.
How fast is the distance between the two cars decreasing? 2009-12-08
From Jenny:
Two cares are on a collision course toward point P. The paths of the two cars make a 30 degree angle with each other. The first car is 40 km from P, and traveling toward P at 16 km/hour. The second car is 50 km from P, traveling at 20 km/hour. How fast is the (straight line) distance between the two cars decreasing. (Hint: Law of Cosines)
Answered by Harley Weston.
7 36 14 18 28 9 56 x y 2009-12-06
From lisa:
what is the sequence of x and y
7 36 14 18 28 9 56 x y

Answered by Penny Nom.
A magic/math trick 2009-12-04
From Valentin:
What is the explanation for the following math trick: you think of any four digit number add those digits take that sum and subtract it from the first number then you say three of those new numbers in any order and the other person guesses the last digit. How does he do it?
Answered by Claude Tardif.
Airline routes 2009-12-01
From Kapilan:
Please answer the following question: From A to B there are four possible air routes. From B to C there are five possible air routes. From C to D there are three possible air routes. How many different trips can be taken from A to D and back without taking the same route on any section of the return trip?
Answered by Penny Nom.
A line through the y-intercepts 2009-12-01
From Rogerson:
Find the equation of the line through y-intercept of the lines 3x + 4y = 12 and 3x - y = 9.
Answered by Penny Nom.
What number comes next? 2009-11-25
From Mark:
Can someone help and give me the break down of how to solve. 5,3,1,-1
Answered by Penny Nom.
Solving two equations, one with a square root 2009-11-23
From kacie:
y = square root of x+3
x-4y = -7

im having trouble with this problem...i have to find where they intersect.

Answered by Harley Weston.
The point of intersection of the pair of straight lines 2009-11-19
From Stephanie:
Find the point of intersection of the pair of straight lines:
y=-3x-9
-y=4x+2
x = _______ and y = ________

I am totally confused. Please help

Answered by Robert Dawson.
One-eighth of one million dollars 2009-11-16
From Jarrett:
what is 1/8 equal to out of a million dollars.
Answered by Penny Nom.
Three angles and one side of a triangle 2009-11-16
From Esther:
How do i find the sides of an acute triangle if i know the angels are 60,45,75 and i only know one side which is 10? Thanks!
Answered by Penny Nom.
Excluded values 2009-11-14
From Janie:
I have to State the excluded values for this equation and then solve, but not sure how to do this. Here is the problem
(x+6)/x+3=(3)/(x+3)+2

Answered by Harley Weston.
A truncated cone 2009-11-11
From Lucian:
I need to calculate the bottom inside diameter of a truncated cone.
The top insdie diameter is 1450mm.
The material is 6mm thick
The cone angle is 20 degrees
The slant length is 152mm
I would like a formula so that I can build a spread sheet

Answered by Penny Nom.
What's the next term? 2009-11-03
From bharat:
if
1=5
2=25
3=325
4=4325
5=?

Answered by Robert Dawson and Penny Nom.
The intersection of two graphs 2009-10-30
From asma:
by using the suitable scales,draw both the graph of x/2 + y/4 =3 and y=2 on the same graph paper.Find the points of intersection of the two graph.
Answered by Penny Nom.
Is it a rectangle? 2009-10-29
From sefora:
The coordinates of the vertices of quadrilaterals are given. Draw each quadrilateral on a grid. Determine whether it is a rectangle (show your work)

P(5,1), Q(-4,4), R(-6,-2), S(3,-5)

Answered by Penny Nom.
Two non zero vectors 2009-10-27
From Nazrul:
Given that a and b are two non zero vectors and ma+nb=0 where m and n are two scalars. How can I prove that m=n=0.
Answered by Robert Dawson.
A word problem 2009-10-24
From Ryan:
Two times one number plus 7 times another number is equal to 17. the first number plus the second number is equal to 25. What are the two numbers? Please Help!
Answered by Penny Nom.
e^pi and pi^e 2009-10-22
From Natalie:
Question from natalie, a student:

Without using your calculator, prove which is bigger: e^pi or pi^e

we talked about ad absurdum in class, so I'm assuming that is how I approach this question, but since neither of them have a variable( like the examples we actually solved in class), I'm not quite sure how to solve it....

Answered by Harley Weston.
Forming a square 2009-10-21
From christianah:
Two parallel lines have the equations y=x+4 and y=x-2.Two additional lines are drawn to form a square.What is the side length of the square?Write possible equations for the additional lines.Explain and justify your reasoning.
Answered by Penny Nom.
A sequence 2009-10-20
From Alyce:
The rule of this sequence is k=1/8n. Find the 8th term of the sequence.

1/8, 1/4, 3/8, 1/2, ...
Thanks!

Answered by Penny Nom.
A linear system 2009-10-20
From marissa:
Solve this linear system
2x-y=5
3x+y=-9

Answered by Penny Nom.
A line and a circle 2009-10-19
From Renson:
Determine whether the line x-2y=0 cuts,touches or fail to meet the circle x^2+y^2-8x+6y-15=0.If it touches or cuts ,find the coordinates of the point(s) of contact
Answered by Harley Weston.
The next three terms 2009-10-16
From Shawn:
I'm a parent and don't know how to help my 6th grader solve for this problem.

Name the next tree terms in this sequence and tell the rule used to find each:

-1,0,1,0,1,2,3,8,25,.....

Answered by Chris Fisher.
541.39(1 + i)^15 = 784.09 2009-10-14
From Fitore:
Hi, I noticed that this question was already posted up, however I was hoping I could solve it without having to use logs. Can you please help me? The equation is: 541.39(1 + i)^15 = 784.09
Answered by Penny Nom.
Integration of cos (x^2) dx 2009-10-12
From Venkat:
How do you evaluate the integration of cos (x^2) dx ?
Answered by Harley Weston.
$6.25 in nickels quaters and dimes 2009-10-10
From miriam:
a piggy bank contains $ 6.25 in nickles quaters and dimes. there are 65 coins all together and there are four times as many nickles as quaters find the number of coins of each kind
Answered by Penny Nom.
Two rectangles 2009-10-08
From Lillian:
A rectangle is 5cm longer than twice its width. The width of another rectangle is 3cm less than the width of the first rectangle and its length is 6cm more than 3 times its width. If the perimeters are equal, find the dimensions of both rectangles
Answered by Penny Nom.
Elimination method 2009-10-08
From Kenty:
How do I solve this problem using the elimination method?
3x-7y=0
6x+4y=0
I am not sure how so if someone can show me a similar problem (instead of solving this one for me) that would be fantastic.

Answered by Penny Nom.
Two jets are traveling towards each other 2009-10-05
From jake:
two jets are traveling towards each other and are 4000 km apart. the rate of one jet is 100 km/h faster than the rate of the other. if the jets pass each other after 2.5 hrs what is the rate of speed of the faster jet?
Answered by Penny Nom.
The maximum number of right angles in a polygon 2009-10-05
From Bruce:
Is there way other than by trial and error drawing to determine the maximum number of right angles in a polygon? Secondary question would be maximum number of right angles in a CONVEX polygon. Is there a mathematical way to look at this for both convex and concave polygons? Or are we limited to trial and error drawing?
Answered by Chris Fisher.
What is the next term? 2009-10-04
From julia:
patterns 18 46 94 63 52 61 what is the next one?
Answered by Penny Nom.
Evaluating -x^2 + (yz - 3)^2 2009-09-28
From Kathy:
My daughter and I are having trouble solving this equation:

-x²+ (yz – 3)²

if x=-4,y=2 and z=0 We are having difficulty with the -x² part of the equation. Thanks

Answered by Penny Nom.
Extraneous solutions 2009-09-20
From iyana:
what is an extraneous solution? what must you do to determine whether a extraneous solution?
Answered by Stephen La Rocque and Harley Weston.
Exponential form 2009-09-18
From karen:
Q: Write the following in exponent from: a * a * a * a * * * a, where there are n -1 a's

note: *asterisk means times.

Answered by Penny Nom.
how many ones are in 1 million 2009-09-17
From djuan:
how many ones are in 1 million
Answered by Harley Weston.
The method of substitution 2009-09-17
From laura:
ok! i really need help with this question plz help!!

x= - 4y +5
x+2y = 7

Answered by Penny Nom.
10x-8 /3 =6x+24 /5 2009-09-16
From Heidi:
10x-8 /3 =6x+24 /5.....I know the answer is 7/2 but how do I get there?
Answered by Penny Nom.
A sequence 2009-09-15
From Derek:
what are the next four numbers in this sequence 1/8, 1/4, 3/8, 1/2...
Answered by Penny Nom.
A trillion grains of rice 2009-09-14
From akaila:
can 1trillion rice grains fit in a classroom with the area of 144.4m3
Answered by Penny Nom.
The slope-intercept form 2009-09-11
From Shelby:
Write an equation in slope-intercept form of the line that is parallel to the graph of 3y-4x=1 and passes through (0,6)
Answered by Penny Nom.
Exponential form 2009-09-08
From hanna:
hi, well i am really bad in math and i have a math paper and i was just wondering what 3x3x3x3x3 was in exponential form and what is exponential in a way a person like me would understand?
Answered by Penny Nom.
Airline overbooking 2009-09-03
From Nikita:
An airline company knows that 8% of it's passengers will not show up for their scheduled flights. A plane has 175 seats.

a) What is the probability that 10 passengers or fewer will not show up?

b) What is the probability that 10 to15 passengers will not show up?

c)What is the probability that exactly 10 passengers will not show up?

d) What is the probability that more than 19 passengers will not show up?

Answered by Robert Dawson.
What is 8 tons in exponential form? 2009-09-01
From Tyler:
what is 8 tons in exponential form
Answered by Penny Nom.
The sum of digits of 4444^4444 2009-08-31
From SHIVDEEP:
The sum of digits of 4444^4444 is A .The sum of digits of A is B .
Find the sum of digits of B ?

Answered by Claude Tardif.
Exponential form 2009-08-31
From cecil:
what is the exponent form 564000?
Answered by Stephen La Rocque and Harley Weston.
Simultaneous equations 2009-08-28
From onias:
solve 3/a - 2/b = 1/2 , 5/a + 3/b = 29/12
Answered by Robert Dawson.
y=2x+1 and y=2x-1 2009-08-28
From MARICELA:
Need help of how to work this problems

y=2x+1 and y=2x-1

First of all what is the difference?

Second I think is easy just to work with the X y with the line in the middle

but how do you get the numbers for each side.

Answered by Penny Nom.
2(x-4)>-4+2x 2009-08-27
From jorna:
2(x-4)>-4+2x
Answered by Penny Nom.
Linear equations 2009-08-23
From shwetha:
The length of a rectangle is greater than the breadth by 18 cm. If both length and breadth are increased by 6 cm then area increases by 168 cm square. Find the length and breadth of the rectangle.
Answered by Stephen La Rocque.
Graph 2x-3y+15=0 2009-08-20
From Diem:
Graph 2x-3y+15=0
Answered by Leeanne Boehm.
Making a cone 2009-08-19
From angela:
how can i make a cone with a 6cm of heigth and has 2cm radius in the opening??
Answered by Stephen La Rocque.
Find the central angle 2009-08-18
From Larissa:
In a circle, the length of a chord AB is 4 cm and the length of the arc AB is 5 cm. Find the central angle theta, in radians, correct to four decimal places. Then give the answer to the nearest degree. I think I'm supposed to use Newton's method, but am not sure how to start with this problem.
Answered by Harley Weston.
The median is greater than the mean 2009-08-14
From Shawna:
If the median is greater than the mean on a set of test scores, describe the situation.
Answered by Robert Dawson.
Fitting the curve y=a*exp(b*x)+c 2009-08-12
From aika:
Could one show me the complete algorithm and formula for finding the coefficients (a,b, and c) in exponential regression model y=a*exp(b*x)+c
Answered by Robert Dawson.
The volume of a telephone pole 2009-08-11
From robert:
12.5"@ base x 7" @ top and 40' height. How would I find the volume in cubic feet for a telephone pole with these dimensions?
Answered by Harley Weston.
Exponential form 2009-08-05
From motaz:
Let x = log_2 1/8 Write the exponential form of the equation and solve the equation for x
Answered by Stephen La Rocque.
A cone with the top cut off 2009-08-02
From Paul:
I am making a cone with a diameter of 1300, a base of 500, a side length 800. I need to workout so I can draw it out flat, cut it out, and make into a cone.
Answered by Harley Weston.
Two equations in two unknowns 2009-07-30
From Alicia:
I need help with the following equation please show me the steps to complete so that I will know how to do this my self.
2x-y=2 5x+y=5 Thanks for the help

Answered by Penny Nom.
Inequalities Proof 2009-07-24
From ABOU:
good morning.......a b c are real positive no zero......proof that sq root(2a/(a+b))+sq root(2b/(b+c))+sq root(2c/(c+a))inferior or equal 3 thank you
Answered by Janice Cotcher.
Circular Measures 2009-07-24
From onyeka:
find the equation of a circle with point [3,7][5,5] and wich center lies on the line x-4y=1
Answered by Robert J. Dawson.
Inequalities 2009-07-22
From Sindhu:
prove that 4^79 < ( (2^100) + (3^100) )< 4^80 ?
Answered by Robert Dawson.
Finding the Line Joining Perpendicular Feet 2009-07-16
From mukulu:
Please help I've been tryng to search in your data but i failed to get the solution Find the equation of a straight line joining the feet of the perpendiculars drawn from the point A(1,1) to the line 3x-3y-4= 0 and 3x+y-6=0.
Answered by Janice Cotcher.
The lateral area of a cone 2009-07-15
From ashley:
What is the radius of a cone with the lateral area being 443.3 mm^2 and the slant height being 14.7 mm.
Answered by Penny Nom.
A sequence 2009-07-11
From jamie:
what is the next number in the sequence and what is the relationship of the numbers? 36,16,20,10,12, _
Answered by Penny Nom.
Fractional Exponents 2009-07-07
From bob:
2^5/2 - 2^3/2
Answered by Janice Cotcher.
Simultaneous Equations 2009-07-06
From Mukulu:
Solve the equation simultaneously X/5=(Y+2)/2= (Z-1)/4 ……………….eqt 1 3X+4Y+2Z-25=0 ………………eqt 2
Answered by Janice Cotcher.
Vector spaces 2009-07-05
From Nazrul:
How can I prove that m(a+b)=ma+mb where a and b are two vectors. Thank you.
Answered by Harley Weston.
Parallel lines 2009-07-01
From chris:
Write the slope of a line that is parallel to each line.
1.y=2x-5
2.y=-x+2
3.3x+y=10
4.5x-y=11
5.x+2y=6
6.2x-3y=9
7.4x+y=3
8.x+2y=14

Answered by Brennan Yaremko.
The product of gradients between 2 perpendiculars lines 2009-06-11
From Alister:
how do i prove that the product of gradients between 2 perpendiculars lines equal to -1....
Answered by Penny Nom.
Vectors and the Law of Cosine 2009-06-08
From lauren:
once force of 20 pounds and one force of 15 pounds act on a body at the same point so that the resultant force is 19 pounds. Find, to the nearest degree, the angle between the two original forces
Answered by Janice Cotcher.
Properties of Inequality 2009-06-05
From Nazrul:
We know that if a>b then (1/a)<(1/b) But why 5> -2 implies (1/5)>(1/-2) . Thank you in advance for your help.
Answered by Robert J. Dawson.
At what speed does an automobile start to hydroplane? 2009-06-03
From Don:
How do you figure at what speed does an automobile start to hydroplane when driving on a wet surface.
Answered by Robert Dawson.
An aeroplane flying in a wind 2009-06-02
From Maria:
The aeroplane flies in a wind of speed 50 kph from the direction S80◦W. However, the direction in which it is pointed, and its speed in still air, are such that its resultant speed and direction are 800 kph, and direction N71.6◦E

I need to find the size of the angle between v and vw, in degrees to one decimal place, and find the speed at which the aeroplane would fly if the air were still, to the nearest kph.

It would help me greatly if I could see a diagram illustrating the velocity va that the aeroplane would have if the air were still, the velocity vw of the wind, and the resultant velocity v of the aeroplane (diagram should be in the form of a triangle illustrating how one velocity vector is the sum of the other two)

Thanks

Answered by Harley Weston.
Extraneous solutions 2009-06-02
From Ayana:
solve and check for extraneous solutions.

3x+6/ x²-4 = x+1/ x-2

x can not = {-2,2}

Answered by Penny Nom.
Two equations 2009-05-31
From tony:
y = 1.2x - 32.7,

y = -0.7x + 46.15

can you please solve this for me? i am not able to figure it out... thank you tony

Answered by Penny Nom.
Draw a net for a rectangular prism? 2009-05-29
From Mohamad:
Draw a net for a rectangular prism?

Last week when we were studying for our finals we came across a section about drawing nets I would really appreciate it if you gave me some help on it

Answered by Robert Dawson.
A sequence 2009-05-26
From Jay:
what is the equation that would give me the next 3 numbers in this progession: 0,1,5,14,20
Answered by Stephen La Rocque.
Find out the length of a cup when its volume is halved 2009-05-25
From Thomas:
I'm having trouble with a question. What kind of formula would i use to find out the length of a cup when its volume is halved?
Answered by Stephen La Rocque.
Nickels, dimes and quarters 2009-05-25
From Amy:
Coming from South Africa I find working with nickels, dimes and quarters confusing. How many dimes are in a nickel and how many quarters are in a dime?
Answered by Penny Nom.
The dimensions of a larger cup 2009-05-23
From Elizabeth:
Hey If i have a cup that holds a volume of 477mL and the bottom radius is 2.8cm and the top radius is 4.9cm and the height is 10cm. If i increase the volume by one and a half times what is the new measurements if the cup is directly proportional to the first one. Thank you
Answered by Stephen La Rocque and Penny Nom.
(x + 1) / 3 + (x + 2) / 7 = 2 2009-05-22
From MzDonna:
Question from MzDonna, a parent:

We can't seem to get an answer for this problem:
X + 1      X + 2
_____ + _____   = 2

   3              7

The book gives the answer as 10 ; But, we struck. Please help us.
                                               ___
                                                21       

Thanks in advance for your assistance.
Answered by Stephen La Rocque.

The volume of a cone 2009-05-20
From Lillian:
okay! so my math question is about the volume of a cone! my work sheet gave me the volume and base of the cone and asked me for the height! please help me discover the height of a cone when the volume is 78.54 cm (cubed obviously) and the base is 6 centimeters. thanks so much :)
Answered by Penny Nom.
Two equations 2009-05-18
From Bob:
solve the system:

x/2 - 2y= -8
3x + 2/3y= -10

Answered by Stephen La Rocque.
|1/(x+1)| <1 2009-05-16
From Nazrul:
What is the solution of absolute value of (1/(x+1))<1? Please explain.
Answered by Stephen La Rocque.
Rewriting an Expression with Positive Exponents 2009-05-13
From lynda:
(-4x)to the 0 power times x squared over 3to the -3 power i have to rewrite the expression with pos exponents
Answered by Janice Cotcher.
Finding an Array to an Integer Linear Programming Problem 2009-05-12
From Debbie:
how to make an array for 3 box of juice for 25 students and 2 teacher. 1 pack of juices hols 3boxes the answer is 9 packages but how do you make array for this
Answered by Janice Cotcher.
Solve -1<5-2x<10 for x 2009-05-12
From Helen:
solve the following inequality for x
-1<5-2x<10 Te less then sign by the one, should be less then or equal to.

Answered by Stephen La Rocque and Penny Nom.
A number sequence 2009-05-08
From alicia:
what is the next number 1,2,6,42,1806------? is it 1954?
Answered by Robert Dawson, Chris Fisher, Penny Nom and Claude Tardif.
Perpendicular lines 2009-05-07
From shabnam:
the line presented by y= 3x-2 and a line perpendicular to it intersect at R(1,1). Determine the equation of the perpendicular line
Answered by Stephen La Rocque.
Stairway construction 2009-05-02
From Olivia:
Hi, I'm Olivia and I'm in 8th grade studying algebra 1. I'm having some trouble with my math project. It has to do with rules of stairway construction. It says that there are two generally accepted rules to building stairs. Rule 1: x+y=43 and x+y=46 (cm) Rule 2: 2y+x=61 and 2y+x=64 (y is the riser and x is the tread length) I cant figure out how to graph them...it says they're supposed to intersect to make a square which is like the safety zone for building stairs (the stairs' rise and tread lengths are plotted as points) any points outside of that square are considered unsafe. Did i explain it ok? Thanks for your help. --olivia
Answered by Penny Nom.
Percent increase from a negative number to a positive number 2009-04-21
From Brian:
I have no problem calculating year over year net profits but am having trouble when it comes to a negative.
2007 Net Profit = -110,333.00
2008 Net Profit = 66,054.00

Brian

Answered by Harley Weston.
1/(square root 16) 2009-04-21
From Conor:
Hi Guys!

I dont have a square root sign so im just gonna use the ^ symbol :)

I am just wondering if

1/(16^a)

when re-written can it be

1/16 to the power -1/2

Thanks :)

Answered by Penny Nom.
A perpendicular line in standard form 2009-04-21
From Kristy:
Can you help me with this equation? Find the equation, in standard form of the line perpendicular to 2x-3y=-5 and passing through (3,-2) With the equation in standard form with all integer coefficient.
Answered by Stephen La Rocque.
12 oz. cup 2009-04-19
From Tom:
I am a ceramic teacher and wanted my students to make a 12 oz. cup, what formula should we use?
Answered by Chris Fisher.
Exponential form 2009-04-16
From Pete:
Hi, How do you express ³√h^-4 in exponential form. I am having a lot of trouble with this one.
thanks
Pete

Answered by Stephen La Rocque.
Parallel and perpendicular lines 2009-04-08
From Blake:
I need to know what the formula for working a problem. I need to know how to find if i line is parallel or perpendicular by only looking at the equation. i know you must know the slope but how do i find slope?
Answered by Stephen La Rocque.
A large, hollow, ice cream cone 2009-04-03
From Darah:
A manufacturer is making a large, hollow, ice cream cone to serve as an ad for a local Baskin-Robbins. The ice cream cone is made up of a cone with height 8 feet, topped by a hemi-sphere with radius 6 feet. How much ice cream could the hollow object hold? If a gallon is 0.13368 cubic feet, how many gallons does it hold? If 3 gallons of Baskin-Robbins heavy cream chocolate blend weighs 24 pounds, how much would the ice cream cone weigh, excluding the weight of the construction material?
Answered by Stephen La Rocque.
Grain of rice on a chess board 2009-03-30
From andy:
Is there an easy way for my students to find out 8 to the power of 21 to solve the grains of rice doubling investigation on a chessboard? Any suggestions welcome. Thanks very much
Answered by Robert Dawson and Claude Tardif.
Octagonal panels for a horse pen 2009-03-25
From Tony:
I am building a pen for my horse. I am going to use 12' panels in the shape of an octagon. How many feet will he have from side to side using 12' panels.
Thanks,
Tony

Answered by Robert Dawson and Penny Nom.
Comparing positive and negative fractions 2009-03-24
From Christyrose:
Hello. I am a sixth grade math student and i need help on how to compare negative and positive fractions.

Here is is:
3/4 -2/3 7/10 -5/6

I'm not sure how to do the negative parts because i was absent for a couple days. Thank you!

Christyrose

Answered by Penny Nom.
An infinite number of solutions 2009-03-24
From Sean:
this is a linear equations problem;

first:
3535.5 + Fbd (.866) + Fbc (.5) - Fab (.5) = 0
and
-3535.5 - Fab (.866) - Fbc (.5) - Fbd (.5) = 0

Answered by Harley Weston.
An equation for a line with a changing slope 2009-03-21
From Ben:
Is there anyway to write an equation for a line with a changing slope. e.g. A cell [hone company has a initial fee of 50$ paying for the first 100 minutes, the rate then increases to 10 cents per minute, then at 500 minutes the rate is 5 cents per minute.

If so please tell how.
Thanks

Answered by Penny Nom.
A dead fly is stuck to a belt that passes over two pulleys 2009-03-21
From Jules:
A dead fly is stuck to a belt that passes over two pulleys 6 inches and 8 inches in radius. Assuming no slippage,how fast is the fly moving when the large pulley (8 inches) makes 21 revolutions per second ? How many revolutions per second does the small pulley make ? How long will it take the dead fly to travel 1 mile ?
Answered by Penny Nom.
The volume of water in a cone 2009-03-17
From Freddie:
A ball of diameter 20cm rests in a conical container whose angle with the slant height and the vertical axis is 25degrees. if water is poured into the container just enough to touch the bottom of the ball, find the quantity of water in the container.
Answered by Penny Nom.
A plane cuts a line segment 2009-03-17
From Manis:
Find the ratio in which the line joining (2,4,16) and (3,5,-4) is divided by the plane 2x-3y+z+6=0.
Answered by Robert Dawson.
A slopped table top 2009-03-16
From anthony:
A perfect circle with radius 100m is supported by four legs, 50m long. The circle is flat when all four legs are 50m long, and tilted when the legs are different in length. The four legs are spread evenly 90 degrees apart at 0, 90, 180, and 270. Find the high and low points in degree and height when the legs are 55m at 0 degree, 70m at 90 degree, 45m at 180 degree, and 30m at 270 degree.

Not sure if they can be calculated using only 2 or 3 legs.

I have tried by picking the two adjacent highest legs (55m and 70m) so I know the high point falls into the first quadrant. Then I tried using ratio to determine where the high point is, which I think high point should be closer to the 70m leg, I am not sure. If I can find the high point, the low point should be opposite to it.

Answered by Chris Fisher and Harley Weston.
How many ball bearings can you make from a cone? 2009-03-14
From Vatsal:
there is a sum in my maths book which states the following: how many ball bearings of diameter 2.5 cm. can you get by melting a solid metalic cone of radi. 20cm and a Hight of 5 meters? how do i do this please tell me even the minute things like cancellatoins and where to use = , ; or any such thing because Im a bit weak in maths
Answered by Harley Weston.
Express 13 mills as a decimal part of $1.00. 2009-03-13
From Kenneth:
Hello:

I read the following in an old business mathematics textbook: "Express 13 mills as a decimal part of $1.00."

I can determine the answer as follows: There are 1000 mills in or per $1.00. 13 mills divided by 1000 mills/$1.00 equals $0.013, the answer.

My question is as follows: What does "...as a decimal part of $1.00" indicate or represent from "Express 13 mills as a decimal part of $1.00."?

Is (a decimal part of $1.00) any decimal less than $1.00 with the decimal having a dollar sign before it.

I thank you for your reply.

Answered by Harley Weston.
A linear system 2009-03-13
From Rasanga:
Use the row echelon form of the augmented matrix to solve the following linear system.
X1+2X2+X4=6
X3+6X4=7
X5=1

Answered by Harley Weston.
The equation of a line 2009-03-11
From Teresa:
Write an equation in slope-intercept form that has a horizontal intercept of -7/3 and a vertical intercept of -7/9
Answered by Harley Weston.
Related rates 2009-03-09
From Megan:
A plane flying with a constant speed of 330 km/h passes over a ground radar station at an altitude of 3 km and climbs at an angle of 30°. At what rate is the distance from the plane to the radar station increasing a minute later?
Answered by Harley Weston.
An inequality with logs 2009-03-06
From Indrajit:
(x/10)^(logx - 2)<100..........how to solve this one??
Answered by Stephen La Rocque.
If 5 to the power of y = 625, what is y? 2009-03-02
From Charlene:
If 5 to the power of y = 625, what is y?
Answered by Penny Nom.
Comparing 2^3^4^5 ; 2^5^4^3 ; 5^4^3^2 etc. 2009-03-01
From JIM:
WHICH WOULD BE LARGEST USING THE NUMBERS 2,3,4,5 : 2^3^4^5 ; 2^5^4^3 ; 5^4^3^2 : ETC. OF ALL THE POSSIBLE COMBINATIONS ? IS THERE A GENERAL FORMULA ? I AM UNABLE TO FIND A COMPUTER THAT CAN HANDLE SUCH LARGE CALCULATIONS . I ALSO DO NOT THINK IT IS AS STRAIGHT- FORWARD AS FIRST APPEARS . THANK-YOU , JIM
Answered by Stephen La Rocque, Robert Dawson and Harley Weston.
Find the resultant of this displacement pair 2009-02-22
From katydidit:
Find the resultant of this displacement pair:
500 miles at 75 degrees east of north and 1500 miles at 20 degrees west of south.
How do I graph this and how do I solve this problem?

Answered by Penny Nom.
The graph of y = -6 2009-02-21
From Barb:
I was given the problem that says "graph the line y = -6" how do I find all the set points that have a y coordinate of -6? I am so lost lost and have no clue how to start. Can you help me please? thanks.
Answered by Penny Nom.
Find an expression for the nth term 2009-02-20
From Dawn:
Find an expression for the nth term:

x= 1 2 3 4...n
y=11 8 5 2...?

I think it's a linear relation but don't know the equation that goes with it.

Answered by Stephen La Rocque.
Linear systems 2009-02-20
From Rose:
I have been having trouble trying to figure out these three math problems , I need help breaking them down so I could understand them better please help.
1. x = 7 - x
2 x - y + 8

2. 8 x + 5 y = 1 8 4
x - y = 3

3. y + 2 x = 3
y + 2 x = 4

I can't figure out how to break them down in the right order.

Answered by Penny Nom.
The substitution method 2009-02-18
From Nicolette:
I am working on linear programming problems that I need to solve pairs of linear equations for. I am confused and am not sure how to solve these problems!
5x+3y=7      y=x-3 How can i solve this using the substitution method?

Answered by Penny Nom.
A line parallel to a given line 2009-02-18
From Lisa:
I'm having a hard time understanding the concept of how to write an equation of the line containing the given points and parallel to the given line.
Express the answer in the form of y=mx+b. (7,9); 2x+5y=4. i would appreciate it alot if someone can show ne how to do this. Thank you, Lisa

Answered by Penny Nom.
A conical sleeve 2009-02-17
From Jonathan:
I'm having a hard time making a design pattern for a cone sleeve, the thing is the cone sleeve is 22 degrees, how can i know the angle of this when it is flat on paper, based on my calculations, it should be around 66 - 69, but i want it to be exact can anybody help?
Answered by Penny Nom.
A fraction with negative exponents 2009-02-14
From Gabriel:
What does the line seperating the top from the bottom mean? what does it tell you to do and what is the answer to the question?

Question 3

Simplify. Write the answer using only positive exponents. Assume all variables represent nonzero numbers.

(x^-5)^-4(x^-1y)^3
________________
(xy^4)^3


Answered by Penny Nom.
Large exponents 2009-02-12
From Dan:
I would like to know how to calculate a whole number as the base that has a large exponent;Such as 7 to the 23rd.power=. Otherwise if I use a calculator it goes off the scale.
Answered by Stephen La Rocque.
6^x = y 2009-02-12
From Jamie:
Find x: 6^x=y
Answered by Harley Weston.
Cubic feet gas to grams of gas 2009-02-11
From tim:
I was trying to convert cubic feet gas to grams of gas. Is there a formula to complete this task?
Answered by Robert Dawson.
A sphere in a cone 2009-02-10
From Shubham:
An upturned conical vessel of radius 6cm and height 8cm is completely filled with water. A sphere is lowered in the conical vessel filled with water and the size of sphere is such that it just touches the sides of cone and is just immersed. What fraction of water overflows?
Answered by Harley Weston.
Solving an equality 2009-02-10
From Deborah:
2(v+10)<18
Answered by Penny Nom.
Are these lines parallel? 2009-02-06
From Conor:
are these lines parallel?

y = 3x -2 and 6y = 3x +2

Answered by Stephen La Rocque.
Congruent line segments 2009-02-05
From casie:
marica drew one-line segment on a graph with endpoint of (0,9) and (0,4). she drew another line segment with endpoints (1,1) and (6,1). are the line segment congruent?explain
Answered by Penny Nom.
A point on -8x^2+5xy+y^3=-149 2009-02-04
From Vivian:
Consider the curve defined by -8x2+5xy+y3=-149
a) find dy/dx
b) Write an equation for the line tangent to the curve at the point (4,-1)
c) There is a number k so that the point (4.2,k) is on the curve. Using the tangent line found in part b), approximate the value of k.
d) write an equation that can be solved to find the actual value of k so that the point (4.2,k) is on the curve
e) Solve the equation found in part d) for the value of k

Answered by Harley Weston.
Fertilizer in a bin 2009-02-03
From Todd:
Hello I am looking for a formula to figure out the fertilizer volume in a hopper bottom bin not only when it is full but part full as well. When you are filling it is heaped up in the middle to make a cone and when you are emptying the bin the cone is inverted so it would be nice to be able to quickly figure out the tonnes partly filled and when full.
Lets say the bin is 32 feet high from top of bin where you fill to the bottom where the product goes out and it is 16 feet in diameter. I know how to calculate the cylinder it is the cones on the top and bottom of the bin I have the main question on.

Answered by Harley Weston.
Simultaneous equations with an xy term 2009-02-01
From angelee:
xy+5x-2y-10=0 2x+y=1
Answered by Penny Nom.
The intersection of two lines 2009-01-30
From kaylee:
how do you find and graph the intersect of these two equations: y=2x-4 (and) 2y=x thanks, kaylee
Answered by Stephen La Rocque.
Missing three digits in a phone number 2009-01-30
From Elmer:
I have part of a phone number. the first 3 digits are 212, the last digit is four, and the three missing digits are all odd, how many numbers will I have to call to get the right number?

212- _ _ _ 4?

Answered by Stephen La Rocque and Penny Nom.
The vertical line test 2009-01-26
From bob:
what is a Vertical line test
Answered by Stephen La Rocque.
Linear speed 2009-01-26
From Hom:
A semi is driving cross country at 70mph. The tires on the semi have a 3 foot diameter. What is the linear speed in feet per minute?
Answered by Robert Dawson.
Using y = mx + b 2009-01-26
From Caroline:
How do you write a symbolic representation using the y=mx+b method?
Answered by Robert Dawson.
How fast is the visible surface of the earth decreasing? 2009-01-24
From Ray:
A dive bomber loss altitude at a rate of 400 mph. How fast is the visible surface of the earth decreasing when the bomber is one mile high?
Answered by Harley Weston.
Water flowing from a cone to a cylinder 2009-01-23
From Ray:
Water is passing through a conical filter 24 cm deep and 16 cm across the top into a cylindrical container of radius 6 cm. At what rate is the level of water in the cylinder rising when the depth of the water in the filter is 12 cm its level and is falling at the rate of 1 cm/min?
Answered by Harley Weston.
20 miles of gasoline 2009-01-18
From Roger:
I used the volume formula and found that a cylinder with a 3" radius X 8.75" height yields just over 231 cu. in. (one gallon)- doesn't seem right, but that's what I got. Then found the percent that 8.175" is to one foot and multiplied that into the number of feet in 20 miles. I'm not good at math, so I'm starting to lose myself here. The short story is I get a cylinder that is 20 miles long and only .0000387" in diameter. Is this even close to the mark?
Answered by Harley Weston.
The point of intersection of the straight lines 2009-01-18
From Kieran:
Find the point of intersection of the straight lines

y=2x+4, and y+2x-16=0

Answered by Robert Dawson and Penny Nom.
How far is the airplane from the control tower? 2009-01-17
From Murtaza:
An aircraft is vertically above a point which is 10 km west and 15 km north of a control tower. The aircraft is 4000 m above the ground, how far is it from the control tower?
Answered by Penny Nom.
Solve for x 2009-01-16
From maritza:
5=3(x-2)=
Answered by Penny Nom.
0.0708 lbs per linear yard 2009-01-16
From Kevin:
If I know that my product is .0708lbs per linear yard how do I calculate how many Linear yards are in a lb?
Answered by Penny Nom.
A line is perpendicular to another line 2009-01-16
From Greg:
A line is perpendicular to the line y=2x+3 and has the same x-intercept as x+3y+10. Find the equation of this line. Express your answer in the form of y=mx+b. Justify your answer.
Answered by Robert Dawson.
Multiplying negative numbers 2009-01-15
From stephanie:
hi, I'm currently working in a grade 8 class where the students are learning about integers. the students were asked to find examples of how integers are applied in real life. they were able to find aplications for: adding and subtracting both positive and negative integers. however, we could not find an example of where two negative integers would be either divided or multiplied together in real life. For example: (-2)(-4)= +8. please help.
Answered by Harley Weston.
The volume of a cone 2009-01-13
From ab:
the area of the base of the cone shown is 314 cm and it's height is 12 cam what is the volume?
Answered by Penny Nom.
Negative rate of change 2009-01-12
From hemanshu:
when i have to find rate of change of decrease in any value my ans comes in negative why??????????
Answered by Penny Nom.
The table of values for y=3x-10 2009-01-09
From Jade:
How can I know the value of x for y=3x-10 in this table of values pls. include the deatails

if x=-2 y=-16
if x=3 y=-1
but how if y=2 what is x

Answered by Penny Nom.
The top half of a cone vs. the bottom half of the same cone 2009-01-08
From Tammy:
What is the ratio of the volume of the top half of a cone vs. the bottom half of the same cone? Is it seven times larger, or four times larger? and is the ratio consistent regardless of the size of the cone opening?
Answered by Penny Nom.
50 coins with a value of $1.00 2009-01-08
From Ana:
you have 50 coins which have a total value of 1.00. What are the coins and how many of each do you have?
Answered by Robert Dawson.
A kennel with 3 individual pens 2009-01-06
From Jean:
An animal clinic wants to construct a kennel with 3 individual pens, each with a gate 4 feet wide and an area of 90 square feet. The fencing does not include the gates. Write a function to express the fencing as a function of x. Find the dimensions for each pen, to the nearest tenth of a foot that would produce the required area of 90 square feet but would use the least fencing. What is the minimum fencing to the nearest tenth?
Answered by Harley Weston.
The area of a region in the plane 2009-01-03
From Rogerson:
Find the area, S, of the shaded region enclosed by the given cureve, the given line and the x-axis.

y = -x^2 + 1
line x = 2

Answered by Harley Weston.
The radius of a cone 2009-01-02
From kalpaj:
A conical funnel holds 100ml. If the height of the funnel is 10 cm, determine its radius, to the nearest tenth of a centimeter.
Answered by Penny Nom.
Solving an inequality 2009-01-02
From jarred:
3x+8>2
Answered by Penny Nom.
What is i^i? 2008-12-27
From randomness:
i have learnt that 'i' is square root of -1. What is then i^i ? It baffles my maths teacher...
Answered by Robert Dawson and Penny Nom.
Solving an equation for D 2008-12-18
From Darrin:
In his finance class, my son is being asked to take the following equation and solve for "D":

1 / [1 - (1 + D/12)^T] = (1 / D) * [((12*B) / (P*T)) + Y + ((12N * Z) / (360T))] - [(12Z) / (360T)] * F

Answered by Robert Dawson.
The 2 rightmost digits 2008-12-18
From Peter:
Is there a pattern for the 2 rightmost digits of a power? For example, one problem for a math competition was what are the 2 rightmost digits of 3^1993?
Answered by Robert Dawson and Victoria West.
The angle between two lines 2008-12-17
From abhi:
how to calculate the angle between two lines, given the length of the lines.. angle should vary from 0 - 360 in the counterclockwise direction
Answered by Robert Dawson and Harley Weston.
The nth power 2008-12-12
From Kellie:
I am taking a 9th grade course yet im in 7th grade. I do not understand the nth power and i have no clue what it is. could you explain to me what it is?
Answered by Robert Dawson.
A wine decanter 2008-12-10
From Suzanne:
I am trying to buy a wine decanter for my dad for Christmas. One big problem I am having shopping online is finding the right size. Wine is sold by metric ml, with a standard bottle holding 750ml of wine. Decanters are measured by oz, and the sizes are all over the place, with one site alone having sizes 25oz, 26 1/2oz, 50oz, 59 1/8oz, and 68oz.

Can you help me with metric conversions so I can figure out how much wine these decanters hold? Like everybody else, I need this as soon as possible so I can get it shipped in time for Christmas. (I'm disabled & don't drive, so online shopping is my only way to get gifts.)

Thanks for your help!

Suzanne

Answered by Chris Fisher and Robert Dawson.
A quadrilateral with exactly 1 pair of parallel sides and no congruent sides 2008-12-10
From alie:
a quadrilateral with exactly 1 pair of parallel sides and no congruent sides is what?
Answered by Robert Dawson.
Electrical energy 2008-12-09
From ROHAN:
A lamp consumes 1000J of electrical energy in 10s. what is its power
Answered by Stephen La Rocque.
A line in slope intercept form 2008-12-08
From laurie:
the question: Write and equation of the line that passes through the point and has the give slope. Then rewrite the equation in slope intercept form. (2, 5 ) m= 3
Answered by Penny Nom.
The sides of a rectangle 2008-12-08
From Mitchell:
The perimeter of rectangle ABCD (represented on a Cartesian plane) is 52cm. The equation associated with segment AD is 3x+4y-60=0 Point C is at (18, 14). What is the length of segment AD?
Answered by Penny Nom.
Ratio of Volumes of a Cylinder and a Cone 2008-12-06
From rohan:
A CYLINDER IS WITHIN THE CUBE TOUCHING ALL THE VERTICALS FACES . A CONE IS INSIDE THE CYLINDER.IF THEIR HEIGHT ARE SAME WITH SAME BASE ,FIND THE RATIO OF THEIR VOLUMES.
Answered by Janice Cotcher.
Three one-digit numbers 2008-12-06
From JOANNA:
Could you please tell me what Adding Three One-Digit Numbers += 15 1-9 without using any number twice
Answered by Penny Nom.
The volume of a cone 2008-12-03
From Tamriko:
Hi! Help me, please to solve the following problem: The diameter of an ice-cream cone is 6 cm and the slant height is 10 cm. What volume of ice-cream would fit inside the cone? Thanks!
Answered by Penny.
What is so important about quadratics? 2008-11-29
From zoe:
what is so important about quadratics?
Answered by Harley Weston.
Four nines to make 100 2008-11-27
From jane:
use 4 nines in a math equation that will give you an answer of 100
Answered by Penny.
Related rates 2008-11-26
From Lyudmyla:
How fast is the volume of a cone increasing when the radius of its base is 2 cm and growing at a rate of 0.4 cm/s, and its height is 5 cm and growing at a rate of 0.1 cm/s?
Answered by Harley Weston.
Solve 3x +5= 4x-10 2008-11-22
From Nikko:
HELP please how do I solve these type of questions? 3x +5= 4x-10 or 4x +10= 6x-8
Answered by Penny Nom.
The nth term of a sequence 2008-11-20
From kim:
Find the next 2 terms of the sequence.
11n+2.5

Answered by Harley Weston.
The path of a small sailboat 2008-11-19
From jane:
a sailor in a small sailboat encounters shifting winds. she sails 2.00 km East then 3.40 km North East, then an additional distance in an unknown direction. Her final position is 6.68 km directly east of the starting point. find the magnitude & direction of the third leg of the voyage.
Answered by Harley Weston.
How do you solve " (x-4)^3/2 = -6 "? 2008-11-17
From Jim:
How do you solve " (x-4)^3/2 = -6 " ?
Answered by Penny Nom.
The nth term of a sequence 2008-11-16
From Robin:
How do I solve for the next term in the following sequence, I just don't want the answer...I want how to do it to. x,x -1, x -2

Also, how do I solve for the nth term? I can't understand the explanations I've found. He has numbers subscripted in his equations.

Answered by Victoria West.
The midpoint o a line segment 2008-11-15
From Jane:
The vertices of a triangle are at (1,7), (6,-1) and (0,3). Find the coordinates of the midpoints of the sides.
Answered by Penny Nom.
How far are the boats apart? 2008-11-14
From dom:
Two boats leave port at the same time. They leave at 150 degree angle. One boat travels at 10mph and the other at 20mph. After two hours how far are the boats apart?
Answered by Penny Nom.
What is -5x+3y=24? 2008-11-13
From Robert:
What is -5x+3y=24???????
Answered by Stephen La Rocque.
The angles and sides of a triangle 2008-11-13
From JAMIE:
a triangle with a side(b)37m an angle(C)70degrees and (a)79m find values of angles A and B and length of side c
Answered by Stephen La Rocque.
The volume of a cyclone 2008-11-13
From Dianna:
I need to figure the cubic yards of a cyclone. 144" down to 48" 60" high
Answered by Penny Nom.
A conical funnel 2008-11-12
From Rachael:
Hello, I am a 10th grader in AP Calc, and can not figure out this question: Water is running out of a conical funnel at the rate of 1 inch^3/sec. If the radius of the base of the funnel is 4 in. and the altitude is 8 in., find the rate at which the water level is dropping when it is 2 in. from the top.
Answered by Harley Weston.
Liquid in a drum 2008-11-10
From RAY:
How can I determine the exact amount of liquid in an overturned drum?
Answered by Penny Nom and Victoria West.
How fast is the distance between the airplanes decreasing? 2008-11-10
From Crystal:
At a certain instant, airplane A is flying a level course at 500 mph. At the same time, airplane B is straight above airplane A and flying at the rate of 700 mph. On a course that intercepts A's course at a point C that is 4 miles from B and 2 miles from A. At the instant in question, how fast is the distance between the airplanes decreasing?
Answered by Harley Weston.
1/8, 2/7, 1/2, 4/5, ..., ... 2008-11-06
From katie:
Inductive reasoning, what 1/8, 2/7, 1/2, 4/5, what would be the next two in th sequence.
Answered by Penny Nom.
A belt drives a pulley 2008-11-03
From Rebecca:
A point on a belt is moving at the rate of 30 ft/sec and the belt drives a pulley at the rate of 200 rpm. What is the radius of the pulley?
Answered by Penny Nom.
The prime factorization of one billion 2008-11-02
From Alta:
The prime factorization of 1000 is 2 cubed times 5 cubed. How do you write the prime factorization of one billion using exponents?
Answered by Penny Nom.
A circle tangent to a line and with its centre on another line 2008-11-01
From liza:
Find the equation of the circle of radius squareroot 26 tangent to the line 5x+y=13 and having its center on the line 3x+y+7=0.
Answered by Chris Fisher.
Equations with the variable in the exponent 2008-10-28
From fari:
Solve for x

5 * 4^x-3=40
5^x=25^(x+8)

Answered by Penny Nom.
Tangent line 2008-10-27
From Maddie:
Find a general equation for a line that touches, but does not pass through the function Y=X2(X squared) + BX+C.
Answered by Stephen La Rocque and Victoria West.
A sequence 2008-10-26
From Nicole:
what is the rule to this sequence 3,6,12,24,48.....
Answered by Penny Nom.
The sequence 49,48,46,43,39... 2008-10-26
From Kyle:
What number comes next in the series 49,48,46,43,39...?
Answered by Stephen La Rocque.
The sequence 6,1,6,2,6,4,6,8,6, 2008-10-26
From Kyle:
What number comes next in the series 6,1,6,2,6,4,6,8,6,?
Answered by Stephen La Rocque.
Water is leaking from a conical tank 2008-10-24
From Kimberly:
Water is leaking out of an inverted conical tank at a rate of 12000 cm3/min at the same time that water is being pumped into the tank at a constant rate. The tank has height 6 m and the diameter at the top is 4 m. If the water level is rising at a rate of 20 cm/min when the height of the water is 2 m, find the rate at which water is being pumped into the tank.
Answered by Stephen La Rocque.
Put in lowest terms 6a^2c/8ab 2008-10-24
From hana:
put in lowest terms 6a^2c/8ab
Answered by Stephen La Rocque.
A linear system 2008-10-22
From Tiffany:
solve:

x + y + z = 4
2x - 3y - z = 1
4x + y - 2z = 16

Answered by Penny Nom.
nth term 2008-10-22
From jen:
what is a nth term and how do i find the nth term of any sequence
Answered by Penny Nom.
Exponential form 2008-10-21
From layla:
how we wite 256 in exponential form
Answered by Harley Weston.
A falling block 2008-10-18
From KOBINA:
A 50 kg block being held at rest 20m above the ground is released. the block falls (no friction). how fast is the block traveling (in m/sec) when it has lost 40% of its original potential energy.
Answered by Stephen La Rocque.
The slope of a tangent line 2008-10-18
From Amanda:
If f(x)=square root of (x+4), and the slope of the tangent line at x=21 was 1/n for some integer n, then what would you expect n to be?
Answered by Stephen La Rocque.
Two equations in two unknowns 2008-10-17
From Dushayne:
Please help me in solving this problem:
a. 3x-4y=32
5x+2y=10

b. 2x+3y=11
4x+3y=10

Answered by Penny Nom.
A geometric construction 2008-10-17
From M:
Given any 3 parallel lines on a plane, how to construct an equilateral triangle with each vertex on each line?
Answered by Chris Fisher.
Using the augmented matrix 2008-10-17
From Karlena:
I am supposed to write the augmented matrix of the system and use the matrix method to solve the system. I must show my work algebraically

x+y+2z=30
2x+3y+2z=53
x+2y+3z=47

Answered by Harley Weston.
A tangent line to f(x) = 1/(x - 1) 2008-10-16
From Amanda:
If f(x)=1/(x-1) then what is the slope of the tangent line at x=-2?
Answered by Harley Weston.
Perpendicular lines 2008-10-16
From Hailey:
In the xy-plane, the line with equation 2x + y = 3 is perpendicular to the line with equation y = mx + b, where m and b are constants. What is the value of m?
Answered by Penny Nom.
The volume of a slice from a wedge 2008-10-16
From Jeff:
If I have a wedge shaped object (ie, a triangle of non-specific type extruded along a length) and it is sliced from the two-corner base at one end to the one-point tip at the other, what is the volume of the upper (smaller) part as a percentage of the whole? I hope I have explained it clearly enough. Thanks.
Answered by Harley Weston.
The rate of change of the volume of a cone 2008-10-15
From Barbara:
Suppose that both the radius r and height h of a circular cone change at a rate of 2 cm/s. How fast is the volume of the cone increasing when r = 10 and h = 20?
Answered by Harley Weston.
A cone shaped pile 2008-10-10
From Nadine:
I have a pile of wheat in the shape of a cone. I would like to know how much wheat I have. I have found the equation "V=1/3 pie r squared h" , but it dosn't work! The pile is 7 feet high, diameter is 50 feet, circumference is 185 feet. I also measured the slope 19feet. (Probably not needed) You need to know that wheat weighs 60 pounds per bushel, and I would like to know how many bushels I have. Even if I could receive the # of Volume Bushels, I could convert that. Can you help me??
Answered by Stephen La Rocque.
Nets of Square-Based Pyramids 2008-10-10
From Miss:
How many different nets does a square-based pyramid have?
Answered by Janice Cotcher.
Linear and Angular Velocity 2008-10-10
From Matthew:
The pedal and gear relationship of a bicycle is shown. The radii of the gears are r(sub1) = 5 cm and r(sub2)= 12 cm. The radius of the wheel is r(sub3) = 30 cm. How many rotations per minute of the pedal gear will produce a racing cyclist's speed of 60 km per hr. For a diagram: http://bikepedalsgears.weebly.com/
Answered by Stephen La Rocque.
Two inequalities 2008-10-08
From s:
3y > 4x
2x-3y>-6

Answered by Stephen La Rocque.
A linear equation with fractions 2008-10-07
From Christopher:
1/6(x-18)+1/8(x+8)=x-9
Answered by Penny Nom.
The rise over the run 2008-10-07
From Mak:
what is the ratio of the rise to run?
Answered by Penny Nom.
Logs and exponentials 2008-10-06
From Charles:
I seem to remember you could solve exponential problems using natural logs quite simply without calculators, such as the fifth root of 400 or 33 to the 4th power. All you did was convert the number to its natural log and then just divide by 5 or multiply by 4 and then convert it back. I just remember how to do this. Thanks.
Answered by Harley Weston.
A product of positive and negative numbers 2008-10-03
From kaylou:
Suppose you were given 13 numbers and asked to find their product. Seven of the numbers were positive, and the rest were negative. Would your product be positive or negative? Why?
Answered by Penny Nom.
4^x = 2^x + 12 solve for x 2008-10-02
From Dave:
4^x = 2^x + 12 solve for x.
Answered by Harley Weston.
Lines that pass through a point 2008-09-30
From Abigail:
what is the name for a point that a group of lines pass through that contains the letters c e n p i l?
Answered by Penny Nom.
The vertices of a triangle 2008-09-27
From T.:
how do you find the coordinates of the vertices of the triangle with the sides determined by the graphs of the following equations:4x+3y+1=0,4x-3y-17=0,4x-9y+13=0?
Answered by Harley Weston.
The standard form Ax+By=C 2008-09-26
From Susan:
I am having difficulty changing the point-slope form of the equation of the line y-y1=m(x-x1) to the standard form Ax+By=C. You must express A,B,C in terms of the constants m,x1,y1. I have been able to insert numbers and switch easily from the general to the point slope equation, but I am stumped on how to convert it this way.
Answered by Penny Nom.
5 units from (1,2) and 5 units from y-axis 2008-09-26
From Shaun:
Find the points (x,y) in the plane that are 5 units from (1,2) and 5 units from y-axis. I am more interested in the approach, in general terms, than the numerical values.
Answered by Penny Nom.
Extraneous solutions 2008-09-25
From crystal:
/6x+7/=5x+2
Answered by Penny Nom.
Exponential form 2008-09-25
From Pat:
The African bush elephant is the largest land animal and weighs about 8 tons. Write this amount in exponential form.
Answered by Penny Nom.
Exponential form 2008-09-16
From shawn:
Write the product in exponential form: 5.5.5.2
Answered by Penny Nom.
What is the derivative of (2^sinx)/(logbase4(2x+1))? 2008-09-16
From Jesse:
What is the derivative of (2^sinx)/(logbase4(2x+1))
Answered by Harley Weston.
Solving Exponential Equations Using Logarithms 2008-09-15
From Todd:
7(6^x)=6e^-2x
Answered by Janice Cotcher.
The nth term of a sequence 2008-09-13
From lavett:
what is the Nth term in the sequence when the sequence is 2,4,8,16... and the term numbers are 1,2,3,4...
Answered by Stephen La Rocque.
A line through two points 2008-09-10
From Conor:
Find the equation of the straight line which passes through the points (-2,14) and (8,-1)
Answered by Penny Nom.
An ice cream cone 2008-09-09
From olivia:
Judy has a sugar cone and wants to know how many cubic inches of ice cream it will hold if it is filled completely to the top of the cone and no more. The cone has a height of 4.5 inches and a radius of 1.5 inches.
Answered by Penny Nom.
Collinear 2008-09-08
From Patrick:
On page 17 of my son's 9th grade math book Geometry by Jurgensen, Brown, and Jurgensen (copyright 2004), the authors state from the diagram that you can conclude the points A, B, and C are collinear. In the diagram, the points A, B, and C appear to be situated on a straight line; however, nothing in the diagram states that it is a straight line. I told him that you can't conclude they are collinear because angle ABC could be 179.9 degrees instead of 180 degrees since nothing in the diagram states otherwise. His teacher marked his answer incorrect on a test but I still believe that I am correct. How do conclude points are collinear if nothing tells you that they are situated on a line?
Answered by Harley Weston.
The biggest right circular cone that can be inscribed in a sphere 2008-09-08
From astrogirl:
find the volume of the biggest right circular cone that can be inscribed in a sphere of radius a=3
Answered by Harley Weston.
An exclusion zone around a triangle 2008-09-07
From Awrongo:
A long time ago Mr Gibson found an island shaped as a triangle with three straight shores of length 3 km,4 km and 5 km. He declared an 'exclusion zone' around his island and forbade anyone to come within 1 km of his shore. What was the area of his exclusion zone?
Answered by Stephen La Rocque and Penny Nom.
2^x = 1,000,000 2008-09-06
From Peter:
How do I solve for "x" in the following equation: "2 to the power of x = 1,000,000" ?
Answered by Penny Nom.
A tangent to a circle 2008-09-06
From Jake:
Find an equation of the line that is tangent to the circle x^2 + y^2 = 3 at the point (1,√2)
Answered by Penny Nom.
Parallel and perpendicular lines 2008-09-06
From C:
Can you help me determine whether these lines are parallel, perpendicular or neither and why?

Problem 1: y=3x+2
y=1/3x - 4

Problem 2: y=1/3x + 1/2
y=1/3 x -2

Answered by Penny Nom.
The square root of 2 the the X power 2008-09-05
From Peter:
How can I re-write the expression: The square root of 2 the the X power?
Answered by Penny Nom.
The selling price 2008-09-04
From Robert:
Client wants to sell his property providing he is left with 100000.00$ net. How do I calculate selling price if commission is 6% and taxes on the commission 6% Fed and 7.5%prov. I'm taking the real estate class and this one really got me.
Answered by Penny Nom.
A line parallel to y= 1/2x + 5 2008-09-03
From Michelle:
Find the equation of a line parallel to y= 1/2x + 5 if the required line passes through ( 3,4)
Answered by Penny Nom.
1,679,616 as a power of 6 2008-09-03
From Denise:
express 1,679,616 in exponential form as a power of 6.
Answered by Penny Nom.
A word problem with three numbers 2008-09-02
From Wanda:
"Jabari is thinking of three numbers. The greatest number is twice as big as the least number. The middle number is three more than the least number. The sum of the three numbers is 75. Find the numbers."
Answered by Victoria West.
A cardboard spaceship 2008-08-31
From Lee:
I am building a "cardboard" spaceship for my 4 year old grandson. The cabin is a dishwasher box with a cool control panel and elipse shaped windows. I really need help designing the cone or"nosecone". The diameter of the base is 26", 24" tall, with a 6" diameter opening at the top. Thank you for the help! Lee
Answered by Penny Nom.
The flight of an airplane 2008-08-29
From kobina:
You are traveling on an airplane that is moving at 500 mph in a direction 40deg north of west.Therefore , You are travelling (due to the plane ) at how many mph toward the west.
Answered by Stephen La Rocque.
Solve e^x + e^(x+1) = 250 2008-08-25
From Laura:
"solve this equation correct to three decimals"
e^x + e^x+1= 250

Answered by Penny Nom.
Radii and Chords Create a Non-Right Triangle 2008-08-22
From Beary:
AOC is a diameter of circle O. Line AB is 12, and lines OA and OC (the radii) are 10. Find the length of line BO and chord BC.
Answered by Janice Cotcher.
Function or Not? 2008-08-22
From Katie:
Determine whether y is a function of x: [(x^2)y]-(x^2)+(4y)=0 ....thx
Answered by Janice Cotcher.
Angular & Linear Speed from a Sine Graph 2008-08-19
From Kim:
Kim, a student: I am given a graph with a wave. The amplitude is 5cm and the period is 4cm. I am suppose to find the angular speed. What I need to know is the formula for angular speed and how do I use these numbers to get the correct answer.
Answered by Janice Cotcher.
Joining vertices in a polygon 2008-08-18
From Megan:
I'm trying to develop a relationship between the number of points of a regular polygon and the
(a) number of lines you could draw between those points,
(b) number of triangles you could draw,
(c) number of quadrilaterals,
etc.

Answered by Penny Nom.
Solving for Shared Height of Two Right Triangles 2008-08-17
From Heidi:
find the height of a triangle, which can be split into two right triangles, but the base (50m) is not split equally in half. one end of the base is 40 degrees, while the other is 30 degrees.
Answered by Janice Cotcher.
How do you solve for x when (x^4)-x is greater than or equal to 0? 2008-08-14
From Katie:
How do you solve for x when (x^4)-x is greater than or equal to 0?
Answered by Penny Nom.
BEDMAS 2008-08-12
From Rebecca:
I have 3 questions. 1) I don't really understand BEDMAS.Im going in to the 6th grade and im kind of nervis about it.? 2)When you're doing BEDMAS what does the small 3 or 2 above the other numerals mean? 3)How would you answer this:5+2 x 9 - 9 x 12= ??
Answered by Janice Cotcher.
A linear function 2008-08-07
From Warren:
What are the conditions of the numerical coefficients of the standard form Ax+By=C in Linear Function? How will you transform y=square root of 3 x + 1 in standard form Ax+By=C?
Answered by Penny Nom.
Arc-length and sector-angle 2008-08-06
From Benson:
If chord length, radius are given, How to find the sector angle and arc-length
Answered by Janice Cotcher.
A cone on the end of a cylider 2008-08-05
From Jerry:
Hi. I need to make a plastic cone to fit on the end of a cylinder. The only dimension i have is the diameter of the cylinder. This 553 mm. I need to find the formulae required and how much material will be required.
Answered by Penny Nom.
Find the product of 2^35 and 5^38 in sci. notation. 2008-08-03
From Peter:
I am preparing for a competition and a lot of the non-calculator problems are like find the product of 2^35 and 5^38 in sci. notation. How would you do that?
Answered by Penny Nom.
Two equations in two unknowns 2008-08-01
From James:
Question from james, a student:

3x+2y=9;24x-2y=18

My math teacher says that it is (2,-4)

Can someone please explain how to figure this out by graphing?

Answered by Victoria West.
Right Sided Limit of an Exponential Function 2008-07-29
From joseph:
I am trying to find the limit as X approaches 0 from the positive side of x to the sin of x which look like this lim x^sinx x->0+
Answered by Harley Weston.
Going to Neptune 2008-07-29
From mariah:
how fast would you be going per hour if you wanted to get to neptune in 1 week ?
Answered by Penny Nom.
Trigonometric 2008-07-28
From kiran:
In triangle , Sin@ = ?
Answered by Penny Nom.
Braking Distance Relationship 2008-07-23
From Mr.Adjetey:
you are travelling at 30km/h.According to the rule of thumb,the braking distance is 9 metres when braking normally.How long is the braking distance under the same conditions when travelling at 60km/h?
Answered by Janice Cotcher.
Simultaneous equations 2008-07-23
From Franco:
Solve

3 D + E - F = -10
-2 D - F = -4
-3 D - 4 E - F = -25

Franco

Answered by Penny Nom.
The midpoints of the sides of a quadrilateral 2008-07-22
From JOEL:
A QUADRILATERAL A,B,C,D HAS THE CO -ORDINATE [2,5],[8,7],[10,3]&[0,1] RESPECTIVLY [B,E,G,H] ARE THE MID POINTS OF THE SIDES AB,BC,CD&DA RESPECTIVLY FIND THE MID POINTS OF [FG]&[FH]
Answered by Penny Nom.
The number of digits in a really big exponent 2008-07-21
From Pete:
how would you find the number of digits in a really big exponent without a calculator?
Answered by Penny Nom.
A cube inscribed in a right cone 2008-07-16
From Steven:
A cube is inscribe in a right cone of radius 2 and height 5. What is the volume of the cone?
Answered by Victoria West and Harley Weston.
A reflected line segment 2008-07-14
From Peter:
Segment AB has endpoints A(-1,2) and (3,1). Segment AB is reflected over the y-axis such that A becomes A' and B becomes B'. What is the positive difference between the lengths of segment AA' and segment BB'?
Answered by Janice Cotcher.
Overbooking flights 2008-07-10
From DON:
Overbooking by Airlines This is a simplified version of calculations used by airlines when they overbook flights. They realize that a certain percentage of ticketed passengers will cancel at the last minute. Therefore, to avoid empty seats, they sell more tickets than there are seats, hoping that just about the right number of passengers show up. We will assume that the no-show rate is five percent. For a flight with 220 seats, the airline wants to find how sensitive various probabilities are to the number of tickets it issues. In particular, it wants to calculate
a) the probability that more than 225 passengers show up
b) the probability that more than 220 passengers show up
c) the probability that at least 215 seats will be filled
d) the probability that at least 210 seats will be filled.
To assess the benefits and drawbacks of issuing various numbers of tickets on an airline flight with 220 seats, create a table showing as many different scenarios as possible (table only on one page when printed) and use a second page for your analysis and recommendation to the airline. Which are the good cases, which are the bad cases for the airline?

Answered by Janice Cotcher.
Making Change 2008-07-09
From Peter:
In how many ways can you make 30 cents with quarters, nickels, and pennies?
Answered by Penny.
Exponential form 2008-07-03
From Kent:
How to write 8 365 005 106 in expanded and exponential form? May I request you also to explane how does it solve.
Answered by Stephen La Rocque.
Units digit 2008-07-02
From seemachauhan:
How to find unit digit of various numbers and what are unit digits?
Answered by Penny Nom.
Some phone numbers 2008-06-30
From star:
What do you think the probability of finding the last 3 digits of a telephone number if the 1st digit begins with 7?
Answered by Janice Cotcher and Harley Weston.
A rectangular room 2008-06-30
From Peter:
A rectangular room has a perimeter of 42 feet and an area of 108 square feet. What is the length, in feet, of the shorter side?
Answered by Penny Nom.
Write log_2 (1/8) = -3 in exponential form 2008-06-27
From Jaime:
how do you write log2 (1/8) = -3 in exponential form?

the 1/8 is a fraction and the 2 is a base of 2

Answered by Penny Nom.
How many face does a cone have? 2008-06-21
From Vanessa:
How many face does a cone have?
Answered by Harley Weston.
A line which is perpendicular to another line 2008-06-18
From Emily:
How do you solve perpendicular bisectors with a slope and a point?
Answered by Penny Nom.
(2b^-1c^3)^3 (b^4c^-2)^0 2008-06-05
From ashley:
Simplify this expression

(2b^-1c^3)^3 (b^4c^-2)^0

Please help me. I not sure about the zero on the outside of parentheses.

Answered by Penny Nom.
A quadratic equation 2008-06-03
From Drew:
A solution of x^2-8x=-17 is

-4 or -4+I or 4 or 4+i

Answered by Janice Cotcher.
Substitution method 2008-05-30
From taquicia:
2y-x=3
x=5+y

Answered by Janice Cotcher.
x/4 = 3 1/2 2008-05-30
From Kelsey:
How do you solve for "X" in the problem below?

X
--- = 3 1/2
4

--Kelsey

Answered by Victoria West.
Liquid is being pored into the top of a funnel 2008-05-25
From Stella:
Liquid is being pored into the top of a funnel at a steady rate of 200cm^3/s. The funnel is in the shape of an inverted right circular cone with a radius equal to its height. It has a small hole in the bottom where the liquid is flowing out at a rate of 20cm^3/s. How fast is the height of the liquid changing when the liquid in the funnel is 15cm deep?
At the instance when the height of the liquid is 25cm, the funnel becomes clogged at the bottom and no mo re liquid flows out. How fast does the height of the liquid change just after this occurs?

Answered by Stephen La Rocque.
Net weight 2008-05-23
From Leticia:
Hello, My name is Leticia, I am having trouble with Net weight and I need your help. When I am in class my friends answer questions that they say are easy,but I have no clue what's happening, we are going to have a test soon so please send me fast.
Answered by Janice Cotcher.
A 9 team flag football tournament 2008-05-22
From James:
I like to run a 9 team flag football tournament where each team is guaranteed 3 games prior to playoffs. All games must have playoff implications and must not be a consolation game.
Answered by Victoria West.
A season schedule for 9 teams 2008-05-22
From James:
would like to put together a season schedule for 9 teams with each team playing three or 4 games leading to a 1 to 9 ranking.
Answered by Victoria West.
Nine digit numbers 2008-05-21
From Alex:
List of Nine digit numbers, that can be divided by nine?
Answered by Janice Cotcher.
The cube of -2y in exponential form 2008-05-19
From Xiu:
I dont get this question.. Write "the cube of -2y" in exponential form. I know the answer is -(2y)^2 but i really don't understand why. Can you explain how to do this whole problem for me?
Answered by Leeanne Boehm.
Guy wires for a tower 2008-05-19
From larissa:
a radio tower 500 feet high is located on the side of a hill ( the hill has an inclination to the horizontal of 5 degrees.) How long should two guy wires be if they are connected to the top of the tower and are secured at two points 100 feet directly above ( up the hill ) and directly below the base of the tower?
Answered by Penny Nom.
Elevation gain 2008-05-18
From Mark:
How do you calculate elevation gain on a treadmill? e.g. 5.0 miles (26,400ft) @ 15% incline. I have been estimating it as 26,400 x .15 = 3,960 ft of elevation gain. Hopeful Mount Rainier Climber
Answered by Victoria West.
How old are the sisters? 2008-05-11
From George:
Theresa is four years older than her younger sister Joni. Twelve years ago, she was twice as old as her sister. Find the present age of the two siblings.
Answered by Penny Nom.
A linear model 2008-05-09
From Shelby:
How would you write a linear model to represent the population of a city that has a population of 547,725 and a growth rate of -25,195 per year with t represents the number of years since 1994?
Answered by Penny Nom.
A net for a hemisphere 2008-05-06
From Julie:
how would you draw the net of a hemisphere?
Answered by Walter Whiteley.
The inverse of a function 2008-05-03
From keith:
please help me find the inverse of this function:

h(t) = 2 + 4 ln(1-5t)

Answered by Stephen La Rocque.
-5x < 25+10 2008-04-25
From Courtney:
What is the answer to -5x<25+10
Answered by Stephen La Rocque.
15 + 0.10n = 0.25n, solve for n 2008-04-24
From monica:
15 + 0.10n = 0.25n

Solve for n.

Answered by Penny Nom.
What is the altitude of the plane? 2008-04-23
From tiana:
If a plane is taking off and its climbing at 380 feet per minute at 205 MPH, what is the altitude of the plane after it traveled 115 miles?
Answered by Penny Nom.
How is the square root of 3/4 is greater than 3/4? 2008-04-22
From Serena:
How is the square root of 3/4 is greater than 3/4?
Answered by Penny Nom.
The isoperimetric inequality 2008-04-21
From Dutch:
I'm searching for the proof that the sphere has the smallest volume of any figure and maximizes the volume of any figure.
Answered by Chris Fisher.
x/3 -5 = 3(x-2) 2008-04-20
From Mrs:
I am having trouble helping my son solve the following equation:

x/3 -5 = 3(x-2)

Answered by Penny Nom.
The volume of a stone 2008-04-17
From sara:
a rectangular tank 30 cm long and 20 cm wide is filled with water to a depth of 8 cm. When a stone was put in, the water level rose to 11 cm. Find the volume of the stone. (Assume that the stone is completely under water.)
Answered by Penny Nom.
A parallel line to a given line 2008-04-12
From john:
A parallel line to a given line through a given point not on the a given line
Answered by Harley Weston.
The slope intercept form 2008-04-11
From Dave:
Hi, why is there a y-intercept in the slope intercept form?What does the y-intecept have to do with the equation?
Answered by Penny Nom.
A truncated cone 2008-04-11
From jason:
i need to know how to figure the cubic feet of a cone that the top is 72" wide the bottom is 25" wide and it is 48" tall. is there a specific formula for this?
Answered by Penny Nom.
The amount of a washing machine 2008-04-07
From Eddy:
we have a washing machine with a specific cylinder size, which we could calculate the volume as a regular cylinder formulas. The question is, if we fill the washer cylinder with water at a specific height on water , example 15 cm height, how can we find the volume of water at that specific height?
Answered by Penny Nom.
1-(1-P)^75=0.05 2008-04-05
From Amy:
1-(1-P)^75=0.05
Answered by Stephen La Rocque and Penny Nom.
The shortest distance from a point to a line 2008-04-03
From Katherine:
How do I find the shortest distance from the point (3,5) to the line x = -2
Answered by Stephen La Rocque.
Two consecutive odd numbers 2008-04-01
From prinsilla:
of all pairs of consecutive odd integers whose sum is greater than 75. find the pair whose sum is least?
Answered by Stephen La Rocque.
The exponential form of x radical y to the third power 2008-03-30
From Jennifer:
what is the exponential for of x radical y to the third power
Answered by Penny Nom.
The nth term 2008-03-16
From ryan:
need help figuring nth term for 1, 2, 5, 14, 41...... could you please help
Answered by Penny Nom.
1, 1, 2, 3, 5, 8, 13, ___ ___ ___ 2008-03-13
From Karen:
What is the rule for the following pattern, and what would come next?

1, 1, 2, 3, 5, 8, 13, ___ ___ ___

Answered by Penny Nom.
Estimate the cube root of 270 2008-03-11
From Tom:
(a) By plotting suitable graphs, estimate to one decimal place the cube root of 270.
(b) With reference to your answer to part (a), use 8 iterations of a Bisection Search to refine your estimate. Use the nearest whole numbers either side of your estimate from part (a) as starting values.
(c) Using either of your starting values from part (b) as first guess, use the Newton-Raphson method to find the true value of the root (to 6 decimal places). Repeat using the other starting value from part (b) and compare the two results.

Answered by Harley Weston.
A linear system 2008-03-10
From sandhiya:
one group of people purchased 10 hotdogs and 5 soft drinks at a cost of $12.50. a second group bought 7 hotdogs and 4 soft drinks a ta cost of $9.00 find the cost of a single hot and a single soft drink.(using the row echelon form to solve)
Answered by Penny Nom.
Two perpendicular lines 2008-03-07
From Daphne:
Does anyone know how to find a line perpendicular to -3x=4y=20 and passes through the point (-12,0)?
Answered by Stephen La Rocque and Penny Nom.
How large is a container that will hold 3.5 cu meters? 2008-03-05
From Robert:
If a company allows you to ship personal belongings while on an over seas assignment and the figure is 3.5 cubic meters. Just how much is the total area in square feet?? 10X10X10 or??? I just wanted to understand the size of a container that would hold 3.5 cubic meters!!
Answered by Harley Weston.
The angles of a triangle 2008-03-01
From Allen:
How to find the other two angles of a scalene triangle if one angle is given
Answered by Steve La Rocque and Penny Nom.
Simultaneous equations 2008-02-29
From CONOR:
I was wondering if you could help me with this problem

7x - 5y = -1
3y = 4x

Answered by Penny Nom.
A package of supplies is dropped from a plane 2008-02-29
From beth:
An airplane flies west at a speed of 200m/s. A package of supplies is dropped from the plane to some campers. if the plane is at an altitude of 1000meters, how far from the campers should the package be dropped to land near them? ignore wind resistance.
Answered by Stephen La Rocque.
A line drawn between 2 vertices 2008-02-26
From Arshad:
what is a line drawn between 2 vertices which are not next to each other called.
Answered by Penny Nom.
A fractional inequality 2008-02-20
From ed:
solve the inequality
(x+3)^2 (2-x)
___________ < 0
(x+4) (x^2 - 4)

Answered by Penny Nom.
Find the next terms 2008-02-20
From Darrin:
I was given a number sequence and can't figure out the next terms to this

2,9,20,35,66,91......

Answered by Penny Nom.
A complex quadratic 2008-02-18
From Ash:
z^2-(6+2i)z+(8+6i)=0

Solve for Z

Answered by Steve La Rocque and Penny Nom.
The equation of a line 2008-02-17
From Jennifer:
Given the point A(3,2)B(5,8) find the equation of the line AB ( in standard form )
Answered by Penny Nom.
The slope intercept form of a line 2008-02-17
From chris:
find the slope and y intercept of the line (2x+4)-3y=11(x-1) is this equation in slope point form.
Answered by Penny Nom.
Exponential form 2008-02-16
From annna:
i already got the answer for what is the prime factorization of 648 which is 2x2x2x3x3x3x3 but how do i write that answer in exponential form?
Answered by Penny Nom.
The length of the third side of a triangle 2008-02-16
From mary:
I have an angle of 72 degrees and each of the sides are 5' long. What is the distance from each of the ends of the 5 feet to form a triangle.
Answered by Stephen La Rocque.
Growth factor and growth rate 2008-02-13
From william:
what is the difference between growth factor and growth rate?
Answered by Stephen La Rocque and Harley Weston.
Classifying a triangle 2008-02-07
From kevin:
scalene triangle 8 ft base right side 9.5 left side 12 ft what is the angles
Answered by Penny Nom.
Why is the number 1 neither prime nor composite? 2008-02-07
From ronni:
why is the number 1 neither prime nor composite?
Answered by Harley Weston.
Two spinners 2008-02-05
From Kim:
Zack has two spinners with numbers on them. the probability of spinning a 7 on both spinners is 3/20. if the probability of spinning a 7 on the first spinner alone is 1/4 what is the probability of getting a 7 on the second spinner alone?
Answered by Penny Nom.
A inear system 2008-02-03
From Kelly:
is (-1,5) a solution of each system? y=-x+4 y=-1/5x
Answered by Penny Nom.
Two equations in x and y 2008-02-01
From leslie:
6x+8y=40 4x+5y=25
Answered by Penny Nom.
The standard form of a line 2008-01-30
From Thomas:
I'm attempting to get this in standard form

slope=3;(1,5)

Answered by Penny Nom.
Negative Exponents 2008-01-29
From Bill:
t to the negative fourth power inside parenthesis to the ninth power
OVER
t to the negative fourth power inside parenthesis to the third power

I am confused on how to start the problem. What I did was invert the Ts to make them positive and I then multiplied the powers. I am not sure if this is the correct process.

Answered by Harley Weston.
Belled-out pier 2008-01-28
From Gina:
I need to know how to find the total yards needed to fill a concrete pier that is 54"/ 108" and 26' deep. That is...54" @ the top of the pier belled to 108" @ the bottom...26' deep.
Answered by Stephen La Rocque.
How do I convert mcg to ml? 2008-01-27
From David:
How do I convert mcg to ml?
Answered by Stephen La Rocque.
O.T.T.F.F.S.S 2008-01-25
From Omar:
What is the next letter in the following sequence:

O.T.T.F.F.S.S

Answered by Penny Nom.
Linear feet to metres 2008-01-25
From Jillian:
i have a measurement of something in linear feet. i dont know what a linear foot is so i want to convert it to meters to have an idea. how do i convert.
Answered by Penny Nom.
The standard form of a line 2008-01-24
From James:
I need to put it in Standard from ax+by=c the problem is (-4,0); parallel to y=-2+1 how can I put this in Standard from
Answered by Stephen La Rocque.
x^3 y^(-6) 2008-01-24
From Bill:
I need help completing this negative exponents problem:

x to the power of 3 multiplied by y to the power of negative 6.

What would the process to solving this problem be?

Answered by Penny Nom.
Find the slope of the line with equation 7-3y=2x 2008-01-23
From nell:
Find the slope of the line with equation 7-3y=2x
Answered by Stephen La Rocque.
A linear system 2008-01-22
From Lisa:
I am supposed to use augmented matrices to solve this system of equations. x - 3y = 5; 4x - 12y = 13. I think it is unsolvable but I want to be sure I am not just giving up. Thanks in advance for your help! Lisa.
Answered by Stephen La Rocque and Brennan Yaremko.
1/2, 1/2, 3/8, 1/4, 5/32, 3/32, 7/128 2008-01-22
From Neil:
Find the next two terms in the following number sequence

1/2, 1/2, 3/8, 1/4, 5/32, 3/32, 7/128

Find a general rule for the nth term of the sequence

Answered by Penny Nom.
The cosine of an angle 2008-01-21
From Kristine:
Find measure of unknown side cosA=0.5
Answered by Harley Weston.
The equation of a line 2008-01-20
From Sara:
The question is:

Write the equation of a line that is parallel to y=3x-2 through the point (-5,9). I think the correct equation for this is y=3x+24. But I have to write it in standard form, and I can't seem to figure out how to do that. Please answer soon! Thanks!

Answered by Stephen La Rocque and Penny Nom.
The line through (-22 ;-1) and (-23, -2) 2008-01-20
From Zachery:
i have a slope that equals 1/1 and my coordinates are (-22;-1) and (-23;-2) and i am trying to change it into the equation y=mx+b and the farthest that i've gotten is y=1x_ (i can not find out how to find b.) So i was wondering if you could help me find out what b is because i have got no clue!!!!!
Answered by Penny Nom.
The angles of a triangle given the three sides 2008-01-17
From Lucy:
Is there a way to find the angles of a triangle just by knowing the lengths of it's sides? It seems like the would be a relationship between the two, but I'm not sure.
Answered by Stephen La Rocque and Harley Weston.
How many rolls of stock do I need to buy? 2008-01-16
From Dan:
How many rolls of stock do I need to buy?
Given:
The rolls are 40" wide, 20" roll o.d., 4" core o.d., gauge is 0.008" I need to cut these master rolls in 6" strips (4" of waste per master roll) and I need 1,270,500 linear feet of this strip.

Answered by Stephen La Rocque.
Two solutions using the law of sines 2008-01-14
From Kate:
I am working on the Law of Sines and I have a problem that says: Find a value for b so that the triangle has 2 solutions.

I am given that A = 36 degrees and a = 5. Now, I learned that for a triangle to have 2 solutions, h < a < b. BUT...my answer key says the answer is: 5 < b < 5/sin 36. I can't figure out how to make this fit with h < a < b.

Answered by Harley Weston.
Linear systems 2008-01-10
From Sandra:
I have two questions. Please solve both. Thank you.

1.) In your chemistry class, you have a bottle of 5% boric acid solution and a bottle of 2% boric acid solution. You need 60 milliliters of a 3% boric acid for an experiment. How much of each solution do you need to mix them together?

2.) You plant a 14-inch hemlock tree in your backyard that grows at a rate of 4 inches per year and an 8 inch blue spruce that grows at a rate of 6 inches per year. In how many years after you plant the trees will the trees be the same height? How tall will each tree be? Thank you. I did not understand neither problem, so I could not show any work. Please help.

Answered by Stephen La Rocque and Harley Weston.
Forces on an inclined plane 2008-01-10
From Ron:
A body that weighs 540lbs is caused to slide up an inclined plane with a uniform velocity by a force that acts parallel to the plane. For each foot of horizontal distance, there is a vertical rise of 2in. If the coefficient of sliding friction is 0.16, what force is required to move the body?
Answered by Stephen La Rocque.
How do I write the following in exponential form? 2008-01-09
From Mandy:
How do I write the following in exponential form?

( square root of a^-2 b)^5

Answered by Penny Nom.
Exponential form 2008-01-08
From liam:
i was wondering if anyone could help me with this question and tell me how and what the did; exponetial form of 4 326 180 501
Answered by Stephen La Rocque.
Donuts and bagels for the first period class 2008-01-08
From Karen:
Mr. Pace bought breakfast for his first period class. Ten donuts and fifteen bagels cost a total of $15.75. Mrs. Pace bought breakfast for her class. Five donuts and twenty bagels cost a total of $17.00. Set up and solve a system of linear equations to determine the cost of one donut and one bagel.
Answered by Penny Nom.
How far is the jet from the lighthouse? 2008-01-07
From Natalie:
Question: A ship spots a lighthouse that is 53m high, at an angle of elevation of 7 degrees that is directly north of the ship. The same ship spots a jet travelling N62E at an altitude of 1500m with an angle of elevation of 15 degrees. How far is the jet from the lighthouse?

Natalie

Answered by Harley Weston.
The radius of a planet 2008-01-06
From Ben:
Two people who are both h feet tall are standing on a spherical planet. One person walks a distance d in feet away from the the other person. At this point, the person walking turns around and can no longer see the top of the other persons head. What is the radius of the planet?
Answered by Stephen La Rocque and Harley Weston.
What is the speed of the plane in still air? 2008-01-05
From oscar:
A plane flies 400 miles with the wind and 340 miles against the wind in the same length of time. If the speed of the wind is 24mph, what is the speed of the plane in still air?
Answered by Penny Nom.
The shortest distance to a line 2008-01-04
From nana:
3y+4x-12=0 is the equation of the line AB on a graph......the line AB joins both y and x axises together.....point A is on y axis and its value is 3....point B is on x axis and its value is 4.....Question : from point O(0,0).find the point ont the line AB
Answered by Stephen La Rocque.
Multiplying exponentials 2008-01-02
From jessie:
Can you please explain how to put x^3 y^2 times x^1 y^3 in simplest exponential form?
Answered by Penny Nom.
Radicals in exponential form 2008-01-02
From Andre:
The title of the section in my textbook is to write each of the following radicals in exponential form.

My question is how do u write the squareroot of 10 in exponential form?

Answered by Penny Nom.
How would one find the radius? 2007-12-29
From Ned:
Given an arc with length of 192 inches (don't know chord length), and arc height of 6 inches, how would one find the radius?
Answered by Stephen La Rocque and Harley Weston.
Finding the equation of a line from its graph 2007-12-25
From ken:
I have a graph and it is asking me to find the slope of each line, and then write its equation. the graph show a negative slope. Would appreciate the answer if you could show me how to do this question.
Kind regards: Ken

Answered by Stephen La Rocque.
Smallest cone containing a 4cm radius inscribed sphere 2007-12-19
From Eva:
A sphere with a radius of 4cm is inscribed into a cone. Find the minimum volume of the cone.
Answered by Stephen La Rocque.
Linearize a graph 2007-12-19
From Justene:
My daughter has a physics lab where she must linearize a graph of acc vs. mass. I have no idea how to help her.
Answered by Stephen La Rocque.
Negative numbers 2007-12-14
From mannal:
what negative integers do we use in the real world? like -30 temperature?
Answered by Penny Nom.
Solve sin(x)=x^2-x 2007-12-11
From ming:
is there anyway you can solve
sin(x)=x^2-x without a calculator?

Answered by Stephen La Rocque.
A math trick 2007-12-10
From Megan:
I need to write a mathematical explanation of why this works!

Start with a four digit number. (a positive integer, and all digits can NOT be the same. At least one must be different)
Rearrange that four digit number.
Subtract the smaller 4-digit number from the larger.
Now circel one digit. (canNOT be zero, because that is already a circle)
Now re-write that number excluding the circled digit.
Compute the sum of the digits.
Now write down the next multiple of 9 that is larger than the sum.
Subtract the Sum from the multiple. (multiple - sum of digits)
Report Difference = to number circled.

The resulting number should be the number that originally circled.

Answered by Penny Nom.
Plotting a function 2007-12-06
From Paula:
My son has a problem that says plot the point: 2x-3y=9
I have explained to him that there must be a function for y in order to plot that point, but he doesn't think so. The question above that one says find the function for y, and I assume we should use that function. Am I wrong? Can we plot a point without a y function.

Answered by Stephen La Rocque.
System of equations 2007-12-06
From Jenn:
change the equation,x-y=4 to form y=mx+b the solution to the system of equations y=2x and y=-x+3 is
Answered by Stephen La Rocque.
Chicken and goat feet 2007-12-05
From Kim:
Old McDonald raises goats and chickens. The animals have a total of 100 heads adn 360 feet. How many goats and how many chickens does Mr. McDonald have?
Answered by Stephen La Rocque and Penny Nom.
Ramp height 2007-12-03
From Steve:
I need to find the height of a angle. 23 degree angle at 12 feet of length is how high from the ground?
Answered by Stephen La Rocque.
Exponential form 2007-11-30
From Sasha:
How do I write the number 127398 in exponential for?
Answered by Penny Nom.
An augmented matrix 2007-11-29
From Beth:
I am having troubles fiinding this augnmented matrix. I know how to do it but everytime i do it it get a different answer and its not the right answer. Could you help me to make sure I do it right?

Here is the problem:
4x - 3y = 5
2x +9y = 6

Answered by Harley Weston.
Multiplying an inequality by a negative number 2007-11-27
From Lisa:
Why is it that you have to switch the sign of the inequality when you divide or multiply by an negative number to solve for a variable?
Answered by Penny Nom and Walter Whiteley.
Straight lines 2007-11-26
From Divyansh:
hello i am in eleventh class and am, preparing a project on straight lines i cant really find uses of straight lines and its equations in daily life i am also thankful to you in advance and am waiting for your answer eagerly because i need to submit my project only this week thanking you
Answered by Penny Nom.
The definition of the sine function 2007-11-22
From Indrajit:
I need a explanation in this theory.......if sinθ = p/h...then
sin 90 deg. = p/h
or 1 = p/h
or p=h .....how can a perpendicular be equal to a hypotenuse.???

Answered by Harley Weston.
Exponential Growth 2007-11-19
From lisa:
Hi i am a 8th grader who is learning about algebra and we are having test on exponent and i don't really get it i asked my teacher and she told me i should pay more attention so i want to know what exponential Growth means
Answered by Penny Nom.
The vertex of a cone 2007-11-18
From miriam:
how many verticies does a cone have
Answered by Penny Nom.
Two lines 2007-11-18
From Kate:
represent qraphically the equations of these lines and their intersecting points

x-y=2 3x+y=11

Answered by Penny Nom.
Exponential form 2007-11-16
From ashley:
write a sentence for exponential form.
Answered by Penny Nom.
Why do they use the letter b in y=mx+b? 2007-11-15
From virginia:
why do they use the letter b in y=mx+b? im asking this because my math teacher said he will give extra points to anyone who finds this out first and i was hoping you guys would be the ones who could help.
Answered by Penny Nom.
x^2 + x^3 = n^2 2007-11-14
From Rapin:
x and n are the whole number and less than 100 , x^2 + x^3 = n^2, please help to solve this equation.
Answered by Penny Nom.
A particle moves in the xy-plane 2007-11-12
From Russell:
A particle moves in the xy-plane with

X = 2t^3 - 12t^2 + 18t

Y = 3t^4 - 28t^3 + 72t^2

find an equation of the line tangent to the given curve at t_0_ = 1

note: t_0_ is t subscript 0

Answered by Harley Weston.
The sequence 3,12,26,45.... 2007-11-12
From Tra:
I can seem to find the realtionship in these sequence of numbers, I am looking for the next three. How can I determine that, I have tried everything??!!!

3,12,26,45....

Answered by Penny Nom.
Cones and pyramids 2007-11-10
From Eric:
I have a question regrading the differences between a cone and a pyramid. In my son's Maths workbook, a cone is always referred to as a pyramid, which confuses me very much. I understand that a pyramid is a special case of a cone and therefore you can refer a pyramid as a cone but not the other way around. Am I correct?
Answered by Harley Weston.
Distributing prize money 2007-11-08
From Barbara:
Question from Barbara, a student: In writing competition, the first place winner receives 1/2 of the prize money. The second runner up receives 1/4 of what the winner won. what was the total amount of prize mone distributed if the winner receives $6,000?
Answered by Penny Nom.
Why is the number 1 neither prime nor composite? 2007-11-08
From amarilis:
why is the number 1 neither prime nor composite?
Answered by Penny Nom.
I have a piece of material 44 inches by 78 inches 2007-11-02
From Marianne:
I have a piece of material 44 inches by 78 inches. How many linear yards can I get from it?
Answered by Penny Nom.
The sum of the ones 2007-10-29
From Hangsun:
When 2 two digit numbers are added what is the greatest possible sum of the ones?
Answered by Penny Nom.
Constantly accelerated motion 2007-10-29
From Priyanka:
A, B and C are three points lying in that order on a straight line. A body is projected from B towards A with speed 3 m/s. The body experiences an acceleration of 1 m/s^2 towards C. If BC =20 m, find the time taken to reach C and the distance travelled by the body from the moment of projection until it reaches C.
Answered by Stephen La Rocque.
collinear points 2007-10-26
From Carl:
the points H,S,D,K,L, and B are collinear.
KS+SB=KB
DH+HS=DS
DH+HK=DK

point L and B are not between any other labeled points on the line.
<------------------------------------------------------------------------->

Answered by Penny Nom.
A graph with 100 vertices 2007-10-24
From amarjeet:
let g be a graph with 100 vertices numbered 1 to 100. Two vertices i and j are adjacent only if i-j=8 or i-j=12. The number of connected components in g are:
1. 8
2. 12
3. 4
4. 25

Answered by Penny Nom and Victoria West.
A line parallel to a given line 2007-10-20
From Samaira:
Given Y=2/3 + 3/4x
Give the equation that is parallel to the given one and which touches the x-axis in x=3

Answered by Leeanne Boehm and Stephen La Rocque.
Stanley Cup costume (truncated cone pattern) 2007-10-19
From Janet:
You hockey fans will love this question. I am making a Stanley Cup halloween costume and need a flat pattern for the bowl portion. I believe a truncated cone will work nicely. Base circumference needs to be 32" (10.19 diameter). Top circumference needs to be 44" (14 diameter). It needs to be 8" high. Thanks for your help.
Answered by Stephen La Rocque.
0 = (1/t) - [ln(1+r) ln(t)] 2007-10-18
From Brandi:
Objective: solve for t, where r = 0.05

0 = (1/t) - [ln(1+r) ln(t)]

Answered by Penny Nom.
Surface area of an open-ended cone 2007-10-16
From Lorne:
What is the surface area of an open ended cone? Measured at 10' high, 16' diameter on the bottom and 2' diameter at the top.
Answered by Stephen La Rocque.
Four triangles in a square 2007-10-15
From Kristina:
A square with side lengths of 6 cm is divided into 3 right triangles and a larger isosceles triangle. If the three right triangles have equal area, find the exact area of the isosceles triangle.
Answered by Stephen La Rocque.
Domain and range 2007-10-12
From Dawn:
Hi, I've been out of school for 8 years and recently picked up a math correspondence course. I'm having trouble trying to figure out the range and domain of a linear function. I've read everything I can find in my text, lessons and on your site and I still can't figure out what I'm suppose to be doing. The function is y=2x+1. Please help.
Answered by Penny Nom.
A dinner club schedule 2007-10-12
From irv:
The problem is that I have 12 couples, 1 through 12, in a dinner club, and each couple is host once with 3 of the other couples. he dinners will take place over 4 months. I am trying to not repeat any of the couples. We had the formula for 16 and 20 couples but not for 12. Are you able to help with the groupings for 12. Thanks very much. Irv
Answered by Victoria West.
77,49,36,18,... 2007-10-11
From Kim:
Find a pattern and predict what comes next in this sequence 77,49,36,18,________________.
Answered by Penny Nom.
Substitution method 2007-10-11
From Kevin:
3xx+2y=-36-y=11
Answered by Stephen La Rocque.
Area of a quadrilateral 2007-10-10
From Courtney:
how would i find the area of a quadrilateral..
the sides are a (/) is 6cm, b (—) is 9 cm, and c (\) is 7 cm..
the angle between a and b is 140 degrees and b and c is 115 degrees..

Answered by Stephen La Rocque.
Coin jar 2007-10-07
From a student:
Sally empties his jar of coins. It contains $3.75 in nickels, dimes, and quarters. The number of dimes is twice the number of nickels and the number of quarters is three less than the number of nickels. Determine how many nickels, dimes, and quarters were in the in the jar.
Answered by Stephen La Rocque.
Doubling every day 2007-10-07
From Jerrie:
Patti saved pennies for 10 days. The first day she saved 1 cent. Every day after that she saved twice as much as the day before. How much money did Patti save altogether?
Answered by Stephen La Rocque.
Solving four simultaneous equations (system of four linear equations) 2007-10-07
From Johan:
I need some help in solving this question
x + 2y - 3z + 4w = 12
2x + 2y - 2z + 3w = 10
0 + y + z + 0 = -1
x - y + z - 2w = -4

Answered by Stephen La Rocque.
Find the equation of a line given two points 2007-10-05
From Tiffany:
The equation for the line through the points (11,2) and (18,12) can be written in the form Ax+By=C. Find A and B.
Answered by Victoria West.
Equation of a tangent line to a function 2007-10-04
From Princess:
If f ' (x) = 3x^2 +1, find the equation of the tangent line to f(x) = x^3 + x at x= -1.
Answered by Stephen La Rocque.
how do you get 9^5-9^4/8 to equal 9^4 2007-10-01
From nathen:
how do you get 9^5-9^4/8 to equal 9^4
Answered by Stephen La Rocque.
Two power expressions 2007-09-29
From James:
find the values for a and b (a*x to the power of -1*y)(-1/2*x*y to the power of b)=6*y to the power of -6 please show me how to do this. Can I begin by taking out the powers?
Answered by Leeanne Boehm.
Size of a sphere fitting inside a cone 2007-09-27
From Juan:
I am supposed to find the largest sphere that will fit into a cone. I am assuming is a maximizing problem, but I am not sure of what relation (between a cone and a sphere) to use.
Answered by Penny Nom and Stephen La Rocque.
Exponents 2007-09-26
From Calvin:
Exponential form 5 3exponent x 5 4exponent divide 5 2exponent
Answered by Stephen La Rocque.
Nine dots 2007-09-25
From al:
Connect the dots.
There is nine dots, three across and three down.
Connect the nine dot with the pencil only using four line and never lifting the pencil off of the paper.

Answered by Stephen La Rocque and Harley Weston.
Exponential regression 2007-09-24
From Anonymous:
I need help in finding this. Find a formula for exponential regression. Determine how to calculate a and b in the formula. Y = ba^x where y is as a function of x.
Answered by Penny Nom.
Large exponents 2007-09-24
From Kira:
Is there an easy way to calculate a number with a large exponent? For example, 2(10)^35.
Answered by Stephen La Rocque and Victoria West.
12 men can complete a work in 8 days 2007-09-23
From farah:
12 men can complete a work in 8 days. 3 days after they started working, 3 more men joined. In how many days will all of them together complete the remaining work.
Answered by Stephen La Rocque.
Finding equations, intersection point of two lines at right angles 2007-09-22
From Yaz:
Find the equation of the line joining A(-1,-9) to B(6,120). Another line passes through C(7,-5) and meets AB at rigth angle of D. Find the euation of CD and calculate the co-ordinates of D.
Answered by Stephen La Rocque.
The incline of my yard as it slopes away from my house 2007-09-22
From Janet:
I need to calculate the degree (or is it percent?) of incline of my yard as it slopes away from my house. Do I visualize a right angle formed by a level line from the bottom of the slope to the side of the house, and then compare the line of the slope against the right angle to get the degree? Or is an incline calculated by the measurement of the rise in 100 feet? For instance, if the ground rises 3 feel in a distance of 100 feet, is that a 3% incline? And if it rises 4 feet in a distance of 25 feet, is that a 16% incline? Thanks so much.
Answered by Stephen La Rocque.
Two equations in two unknowns 2007-09-22
From Mary:
Having problems doing this problem, looking for a solution with the work. I would like to see how you got your answer, to see what I was doing wrong.

solve using the substitution method, is there "no solution" or "infinitely many solutions"

4x+y=4
2x+8y=0

Answered by Stephen La Rocque and Leeanne Boehm.
Points that are 15 units from the origin 2007-09-20
From Paula:
Find the coordinates of any point(s) 15 units away from the origin with an x-coordinate of 9.

I was given the answer: (9,12) and (9,-12), but I do not understand how these numbers were calculated. Thank you for your help!

Answered by Stephen La Rocque.
Solving an equation with fractions 2007-09-20
From Len:
I am having a brain cramp or maybe I just forgot some basic math, but I am having trouble solving for "r" in your truncated cone example where r/(r+w)=r/R or r/(r+282)=911/1728. Could you refresh my memory by showing the steps to solve for "r"?
Answered by Harley Weston.
Tens and ones 2007-09-16
From julia:
My son has come home with a math question "what i know about 10s and 1s" we're totally stumped any information you can provide would be greatly appreciated
Answered by Harley Weston.
The contents of a silo 2007-09-14
From Steven:
I need to know how to find the weight of the contents of a silo that is 4 feet across and is funnel shaped at 60 degrees
Answered by Penny Nom.
Exponential form 2007-09-12
From Ericha:
How do I express 1,679,616 in exponential form as a power of 6 using a calculator?
Answered by Stepehe La Rocque and Penny Nom.
What are the next two terms? 2007-09-11
From Sassy:
I can't figure out what the next two integers in this pattern are 3,9,6,12,9 are.
Answered by Stephen la Rocque.
Segments on a line 2007-09-10
From Cathy:
This question was on my daughter's geometry assignment. Write a general rule of formula for finding the number of segments that can be named by a given number of points on a line. For example, 2 points on a line = 1 line segment; 3 points on a line = 3 segments; 5 points on a line = 10 segments.
Answered by Penny Nom.
nth term in this series 2007-09-07
From danii:
im trying to solve the nth term for this pattern.

1 3 6 10 15 21 28

any help would be appretiated

Answered by Stephen la Rocque.
The next term 2007-09-07
From Ariel:
Please explain to me why -3^2 is a -9. One calculator give me an answer of positive 9.

WHAT IS THE NEXT TERM IN Each sequence/ What is its term number?

1/2, 2/3, 3/4, 5/6 6/7

1, 1/4, 1/9, 1/16

Answered by Stephen la Rocque.
The slope of a line 2007-09-06
From Danielle:
Hello. I need some help with this: i am supposed to find the equation of a line with a point (2/3,5) and (-5/6,-4) the answer in the book says y=6x+1. My question is how did they get for the slope?
Answered by Penny Nom.
Perpendicular lines 2007-09-06
From dinesh:
we know that for two perpendicular lines m1*m2=-1perpendicular but this is not true for x-axis and y-axis?
Answered by Harley Weston.
Prime factors in exponential form 2007-09-06
From Michelle:
how would you write this: factor 48 as a product of primes written in exponential form
Answered by Stephen La Rocque.
Exponential form 2007-09-05
From jeanette:
the African bush elephant is the largest animal and weighs 8 tons. write this amount in exponential form.
Answered by Stephen la Rocque and Penny Nom.
X^3 = 3/10 2007-09-05
From ioana:
If we have X at power 3 that result equally with 3/10 haw much X is ??
Answered by Penny Nom.
Variable expressions - part 2 2007-09-04
From lisa:
how do I write a variable expression for word phrase example x less than = 2 or 2 less than p 4 fewer than job please help
Answered by Penny Nom.
Exponential form 2007-08-31
From Victoria:
The cost of each space shuttle is about $10,000,000. Write this amount in exponential form.
Answered by Stephen La Rocque.
Exponential form 2007-08-29
From Emily:
Write the Exponential Form of 125 x 25 and 81 x 27.
Answered by Stephen La Rocque.
Rotational energy needs 2007-08-29
From will:
how much energy in KW do i need to start and subsequently rotate a 4 ton fly wheel of 1 metre radius at 12 rpm?
Answered by Stephen La Rocque.
2,6,7,21,22,66,67 2007-08-28
From EDWIN:
this doesnt make any sense to me....... i cant find the pattern...

2,6,7,21,22,66,67

Answered by Leeanne Boehm and Stephen La Rocque.
A truncated cone 2007-08-24
From JUAN:
i need to create a template for a cone that has a 4 " opening at top, a 14" base and 12" tall.Can u tell me how to achieve this ?
u have similar problems but when i try to do it , is not coming up right , please help..

Answered by Harley Weston.
Two-column proof for a circle geometry problem 2007-08-24
From Kendra:
i have to prove that tangents to a circle at the endpoints of a diatmeter are parallel by stating whats given, whats to prove and a plane, then write a two column proof i dont understand this
Answered by Stephen La Rocque.
Two coplanar lines 2007-08-21
From Robin:
I am going two show that the two lines are coplanar:

(x-5)/4=(y-7)/4=-(z+3)/5
(x-8)/7=y-4=(z-5)/3

I know I have to find a point that lie on both lines, but dont really get it.

Answered by Stephen La Rocque and Penny Nom.
How much would 3 yards of stone cost? 2007-08-21
From Sue:
If it costs $13.50 a ton of 3/4" stone, how much would 3 yards of stone cost?
Answered by Steve La Rocque.
A geometry problem 2007-08-20
From samhita:
ABC is a triangle. Let D be a point on side BC produced beyond B such that BD=BA. Let M be the mid-point of AC. The bisector of angle ABC meets DM at P. Prove that angle BAP=angle ACB.
Answered by Chris Fisher.
Point-slope form 2007-08-16
From jenny:
write an equation in the point-slope form and the intercept form slope=4,passing through (1,3) slope=8,passing through (4,-1) slope= -5,passing through (-4,-2) slope= -2,passing through(0,-3)
Answered by Leeanne Boehm.
The length of the third side of a triangle 2007-08-15
From Brooklyn:
What is the equation to find the length of the third side of a triangle if you have the length of A, B, and the angles(s)?
Answered by Stephen La Rocque.
Circle Geometry 2007-08-14
From Robin:
In a triangle ABC, angle A=75 and B=60. A circle circumscribes the triangle. The tangents of the at points A and B meet in a point D outside the circle. Show that ABD is an isosceles triangle with a right angle at D. Diagram included.
Answered by Stephen La Rocque.
A nonlinear system 2007-08-14
From Marsia:
Explain how the solution of a nonlinear system could be just a point, or two points. Sketching these may make it easier
Answered by Penny Nom.
The swaying of a building in the wind 2007-08-11
From San:
During a strong wind, a tall builing, such as the CN Tower, can sway back and forth as much as 100cm, with a period of 10 seconds. Please help me to determine the equation for this function, in the form y=asinkx
Answered by Stephen La Rocque.
Linear systems and inverses 2007-08-06
From Marsia:
explain how inverses are used to solve linear systems.
Answered by Penny Nom.
Logs and exponentials 2007-07-31
From Marsia:
Explain how the functions of exponents and logarithms relate to each other.
Answered by Penny Nom.
Equation of a perpendicular line 2007-07-28
From Marina:
Find the equation, is standard form, with all integers coefficients, of the line perpendicular to x+3y=6 and passing through (-3,5)
Answered by Stephen La Rocque.
Multiplcation of two negative numbers 2007-07-26
From Brett:
Someone asked a question about multiplication and division of two negative numbers yielding a positive result here: http://mathcentral.uregina.ca/qq/database/QQ.09.99/butler1.html I was not fully happy with the explanation b/c I want to give me daughter a real-world example and I can't seem to find one.

The following illustrates why multiplying negative numbers has become difficult to explain:

2 X 2 = 4

----(-4)---(-2)---0---2---4
In this example we start with 2 and then want 2 more of them. When we move across the number line from 2 to our answer, which is four, we have moved only 2 units to the right.

-2 X -2 = 4

----(-4)---(-2)---0---2---4
In this example we start with -2 and then want -2 more of them. When we move across the number line from -2 to our answer, which is four, we have moved 6 units to the right.

How can the phenomenon of multiplying two negative numbers being more powerful than multiplying two positive numbers be explained? -Brett

Answered by Stephen La Rocque and Harley Weston.
A cone with two fruits 2007-07-26
From Meg:
You have a cone shaped bag. At the bottom of the bag is an orange with radius r. On top of the orange is a melon with radius R. It touches the orange and fits snugly in the bag, touching it in a ring around the orange. Its top is at the same level as the top of the bag. What is the radius of the cone?
Answered by Stephen La Rocque.
A one to one function 2007-07-24
From Marsia:
Explain what it means for a function to be one to one.
Answered by Harley Weston.
Calculating the area (acreage) of a four sided lot 2007-07-18
From A property owner:
I have a real estate property and the lot size is something I need to find out. I know the lengths of the four sides, but it isn't a rectangle, it is an odd shape. How do I find the acreage?
Answered by Stephen La Rocque.
Trig functions for angles not between 0 and 90 degrees 2007-07-16
From Tim:
My question: Why is the value of a trigonometric function, the same, for an angle over 90 degrees and its reference angle? How are the angle and its reference related? Do they both form a triangle that has equal sides?
Answered by Penny Nom.
Proving a quadrilateral is a rectangle 2007-07-14
From Sonja:
I was having this discussion with another teacher and we need a third opinion. When you are trying to prove a quadrilateral is a rectangle which method should you use:
  1. Prove the shape is a parallelogram by doing slope 4 times by stating that parallel lines have equal slopes. Then proving a right angle by stating that perpendicular lines have negative reciprocal slopes.
  2. Doing the slope 4 times and stating that the shape is a rectangle because opposite sides are parallel because of equal slopes and it contains a right angle because of negative reciprocal slopes.
I guess the real question is do you have to first state that the shape is a parallelogram?

Answered by Stephen La Rocque.
Any regular polygon inscribed in a circle 2007-07-12
From DJ:
Circle with r=12" is inscribed in a regular octagon. What is the length of each octagon segment? Note: Our answer works for any regular polygon inscribed in any circle.
Answered by Stephen La Rocque.
Discount percentage 2007-07-11
From Greg:
List Price: $50.00 Discount: 33 1/3% What would the net price be?
Answered by Stephen La Rocque.
Calculating sales taxes 2007-07-11
From Tonya:
Hi, Im having trouble calculating GST and PST I know that in this province, GST is 6% and PST is 7%, I have a total of $275, and GST is 0.06, and PST is 0.07, so do I add those two and multiply them by the total, $275?
Answered by Stephen La Rocque.
Finding the radius of an inscribed circle 2007-07-05
From Maria:
I need to find the radius of a circle which is inscribed inside an obtuse triangle ABC. I know all the angles and all the lengths of the triangle.
Answered by Stephen La Rocque and Chris Fisher.
The lock code for my phone 2007-06-30
From Amanda:
okay so i forgot my lock code for my phone its four digits but i know it doesn't have a 1 or a 0 in it so it has to be from numbers 2 to 9 i need to know all the four digit combinations asap cause i cant use my phone till can someone please help me Amanda
Answered by Penny Nom.
If A is 1 and Z is 26 2007-06-29
From Sue:
If A is 1 and Z is 26 is there a word that when you multiply all the numbers instead of letters adds up to a million?
Answered by Chris Fisher and Harley Weston.
A linear system 2007-06-29
From Kim:
solve for x,y,z

x-y+z=4 2x+2y-z=-2 x-2y+2z=7

Answered by Harley Weston.
Splitting the profits unequally 2007-06-27
From Maci:
We recently hosted a fundraiser. Not everyone will receive a whole share of the profits because they did not participate the entire time. Several will receive only 1/2 or 1/3 share. How do I divide the profits?
Answered by Stephen La Rocque.
sin|x| and cos|x| 2007-06-25
From Mac:
Can anyone tell me whether sin|x| and cos|x| is differentiable at x=0 ? As far as i know, cos(x) and sin(x) is differentiable at all x.
Answered by Penny Nom and Stephen La Rocque.
Limits as x approaches a constant 2007-06-25
From Mac:
can you please tell me what is the reason they say "denominator is a negative quantity" in the solution 11 and "denominator is a positive quantity" solution 10 ??
If i guess correctly, for solution 10, its because of x^2 in the denominator.

Answered by Penny Nom.
The substitution method 2007-06-23
From mary:
solve the system by the method of substitution: -5x+2y=-15 4x-y=9
Answered by Stephen La Rocque.
Exponential form: x^y 2007-06-22
From Kishor:
whats the easy way to calculate X raised to Y where y is much greater than x.
Answered by Stephen La Rocque.
Rewrite 2x+4y+2=3y+5 in general form 2007-06-21
From dawn:
how do you rewrite 2x+4y+2=3y+5 in general form?
Answered by Stephen La Rocque.
Simultaneous equations : the Elimination method 2007-06-21
From Patricia:
I need to find the value of X and Y using the Elimination method.

5/x + 3/y=4
25/x-2/y=3

Answered by Stephen La Rocque.
Factor X^6 - 64 2007-06-20
From Krystal:
How do I Factor X^6 - 64 into linear factors?
Answered by Penny Nom.
Simultaneous inequalities 2007-06-18
From Freddy:
Watson Electric has production facilities in Valley Mills, Marlin,and Hillsboro. Each one produces radios,stereos,and TV sets. There production capacities are

Valley Mills: 10 radios, 12 stereos, and 6 TV sets per hour Marlin: 7 radios, 10 stereos, and 8 TV sets per hour Hillsboro: 5 Radios, 4 Stereos, amid 13 TV sets per hour

QUESTION

How many hours should each plant be scheduled to fill an order of 1095 radios, 1230 stereos, and 1490 TV sets?

Answered by Stephen La Rocque.
An inequality 2007-06-16
From jerry:
Two real numbers a and b satisfy the equation ab=1.
i) Prove that a^6 + 4b^6 >4
ii) Find out whether the inequality a^6 + 4b^6 >4 holds for all a and b such that ab = 1

Answered by Stephen La Rocque.
A system of linear equations 2007-06-14
From Alfredo:
A student club sponsored a jazz concert and charged $3 admission for students, $5 for faculty, and $8 for the general public. The total ticket sales amounted to $2542. Three times as many students bought tickets as faculty. The general public bought twice as many tickets as did the students. Set up the equations that determine how many tickets were sold to each group and solve.
Answered by Stephen La Rocque and Penny Nom.
Angles of depression 2007-06-13
From Phonda:
The pilot of a small private plane can look forward and see the control tower for a small airstrip. Beyond that is a large factory that is 3 milies from the airstrip. The angles of depression are 12.5 degrees and 4.8 degrees respectively. Find the airplane's altitude, to the nearest ten feet.
Answered by Stephen La Rocque.
How high does the ladder reach? 2007-06-11
From Madi:
A ladder 8 ft long resting on a house makes a 60 degree angle with the ground. how far up the house does it reach?
Answered by Stephen La Rocque.
The area of a quadrilateral 2007-06-10
From Lucy:
Calculate the area of the quadrilateral ABCD. AB= 4.1cm, BC = 7.6cm, AD= 5.4 cm, CD= ? Angle ABC = 117, Angle ADC = 62. Give your answer correct to 3 significant figures.
Answered by Stephen La Rocque and Penny Nom.
The law of sines 2007-06-09
From Felicia:
A parallelogram has one side that is 12.0 cm and one angle that is 65°. The shorter diagonal is 25.0 cm. To the nearest tenth of a centimetre, how long is the other side of the parallelogram? Use the sine law.
Answered by Penny Nom.
Two tangent lines to y=x^3 2007-06-07
From stephanie:
find the equations of two tangent lines to the y=x^3 function through the point (2,8)
Answered by Penny Nom.
What happens when you have zero's on both sides? 2007-06-05
From Lily:
On the substitution method what happens when you have zero's on both sides of the equation? Is that considered no solution or infinitely many?
Answered by Stephen La Rocque and Penny Nom.
Equation of a line 2007-06-03
From Mahrukh:
Hi i would like to know how to find the equation of a line, when you are given a piont (3, -1) and the x intercept which is -1?
Answered by Penny Nom.
Points in a plane 2007-06-01
From Gabrielle:
In general how many planes are there which contain two given points, three given points, and four given points?
Answered by Steve La Rocque and Walter Whiteley.
The intersection of two planes 2007-05-31
From Sarim:
How to find a intersection of two planes?
Answered by Penny Nom.
Volumes of cones and cylinders 2007-05-29
From George:
1. The volume of a cylinder is 1353cm3
A) What is the volume of a cone with the same radius as the cylinder but double the hieght of cylinder?
B) What is the volume of a cone with the same height as the cylinder but with three times its radius?

Answered by Steve La Rocque and Walter Whiteley.
y=mx+b 2007-05-29
From molly:
Im having a problem with the y=mx+b for our homework we have a table that goes from 0 to 6 , 1 to 7 and 2 to 8 and we have to make a equation of that. Please help.
Answered by Penny Nom.
The distance between parallel lines 2007-05-28
From Kristie:
I can't figure out how to determine the distance between parallel lines I am taking linear algebra and vector geometry. I was given two lines in the form #x+#y =# told they were parallel and asked to find the distance between them?? Please help Thanks
Answered by Penny Nom.
System of equations 2007-05-24
From Chris:
Find all real solutions (x,y,z,w) of the system of equations:
2y= x + x/17, 2z= y + y/17, 2w = z + z/17, 2x= w + w/17

Answered by Penny Nom.
Solve for I 2007-05-23
From Mark:
A=4.9(B)*C^(.00925(D-.542))* E^(-.015(D))*E^(-.013(F))* (G/B)^(-.00825(D-.225))* H^-.877*(I/B)^.511*(J/B)^.319
Answered by Stephen La Rocque.
Set up two simultaneous equations 2007-05-21
From Admire:
The cost of producing windscreen wipers blades at a factory ais partly fixed (due to operating overheads) and partly dependent on the number of blades produced. It costs $300 to produce 1000, and $600 to produce 5000 blades. How much would it cost to produce 24000 blades?
Answered by Penny Nom.
Constant rate of sand falling in a cone 2007-05-20
From Nhi:
Sand is falling into a conical pile . After 5 min. the pile has radius 24 and height 26 . After 7 min. tell how far the point c. is from the top of the cone (A).
Answered by Stephen La Rocque.
Lateral area of a right cone 2007-05-17
From Crystal:
In my homework the question says the lateral area of a right cone is 226.08 cm cubed. the slant hieght is 12 cm. Find the total surface area. How do I do that?
Answered by Stephen La Rocque.
Altitudes of scalene triangles 2007-05-17
From Chris:
Two of the altitudes of a scalene triangle ABC have length 4 and 12. If the length of the third altitude is also an integer, what is the biggest that it can be?
Answered by J. Chris Fisher.
Maximizing the volume of a cone given the slant length 2007-05-14
From Christina:
A coffee filter for a new coffee maker is to be designed using a conical filter. The filter is to be made from a circle of radius 10cm with a sector cut from it such that the volume of coffee held in the filter is maximised. Determine the dimensions of the filter such that the volume is maximised.
Answered by Stephen La Rocque and Kerstin Voigt.
Prisoners 2007-05-12
From Ryan:
Last monday, the county jail had 200 prisoners. Of these, 130 were accused of felonies, and 121 were accused of misdemeanors. How many prisoners were accused of both a felony and a misdemeanor?
Answered by Stephen La Rocque.
Exponents and geometry 2007-05-12
From Cynthia:
Hi is (x^6)(x^4)1/2 = 5 And How does 1/2(3.14)(4.1) =26.3917
Answered by Penny Nom and Stephen La Rocque.
Find the equation of a line passing through two given points 2007-05-12
From Kenzie:
I am having problems solving an equation of the line passing through (-2,4) and (6,0)
Answered by Penny Nom and Stephen La Rocque.
Finding the hypotenuse without Pythagorus 2007-05-11
From Shelbie:
How do i find the hypotenuse of a right traingle not using the pythagorean thereom if i have the measurements of the legs?
Answered by Stephen La Rocque.
Linear growth 2007-05-11
From Rebecca:
What is the definition of linear growth?
Answered by Penny Nom.
Understanding log equations 2007-05-10
From Sean:
The number, N, of people who have heard a rumor spread by mass media at time, t, in days is modelled by N (t) = a / 1+ be^ -kt. If 50 people have heard the rumour initially and 300,000 people hear the rumour eventually, find a and b. If the rumour is initially spreading at the rate of 500 people per day, find k.
Answered by Penny Nom.
Is the point on a cone called a vertex? 2007-05-09
From Felicia:
Does a cone have a vertice? My teacher says that a vertice can only be made if two or more edges join up at an angle, so what do you call a point on a cone?
Answered by Walter Whiteley.
The substitution method 2007-05-08
From A student:
I need help on these two problems( substitution method)

-2x+7y=10
x-3y= -3
and
6x-8y= -12
-3x+4y=9

Answered by Penny Nom.
Slicing a double-napped cone with a plane 2007-05-07
From Andrew:
I am writing a paper about creating parallel lines by slicing a double-napped cone with a plane. I have found out how it can be shown by algebra that the equations for parallel lines are generated from the degenerate case of a second degree polynomial in two variables, but I have yet to find a source with a visual representation of this case. Do you know if it exists?
Answered by Chris Fisher.
The radius of a cone 2007-05-07
From Braden:
i need to find the surface area but i only have the slant height and the height i need the radius how do i find it?
Answered by Penny Nom.
Road incline grades 2007-05-07
From cecil:
convert grade percent to degrees
Answered by Stephen La Rocque.
Evaluating sine and cosine 2007-05-06
From Selimovic:
How can i solve sine or cosine for angle of, lets say 10°....Maybe it's easy but i don't know how...
Answered by Penny Nom.
The vertices of a triangle 2007-05-02
From natakie:
Find the coordinate of the vertices of a triangle whose sides lie on the following three lines. 2x+5y-16=0, 4x-3y-6=0 and 3x+y+2=0
Answered by Penny Nom.
A tugboat's speed 2007-04-30
From Amanda:
a tugboat must travel 24 miles against a 4 mile per hour current on the Potomac River and return. At what constant speed must the tugboat travel to make the trip in 12 hours. Round answer to the nearest tenth mph.
Answered by Stephen La Rocque.
Two equations 2007-04-29
From lavon:
I'm having trouble figuring out a linear equation, can you help?

- 3x + 2y = 8
3x + 2y = 8

Answered by Penny Nom.
Finding out if two line equations are parallel or perpendicular 2007-04-28
From Vincent:
Can you determine if the two lines are parallel, perpendicular, or neither 1/2x+4y=3/5 y/1 + 10/5= 8x I think there perpendicular but how would you solve the equations
Answered by Stephen La Rocque.
A line parallel to a given line 2007-04-28
From vince:
write an equation of the line satisfying the given conditions Through (2,-3) parallel to 2x=3y+5 and graph both equations
Answered by Penny Nom.
Maximize the volume of a cone 2007-04-27
From ashley:
hello, I've been stumped for hours on this problem and can't quite figure it out. The question is: A tepee is a cone-shaped shelter with no bottom. Suppose you have 200 square feet of canvas (shaped however you like) to make a tepee. Use calculus to find the height and radius of such a tepee that encloses the biggest volume. Can you help??
Answered by Stephen La Rocque and Penny Nom.
A line parallel to a given line 2007-04-26
From vincent:
I forgot how to write the equation of the line satisfying the given conditions through (2,-3) parallel to 2x=3y+5

and how would you graph this

Answered by Penny Nom.
Two parallel lines 2007-04-25
From prasanna:
Let me know the Equation of a line which is parallel to some other line at a distance of Dmts.
Answered by Penny Nom.
A linear system 2007-04-25
From Tee:
2x -y =5
2x +=y =5

and also
3/7x + 5/9y =27 (fractions)

I have to find the addition method to both problems

Answered by Penny Nom.
What is the square root of 729 to the third power? 2007-04-25
From Tori:
What is the square root of 729 to the third power?
Answered by Penny Nom.
Linear programming 2007-04-24
From Sylvia:
What is graphing linear programming?
Answered by Penny Nom.
Physics/Temperature Change 2007-04-22
From William:
A 3.00 x 10^-3 kg lead bullet travels at a speed of 2.40 x 10^2 m/s and hits a wooden post. If half the heat energy generated remains with the bullet, what is the increase in temperature of the embedded bullet? (specific heat of lead = 1.28 x 10^2 J/kg)
Answered by Stephen La Rocque.
Two lines intersect at a right angle 2007-04-20
From ellen:
The line with the equation 3x + by = 6 intersects with the line 6y + ax = c at right angles at the point (4,6). Determine the values of a, b & c.
Answered by Stephen La Rocque.
Ax + By = C 2007-04-18
From Diana:
find an equation in general form Ax + By = C with the following properties passing through points (a,b) and (2a, 2b)
Answered by Penny Nom.
Exponential form 2007-04-16
From Cassia:
I was just wondering, how do you write the squared root of 7x(to the 5th) in exponential form? If you could help explain that I'd be grateful. Thanks
Answered by Brennan Yaremko.
Simultaneous equations 2007-04-16
From kyrie:
simultaneous equation 4x + 3y = 21 2x * y = 8
Answered by Penny Nom.
How do i form a paper cone 2007-04-14
From Sash:
How do i form a paper cone with the height of 25 cm, the slant height of 25.8 cm, and the radius of 6.2 cm?
Answered by Stephen La Rocque.
Intersection of a line and a circle 2007-04-12
From gaby:
The sum of two numbers is 9. The sum of the squares of the two numbers is 41. Find the numbers.
Answered by Steve La Rocque and Melanie Tyrer.
Principia Mathematica 2007-04-12
From victoria:
i need help making a poster on emilie du chatelet a great mathematician can you describe the newton principia because i know that she worked on it thanks, victoria
Answered by Stephen La Rocque and Penny Nom.
Solving Quadratic-Liner Equations algebraically 2007-04-12
From Hector:
Solving Quadratic-Linear Equations algebraically.
y=x^2-4x+3
y=x-1

Answered by Stephen La Rocque.
A subtraction problem with negative numbers 2007-04-12
From Bridget:
my daughter has to change a subtraction problem with negative numbers in it to an addition problem with negative numbers and then solve the problem such as: -4-(-2)=?
Answered by Penny Nom.
No solution to a linear system 2007-04-09
From Ryan:
When is it possible to get no solution when solving a linear system?
Answered by Leeanne Boehm, Steve La Rocque, Penny Nom and Melanie Tyrer.
A cabinet with an arched front 2007-04-09
From Joe:
I am building an arched front cabinet that is 71 inches wide, 12 inches deep at both ends and 16 inches deep at the center. To accurately build this cabinet I need to known the radius of the circle that would form that arch. Thanks, Joe
Answered by Penny Nom.
3 divided by 3 to it's fifth root 2007-04-06
From Annie:
How do I transform the equation 3 divided by 3 to it's fifth root to simple radical form (getting the radical out of the denominator)?
Answered by Penny Nom.
A "claw setting" for a gemstone 2007-03-30
From Stephanie:
I'm trying to make a cone out of a flat sheet of metal for a "claw setting" for a gemstone. The cone must be 8mm wide at the top and 11mm long tapering to a point. But because the prongs must be cut out of the top the cone should not start to taper for a length of 3mm from that top 8mm. The 3mm prong is then bent over the 8mm stone. That probably doesn't make enough sense. But I don't know how to explain it. If it helps a claw setting is the very common prong setting for engagement rings or earrings. Please help as soon as possible as this is a commissioned piece for someone and I'm running out of time. I don't remember any math really from high school so please make the instructions really easy to follow. Thank You!!
Answered by Penny Nom.
Slopes of perpendicular lines 2007-03-27
From Mykolyn:
How do you find the slope of a line that is perpendicular to the graph of: y= -1/2x+4?
Answered by Leeanne Boehm and Sara Ulmer.
Solve for x and y 2007-03-27
From anthony:
solve y=4x and x+y=5
Answered by Sara Ulmer.
The distance between two fire towers 2007-03-23
From tony:
Two fire towers are 30km apart, tower A is due west of tower B. A fire is spotted from the towers, and the bearing from A and B are N76degreesE and N56degreesW, respectively. Find the distance from the fire to the straight line connecting tower A to tower B.
Answered by Stephen La Rocque.
A linear system 2007-03-23
From Adam:
Solve the system by graphing:
4y>6x
-3x+2y<-6

Answered by Stephen La Rocque and Penny Nom.
Graphing a line 2007-03-23
From mitchell:
Graph each equation. y = 2x + 4
Answered by Jaymi Peterson and Haley Ess.
The law of cosines 2007-03-23
From chetna:
Q 1) In triangle LMN, l=7, m=5 , n=4. find ANGLE N. After applying the rule and substituting values i'm getting Cos n= 58/40. Is there something wrong. The answer at the back of the book is 34 degrees.
Answered by Penny Nom.
The velocity of an aircraft 2007-03-22
From William:
Hello, I don't understand how to work this problem. Thanks in advance for your help!

A jet moving at 500.0 km/h due east moves into a region where the wind is blowing at 120.0 km/h in a direction 30.0 degrees north of east. What is the new velocity and direction of the aircraft relative to the ground?

Answered by Penny Nom.
A one pass calculation of standard deviation 2007-03-19
From Murtaza:
Hi I searched the listings but couldn't find what i am looking for. I have a dataset and i am allowed only 1 pass through it. At the end of this pass i must have the mean and standard deviation. Calculating the mean is easy as also demonstrated in your listings. I think the same can be done for standard deviation as well. But the doubt that i have is this :- Will the calculated standard deviation be sequence dependent ? i.e. if i change the order in which the data items enter the system, will the std deviation also change ?? Thanks in advance Murtaza
Answered by Penny Nom.
A line that is perpendicular to another line 2007-03-16
From Jesse:
How do you write an equation of a line that is perpendicular to y+2x+3 and passes through (3,4); I don't understand the steps of how to do this????
Answered by Stephen La Rocque and Penny Nom.
Skewness 2007-03-16
From Lyndsey:
How do I calculate the skewness of a set of data?
Answered by Penny Nom.
Negative exponents 2007-03-14
From cookie:
(2.50x^-3) - (3.0x^-1) please show me how to do this. The answer in the book is 7.5 x ^-4
Answered by Penny Nom.
Cut a given sphere by a plane 2007-03-14
From Rory:
Cut a given sphere by a plane so that the volumes of the two segments formed shall be in a given ratio. Show that in modern notation, this leads to the cubic equation m(r+x)^2(2r-x)=n(r-x)^2(2r+x) where r is the radius of the sphere, x is the distance of the cutting plane from the centre of the sphere, and m/n < 1 is the given ratio. Can you help me show the solution to this question. Thank you.
Answered by Penny Nom.
exponential decay of Carbon-14 2007-03-12
From Christine:
How do I set up the exponential decay formula for Carbon 14 with an initial amount of 1500 grams.
Answered by Stephen La Rocque.
A truncated cone 2007-03-10
From Russell:
Hello, I have attempted to use two of your answers already given and had no real success. This young lady is making a cat food dispenser using a truncated cone. The top of the cone as a diameter of 5 inches with a height of 6 1/2 inches and diameter of 3 inches for the bottom.

Could you please map out a solution for the both of us to understand? Thank you so much for your time and for this wonderful service.

Russell

Answered by Penny Nom.
Statistics data 2007-03-08
From Parnini:
The following table shows the expenditure on different household items by a family. Draw a line graph for the following data:
Items : Food Rent Cloth Education Saving Others
Expenditure : 2000 2.500 1.500 1.200 4000 3.500

Answered by Stephen La Rocque.
The volume of a styrofoam cup 2007-03-08
From leanna:
find the volume of a styrofoam cup if the diameter of the top is three inches, the diameter of the base is 2 inches, and the height is 4 inches.
Answered by Penny Nom.
A metal shroud for a outdoor fireplace 2007-03-06
From Arnold:
I am making a metal shroud for a outdoor fireplace, it is basically a lampshade type pattern,like the bottom of a cone.The top has to be 6 inches to fit the 6 inch stovepipe,and the bottom will be a 24 inch circle. the sides will be 18 inches in length.With the cost of the sheet metal,I can only afford to cut this out once,can you help me with the pattern ?
Answered by Penny Nom.
Prove that the triangle ABC is isosceles 2007-03-05
From Lisa:
I am stuck on this problem... If AB = CD (parellel line), and
Answered by Stephen La Rocque and Penny Nom.
The net for a pentagonal pyramid 2007-03-05
From MaKayla:
How many bases does the net for a pentagonal pyramid have?
Answered by Penny Nom.
A long narrow cone 2007-03-04
From Amy:
I am an art student making a piece of jewelry out of a flat metal sheet. I'm trying to make a long narrow cone that's roughly 3 1/4" long and just slightly under 1" wide at the base. Since I'm no math wiz, I'm having a really hard time. Please help.
Answered by Penny Nom.
Simultaneous equations with fractions 2007-02-28
From Alyca:
Hello Math Central, I am a grade 10 student taking Academic math. Our unit right now is method of substitution and elimination. I'm stuck on this one question that I've been doing forever. Please help =)

*For this equation I have to do method of elimination, but it's so much harder with fractions...could some one please explain to me how to do it step by step?*
x y 2
-- - -- = - --
3 6 3

x y 1
-- - -- = 1---
12 4 2

Answered by Steve La Rocque and Ashley Mang.
The intersection of two lines 2007-02-28
From Tamara:
I was told to make a graph to help solve this question. I was given two equations. One was x-2y=3 and the other was 2x+y=-4. We have to change it to the equation of the line format. Then we graph the line and find the point where they meet. I've been trying to do that, but when i check the answer, It turns out wrong. It's meeting in the wrong spot. I need help please.
Answered by Penny Nom.
Parallel lines 2007-02-27
From Jennifer:
How to you decide whether the graphs of these two equations are parallel linse: y=3x+2, y=1/3x+4
Answered by Stephen La Rocque and Penny Nom.
The lateral area of a cone 2007-02-27
From Michael:
Ive seen the other question about surface areas. I still don't understand the lateral area. In my math book, it has (pi * r * L) as the equation, r = radius and L = height of the cone. This is 8th grade math. Thanks.
Answered by Penny Nom.
The radius of a cone 2007-02-26
From lee:
hi i have a cone to calculate, the height and radius are equal and the slant height is 0.5m

the total surface area is 15m sq i need to calculate the radius of the cone Note : this will lead to a quadratic equation could somebody have a look please Thanks Lee

Answered by Penny Nom.
The volume of a cone 2007-02-24
From SAFDAR:
How to derive the formula for volume of cone?
Answered by Penny Nom.
x^x = 2x 2007-02-22
From ramsay:
This has been bugging me for ages. How do find both real solutions to x^x = 2x? Obviously I have x=2, and there's another at about 0.35, but I can't work it out properly. Any help?
Answered by Penny Nom.
A common tangent line 2007-02-20
From chris:
I have this problem I have been working on for days and cannot figure it out. it states: Find the two points on the curve y=x^4 - 2x^2 - x that have a common tangent line.

I know you use the first derivative to find the tangent line so if it is a common tangent line should you find two of the exact same tangent line equations at two coordinate points?

Answered by Chris Fisher.
Surface Area of a cone 2007-02-19
From Cari:
I am doing a math project. I am very confused on how to find the surface area of a cone. I have looked at other equtions but i still don't understand. How do you find the surface area of a cone that has a 15cm length and a 3.5cm radius?
Answered by Penny Nom.
An augmented matrix 2007-02-13
From Mary:
I've been trying for quite some time now to figure this out. I have to solve this by using the Gauss-Jordan Method: 3x - y = 15 2x + 3y = 10 Can anyone help me?
Answered by Penny Nom and Gabriel Potter.
Exponential form of complex numbers 2007-02-12
From Austin:
When dealing with imaginary numbers in engineering, I am having trouble getting things into the exponential form. The equation is -1+i now I do know that re^(theta)i = r*cos(theta) + r*i*sin(theta). Just not quite understanding the order of operations. Thanks
Answered by Penny Nom.
What is the difference between -(3) and (-3)? 2007-02-11
From Mckaylla:
What is the difference between -(3) and (-3)?
Answered by Melanie Tyrer.
Volume of an inner tube 2007-02-10
From Bubba:
For a science project, I'm collecting methane gas in an inner tube. In addition to measuring psi of the inner tube, I'd like to calculate the volume of gas collected. What is the formula for or how would I calculate the volume of an inner tube? I appreciate any help you can give me.

Thanks so much. Bubba

Answered by Penny Nom.
Large exponents 2007-02-09
From Nick:
I am trying to figure out an extremely large number. It relates to the estimated number of bacterial divisions in 12775 generations of bacteria.
The problem I need to solve is:
2^12775 or 2 to the power of 12,775.

Answered by Stephen La Rocque and Penny Nom.
Simultaneous equations with envelopes 2007-02-08
From Mick:
There were 17 envelopes bought, some were brown, some were white. The brown envelopes cost one cent more per envelope than the white ones. The total cost was 80 cents. How much of each type of envelope was bought? --Many thanks!
Answered by Stephen La Rocque.
Find the next 3 terms of the sequence 2007-02-07
From rose:
find the next 3 terms of the sequence 2,3,9,23,48,87
Answered by Penny Nom.
Dividing money unequally 2007-02-06
From Linda:
I have a total of $13,826.48. It is to be divided by 15 kids - 2 of the kids only get half of what the other 13 do - how much do the 13 get and how much does the 2 get? Thanks
Answered by Haley Ess and Steve La Rocque.
Nickels, dimes and quarters 2007-02-05
From Avinash:
Mary has 48 coins made up of nickels, dimes and quarters with a total value of $5.10. She has 4 more dimes than nickels and quarters combined. How many coins of each kind does she have? Use matrix to solve the system
Answered by Stephen La Rocque.
Patrick's new job 2007-02-02
From kathy:
patrick has been offered jobs by two different automobile repair shops. his income from the first shop will be $1,200 per month plus 3% of the monthly sales. the second shop will pay patrick $1,500 per month plus 2% of the monthly sales. part a let s equal the amount of monthly sales and i equal patrick's income in dollars. write a system of two equations that could be used to determine the monthly income from either shop. part b solve the equation to determine what amount of monthly sales for each shop would result in patrick earning the same monthly income, i, from both shops
Answered by Penny Nom.
The volume of a cone 2007-01-31
From ajay:
WHY VOLUME OF CONE IS ONE THIRD OF THE VOLUME OF CYLINDER?
Answered by Penny Nom.
The substitution method 2007-01-31
From Victoria:
how do i solve this problem using the substitution method?
2x-5= -14
-7x+14y= -5

Answered by Stephen La Rocque.
The elimination method 2007-01-31
From Addrianna:
x-2y=2
3x-5y=7

Answered by Stephen La Rocque.
11^n +22^n = 55^n 2007-01-29
From Ankit:
11^n +22^n = 55^n

find the value of n?

Answered by Penny Nom.
A boat travels 60 miles downstream in the same time it takes to go 36 miles upstream 2007-01-27
From Liz:
A boat travels 60 miles downstream in the same time it takes to go 36 miles upstream. The speed of the boat in still water is 15 mi/h greater than the speed of the current. Find the speed of the current.
Answered by Stephen La Rocque.
A curve on a cylinder 2007-01-27
From John:
think of a tube,say 50mm in diameter made out of cardboard, project vertical lines at right angles from the base at say every 2mm right round the tube in pencil.No pretend you can put this cardboard tube in a saw and cut it at 45degrees. Get a pair of scissors and cut it at the lowest end and lay it out flat.It now looks like a graph,how do you work out each of these vertical lengths possibly chord lengths
Answered by Stephen La Rocque and Penny Nom.
The sum of the digits of a two-digit number is 13. 2007-01-27
From Liz:
The sum of the digits of a two-digit number is 13. If the digits are reversed, the new number is 9 more than the original number. Find the original number.
Answered by Penny Nom.
Take $.01 and double it every day for 30 days. 2007-01-26
From Pat:
Take $.01 and double it every day for 30 days. How many dollars would you have?
Answered by Stephen La Rocque.
An algebra exercise 2007-01-24
From Clara:
Given x + 2y = -2 and x - 2y = 18, find:
a) x^2 - 4y^2
b) x^2 + 4y^2

Answered by Stephen La Rocque and Penny Nom.
Is -2^2 = -4 or 4? 2007-01-23
From Joan:
Is -2 squared, when written without parentheses around the -2, -4 or could this correctly be solved by squaring -2 (-2 x -2) for an answer of 4? Or, to correctly get an answer of 4, would the problem have to read (-2) squared?
Answered by Stephen La Rocque.
Simplify (2x)^4 (1/2x^3)^2 2007-01-21
From Sarah:
Simplify (2x)^4 (1/2x^3)^2
Answered by Penny Nom.
Line segments on dot paper 2007-01-21
From Khaori:
The three line segments below are drawn on centimeter dot paper.

a. Find the length of each segment to the nearest ten-thousandth of a centimeter.
b. Could these line segments be arranged to form a triangle? If no, explain why or why not. If yes, answer this question: could they form a right triangle? Explain why or why not.

Answered by Penny Nom.
The area of a scalene triangle 2007-01-20
From Kalsi:
I have a scalene triangle in which i have to find the area.
Answered by Penny Nom.
mls and grams 2007-01-19
From JJ:
If I have a 2000litre pool and need to put 2mg of chlorine per litre, how many mls do I need to add?
Answered by Penny Nom.
Points of intersection 2007-01-18
From Mark:
Find the points of intersection of the given pair of curves. 2x - 3y = -8 and 3x - 5y = -13
Answered by Steve La Rocque and Haley Ess.
A line through the point (4, 7) and parallel to the line y=2 2007-01-14
From Chasity:
i am suppose to write an equation of a line through the point (4, 7) and parallel to the line y=2. please help me
Answered by Penny Nom.
A line perpendicular to another line 2007-01-13
From Dusty:
Find an equation for the line perpendicular to: 3x - 4y + 1 = 0 passing through (4,7)
Answered by Leeanne Boehm and Penny Nom.
3x > -12 2007-01-11
From Jen:
In the following inequality, would one flip the inequality symbol?

3x > -12

Answered by Stephen La Rocque.
Negative exponents 2007-01-11
From Bob:
I have to write this "Y-7" (but the -7 is an exponent) as a positive exponent. How?
Answered by Penny Nom.
Write the equation of a vertical line passing through (6,-2) 2007-01-09
From Alex:
Write the equation of a vertical line passing through (6,-2)
Answered by Penny Nom.
x + 2y < 6 2007-01-06
From Mike:
I understand solving by graphing, but i am having trouble with the graphing of in equalities. i solved : x + 2y<6
Answered by Penny Nom.
Phone number combinations 2007-01-04
From Efrain:
i want to know the different combinations for numbers 7 digits the number are 0-9 its basically the different phone number combinations
example: 555-6689 or 565-4896 i just want to know for many different combinations are there....

Answered by Penny Nom.
A linear system 2007-01-04
From Harold:
System A (4, 2/3)
2x-3y=6 and x-3y=2

I approached this problem using the addition method.

Could i have found the answer using the graphing method?

Answered by Penny Nom.
Two cones 2006-12-30
From Cassie:
A cone of radius 6 and height 12 and a different cone of radius 8 and height 12 intersect as shown in the figure below, where the vertex of one matches with the center of the base of the other. Find the volume of the intersection of the two cones (in exact form).
Answered by Penny Nom.
Four 9s 2006-12-15
From A student:
Can you make 67 with four 9's without using the ceilling method?
Answered by Stephen La Rocque.
Place value, coins and money 2006-12-12
From Emily:
I was wondering what the best way would be to teach place value using coins and money. I have used base ten blocks already and am looking for a way to integrate money, perhaps on a place value chart and perhaps using decimals.
Answered by Diane Hanson.
lim x-->infinity cos x 2006-12-07
From Katie:
I was wondering if it was possible to find: lim x-->infinity cos x
Answered by Stephen La Rocque.
The proof of inequality by mathematical induction 2006-12-07
From Carol:
S(n) = 2^n > 10n+7 and n>=10
Answered by Stephen La Rocque.
Linear inequalities 2006-12-01
From Susan:
how would I graph a linear inequality of y>3.
Answered by Stephen La Rocque.
2x > -6 and x - 4 < 3 2006-11-27
From Alyssa:
2x > -6 and x - 4 < 3 , i need to know how to solve it and how to graph it.
Answered by Stephen La Rocque.
cos2x=1 2006-11-21
From Christina:
I'm have a hard time solving cos2x=1 for exact values between 0<_x <360
Answered by Stephen La Rocque and Penny Nom.
Wheat is poured on a conical pile 2006-11-17
From Rachel:
wheat is poured through a chute at the rate of 10 cubic feet per minute and falls in a conical pile whose bottom radius is always half the altitude. how fast will the circumference of the base be increasing when the pile is 8 feet high?
Answered by Penny Nom.
An equation for a line 2006-11-16
From Jessica:
(-3,4) m=undefined write an equation for each line given the slope and a point on the line. show your work
Answered by Stephen La Rocque.
Motorcycle expansion chamber design 2006-11-14
From David:
I'm interested in calculating cone information regarding motorcycle expansion chamber design for example. I guess it's called a truncated cone, from what I've read so far. If I know the center line height, small radius, and large radius of a truncated cone then, how can I calculate the angle (included angle?) the cone forms? I'd like to know the variations of the formula so I can calculate for angle, or length, or one of the diameters if I know the other two measurements.
Answered by Stephen La Rocque.
How high is the satellite above the earth? 2006-11-13
From Naomi:
If the linear speed of a satellite in synchronous orbit is 1000 mi/h, how high is the satellite above the earth?
Answered by Stephen La Rocque.
After how many hours will the class have been notified? 2006-11-10
From Brittany:
A college math class consists of 32 students. On Monday at 9:00 a.m., the teacher tells one student to notify other students that the test scheduled for Wednesday at 9:00 a.m. has been cancelled. The model for the number of students in the class who will have heard this information after t hours is N=32-32e^-0.02t. After how many hours will the class have been notified?
Answered by Stephen La Rocque.
The slope-intercept form 2006-11-08
From Kilihea:
I need to graph this equation but first need to turn it into a slope-intercept form. 6x - 3y = -3 I would really appreciate it if you could help me turn it into a slope-intercept form.
Answered by Penny Nom.
Simultaneous equations 2006-11-06
From An other:
e^2y-x+2=0
ln(x+3)-2y-1=0

Answered by Penny Nom.
Newton meters and ft-lbs 2006-11-04
From Michael:
What is the correlation between Newton meters and ft-lbs of torque? If an engine has 100ft-lbs of torque,what would that equate to in Newton meters?
Answered by Stephen La Rocque.
The distance from a point to a line in 3 space 2006-10-29
From Marie:
find the distance from the point (1, -2, -3) to the line x = y = z-2
Answered by Penny Nom.
Zero as an exponent 2006-10-28
From Don:
Any real number (except zero) to the zero power equals 1 correct?
Question: If for example x to the zero power equals 1
What is -x to the zero power equal to?
What is -(x) to the zero power equal to?

Answered by Penny Nom.
Ratios, proportions and medicine 2006-10-25
From Steven:
I don't understand why we need to learn about ratios and proportions because i want to become a doctor and i don't know if i'll ever use this type of math.
Answered by Penny Nom, Claude Tardif and Walter Whiteley.
Putting things in exponent form 2006-10-18
From Sheila:
Write in exponential form 2 sq. root a with an exponent 5.
I do not understand on how to do it.

Answered by Stephen La Rocque.
How many feet tall is the tunnel 2006-10-14
From Charles:
A tunnel in the Smoky Mountains is semicircular. At a distance of 12 feet from the center of the tunnel, the tunnel has a height of 16 feet. How many feet tall is the tunnel at its center?
Answered by Penny Nom.
A specific 8 digit number in a 11 digit mobile phone number 2006-10-13
From James:
I need to know the chance of finding a specific 8 digit number in a 11 digit mobile phone number. the number must be 29571596 and the 11 digit phone number must end in this. All numbers 0-9 are used in a mobile number but the first 2 digits will always be 07. Is there a way to work this out??
Answered by Stephen La Rocque.
Solving a linear equation 2006-10-12
From Eriona:
6p-5=17. I have to solve the equation.
Answered by Penny Nom.
Plotting a linear equation 2006-10-09
From Jennifer:

I am having problems explaining how to do these problems to my child .

graph:2=x-y

graph the equation: 2y+x=-3


Answered by Penny Nom.
How many line segments are necessary? 2006-10-04
From Varun:
If you place 35 points on a piece of paper so that no three are collinear, how many line segments are necessary to connect each point to all the others?
Answered by Stephen La Rocque.
Inequalities 2006-10-03
From Maleeha:
a)Use algebra to find the set of values of 'x' for which x(x-5)>36.
b)Using your answer to part a, find the set of values 'y' for which y2(y2-5)>36

Answered by Penny Nom.
Family of lines 2006-09-29
From Sasha:
could a "Family of Lines" have both parallel and intersecting equations that aren't related directly but otherwise related by each other to create a curve
Answered by Chris Fisher.
2 5 11 17 23 31 ? 2006-09-28
From Bernice:
How do you determine the next number in the following sequence:
2 5 11 17 23 31 ?

Answered by Penny Nom.
Computing gravity 2006-09-27
From Patricia:
What is the gravity of planet z in inch/hr squared if the weight of an element with mass of 45000 oz on planet z is 2N?
Answered by Stephen La Rocque.
a _________________ line forms a 90 degree angle with another line 2006-09-24
From Lisa:
a _________________ line forms a 90 degree angle with another line is this to it what fills in the blank? first letter is E third letter is R and has 13 letters
Answered by Stephen La Rocque.
Linear systems 2006-09-21
From Alexander:
Hello, i have a math assignment due tonight at midnight and i need some help please

It is using the addition method to solve linear systems
Answered by Penny.

Constructing a cone 2006-09-20
From Suresh:
i want to know the required size of plate for cone rolling,sizes are 2950mm is bottom dia,894 is top dia and 600 is height.I have already read u r answers but i little bit confused ,harely and sue have given useful answers but when i worked both method the required plate size is different. so i like to know which method is easy and correct.and also i like know whether it can be rolled without segment my rolling machine width is 1500.
Answered by Penny Nom.
Is this unsolvable? 2006-09-19
From Al:
Is this unsolvable???

2xy + 3x^2 - 3y + y^3 = 4x + 3 where y=3.
Answered by Penny Nom and Claude Tardif.

Convert a standard form to a slope intercept form 2006-09-14
From Maiko:
I need to know step by step procedures to convert a standard form to a slope intercept form?
Answered by Penny Nom.
Plotting co-ordinates on a number plane 2006-09-08
From Victor:
I need to plot co-ordinates (0,2), (2,-2), (-2,0), and (0,0) on a number or co-ordinates plane, I am not sure how to do this, and get lost after the first point, can you please help?
Answered by Penny Nom.
The radius of a cone 2006-09-08
From Hermanson:
I know the cone is 20 degrees at the top and 80 degrees at the bottom. What is the formula for finding the radius?
Answered by Stephen La Rocque.
Rays 2006-08-31
From Natasha and daughter:
My daughter, Natasha, is in 4th grade. She had the following homework question about rays that confused us: Name as many rays as you can in the figure below.
Answered by Penny Nom.
Grossing up payroll wages 2006-08-27
From Katie:
I really need to understand (spelled out step by step) how to gross up payroll wages.
Answered by Penny Nom.
A base number raised to the power of 0 2006-08-24
From Sheila:
I'm a 5th grade teacher and need some help with a base number raised to the power of 0 always being 1. My kids asked me why and I told them I'd find out.
Answered by Penny Nom.
An angle in a parallelogram 2006-08-13
From Sam:
Parallelogram ABCD has diagonal AC equal in length to side AB. CD is produced to E so that D is between E and C. If angle BAC =30 degrees find the size of angle ADE.
Answered by Stephen La Rocque.
How fast is the water level rising 2006-08-12
From Erin:
Water runs into a conical tank at the rate of 9ft3/min. The tank stands point down and has a height of 10 ft. and a base radius of 5 ft. How fast is the water level rising when the water is 6 ft. deep? (V=1/3 pi r2 h).
Answered by Penny Nom.
A problem with exponents 2006-08-09
From A student:
(8a to the negative 2 b cube c to the negative 4/4a squared b to the negative 3 c squared) to the negative 2
Answered by Stephen La Rocque.
I want to calibrate a glucose meter 2006-07-27
From John:
If I have 3 glucose control solutions having:
Low - 50 mg/dl
Normal - 100 mg/dl
High - 330 mg/dl

I want to calibrate a glucose meter but I need more than these 3 data points. How can I mix these up to get different concentrations in between 50 and 100, 100 and 330. I would like to mix up different concentrations and put them in test tubes or whatever to do some calibration work. Like 70 mg/dl, 90 mg/dl, etc. More data points should be in between say 50 and 100 since this is normal, 90 mg/dl is considered close to normal.


Answered by Stephen La Rocque.
Quadratics and quintics 2006-07-18
From Lynne:
What is wrong with this statement? "The quintic is one step up from the dread quadratic equation." (Newsweek, Dec. 19, 2005)
Answered by Stephen La Rocque.
A problem with arc sine 2006-07-07
From Scott:
How to prove arc sin x = arc tan( (x)/√(1-x2))
Answered by Penny Nom.
Flying with and against the wind 2006-07-06
From Tim:
When a plane flies with the wind, it travels 800 miles in 4 hours. Against the wind it takes 5 hours to cover the same distance. Find the rate of the plane in still air and the rate of the wind.
Answered by Stephen La Rocque.
How old is y = mx + b? 2006-07-05
From Beckie:
I am taking intermediate algebra this summer (Math 95), and have a question for you. My instructor will award extra credit points to any of us who find out how old the formula y = mx+b is. Would you happen to know the answer?
Answered by Penny Nom.
A trig problem 2006-06-24
From Greg:
A and B are two towers, B being 4 km due east of A. The true bearings of a flagpole, C, from A and B are α east of north and α west of north respectively. The true bearings of a second flagpole, D, from A and B are (α + β) east of north and (α - β) west of north respectively. Assuming A, B, C, and D are on level ground, and that α = 25, β = 10, find the distance between C and D.
Answered by Penny Nom.
The area of a sector and a triangle 2006-06-23
From Howard:
I thought of the following problem which is similar but much simpler than the tethered goat problem: What is the angle(it is more illustrative in degrees)of arc of a unit circle so that the area between the chord it subtends and the arc length is equal to the area of the triangle with opposite side the subtended chord.
Answered by Stephen La Rocque and Penny Nom.
2 to the -5th power 2006-06-19
From Murray:
Is 2 to the -5th power = -32?
Answered by Penny Nom.
13 57 91 11 31 51 ?? 2006-06-15
From Chastity:
im trying to determine the missing number in the sequence 13 57 91 11 31 51 ??
Answered by Claude Tardif, Steve La Rocque and Natasha Glydon.
The chord length of a polygon 2006-06-14
From Krishna:
I have to find out the chord length of a polygon - Tetradecagon ! The Radius of the Circle is 11.5 Cms. The Circle is intersepted by 14 arcs. Then how to find out the chord length?
Answered by Stephen La Rocque.
Square feet and linear feet 2006-06-10
From Richard:
Need to know how many linear ft are in 2800 sq.ft
Answered by Penny Nom.
Pyramids and cones 2006-06-06
From Melissa:
I was wondering if a cone can be considered a pyramid. Looking at many definitions of pyramids I have read that pyramids come to a common vertex. A cone comes to a vertex. But I also read that pyramids all have triangular faces. In this case a cone would not be considered a pyramid. Am I correct?
Answered by Chris Fisher.
Three people go to a store where there is a sale 2006-05-30
From Geocee:
Three people go to a store where there is a sale. One person buys two shirts, one pair of pants and one sweater. The second person buys one shirt, two pairs of pants and two sweaters. The third person buys three shirts and four pairs of pants. Find the price of each item if the first person spent $155 and the second and third person spent $235 each.
Answered by Stephen La Rocque.
A truncated cone 2006-05-28
From Phil:
Hi, I am an art student and I am trying to make a "truncated cone" (ie: a cone with the top cut off) out of sheet metal. I need to design a template first and am having problems working out the angles. The truncated cone is 250mm high, 550mm wide at the bottom and 290mm wide at the top. Can you help?
Answered by Stephen La Rocque.
Simultaneous Equations 2006-05-24
From Angie:

Question: solve the equations
2x-3y-z=0
3x-2y+z=-5
x+3y-2z=14

for x,y,z


Answered by Stephen La Rocque.
How do you graph 3x + 6y = -9 by axis intercepts? 2006-05-22
From Crystal:
How do you graph 3x + 6y = -9 by axis intercepts?
Answered by Penny Nom.
The interior angles of a right triangle 2006-05-20
From Greg:
I am wondering if there is a way to figure out the interior angles of a right triangle if we know ONLY the side lengths, and the trick is, we CANNOT use arctangent!
Answered by Leeanne Boehm and Penny Nom.
I want to calculate 0.00353 to the power 1.3 2006-05-14
From Richard:
I want to calculate 0.00353 to the power 1.3 How do I do it?
Answered by Stephen La Rocque.
Simplifying with negative exponents 2006-05-02
From Kristine:

I am having a hard time understanding factoring and multiplying polynomials. I have a couple of problems I just can't get, can you please help me?

Simplify, do not use negatives in the answer:
(5a-1b-7)(-2a4b2)

and:
(5x2y-7z)(-4xy-3z-4)


Answered by Stephen La Rocque.
Pythagorus and cone dimensions 2006-04-26
From Glynnis:
How do you find the measure of a side that is not the hypotenuse using the Pythagorean Theorem? Also, how do you figure the surface area and volume of a cone when the radius is 5 yards and the height is 8 yards?
Answered by Stephen La Rocque.
Marine shipping - fuel consumption 2006-04-26
From Sandy:
When a ship is 800 nautical miles from port its speed is reduced by 20%, thereby reducing the daily fuel consumption by 42 tonnes and arriving in port with 50 tonnes onboard. If the fuel consumption per hour is given by the expression (0.136 + 0.001V^3) where V is speed in knots; Estimate a) reduced consumption per day b) fuel onboard when speed reduced c) normal consumption for 800 nautical mile voyage d) increase in steaming time
Answered by Stephen La Rocque.
Absolute Value vs. Standard Deviation ? 2006-04-24
From Sonia:
Why we don't just take the average of the absolute value of difference scores (use the mean deviation) to describe variability instead of calculating the standard deviation?
Answered by Claude Tardif.
What is the surface area of a cone with a hemisphere on top? 2006-04-20
From Jordan:
How do you find the surface area of a cone with a hemisphere on top? The only information I have is the radius of the hemisphere, which is 4, and the total height, which is 7.
Answered by Stephen La Rocque.
Geometric sequence and basic functions (graphs) 2006-04-20
From Marlene:
Which of the basic functions is related to the geometric sequence: Linear, Quadratic, Rational, or Exponential? Can you give me an example of how it would be used in normal life?
Answered by Stephen La Rocque.
Equation of a line in Ax + By = C form 2006-04-19
From Christopher:
How do you write the equation of a line with a slope of 3/8 and passing through (-4,6) in Ax+By=C form?
Answered by Stephen La Rocque.
Cubic yards of landscape stone 2006-04-19
From Janet:
I need to spread landscape stone around my pool. A landscaper said I need 16 yards. What is the area of a cubic yard if I want the stone 3" deep?
Answered by Stephen La Rocque.
Cone dimensions 2006-04-12
From Mirnela:
The height of a cone is 40 and the radius is 9. How do I find the slant height of this? Also, how do I find the area of the base of a cone if the radius is 7?
Answered by Stephen La Rocque.
Finding the inverse of a function 2006-04-12
From Sam:
If g(x) = 2x+3/5 then how do I find the inverse?
Answered by Walter Whiteley.
Given three angles and a side 2006-04-09
From Jon:
How do you figure out the length of all sides of a scalene triangle if given the measure of all angles, and one side?
Answered by Stephen La Rocque.
Equations of two lines 2006-04-05
From Wael:
Determine a cartesian equation of the line passing through A and V as a direction vector?

1- A (2;1) and V = j

2- A (2;3) and V = i

Answered by Penny Nom.
A 20 foot container 2006-04-05
From Karen:
can you tell me how many cubic square meters are in a 20 foot container
Answered by Stephen La Rocque.
x-7=20 and 2x+5=17 2006-04-04
From Janette:
I'm trying to help out my daughter, it has been much too long since I've been in school. The question is x-7=20 and the other question is 2x+5=17. What are the formulas?
Answered by Penny Nom.
2x+5y=3 And -x+3y=-7 2006-04-03
From Lloyd:
simplify 2x+5y=3 And -x+3y=-7
Answered by Penny Nom.
An exponential function 2006-04-03
From Meadow:
The graph of an exponential function passes through the points (0,1) and (4,10) find the constant of growth or decay. State the function.
Answered by Stephen La Rocque.
Exponential decay 2006-03-29
From Casey:
Fifty milligrams of a drug was injected into a patient at 6am. The drug is known to be eliminated according to the law of exponential decay. At 8.30am it was determined that 60% of the drug remained in the body. How many milligrams will remain in the body at midday?
Answered by Penny Nom.
Angular velocity 2006-03-28
From Ben:
How do you (a) express angular speed in terms of linear speed and (b) express linear speed in terms of angular speed?
Answered by Stephen La Rocque.
A line parallel to a plane 2006-03-28
From Ryan:

I was wondering how I would prove the following theorem. I am completely lost at this point and all my other ideas have been extremely complicated and somewhat blurry.

"Through a point outside a given plane, there is at least one line parallel to the given plane."


Answered by Chris Fisher.
Solve the equation cos x = sin 20 where x is acute. 2006-03-26
From Elle:
Solve the equation cos x = sin 20 where x is acute.
Answered by Stephen La Rocque.
A number puzzle 2006-03-22
From A teacher:
Students brought this website to my attention and asked why this puzzle worked... I'm not sure. The url is: http://digicc.com/fido/ and it tells you to choose a 3 or 4 digit random number with different digits. Write it down, rearrange, subtract the smaller from the larger. then circle a nonzero digit, type the remaining digits into the space provided and they will tell you the number you circled. Can you provide the reason that this works.
Answered by Claude Tardif and Penny Nom.
The volume of water in a cone 2006-03-21
From Ghulam:
A vessel has the shape of an inverted cone.The radius of the top is 8 cm and the height is 20 cm. Water is poured in to a height of x cm.Show that if the volume of the water is V cubic cm,then V=(4/75)pi x3.
Answered by Penny Nom.
How far from Earth is Neptune? 2006-03-21
From Scott:
A laser beam travelling at the speed of light bounces off the planet Neptune and returns to Earth in 29 960.72 s. The speed of light is 299 792.5 km/s. How far from Earth is Neptune?
Answered by Penny.
Three towns are located at the vertices of an equilateral triangle 2006-03-20
From A student:
three towns are located at the vertices of an equilateral triangle. The towns are 8, 5, and 3 miles, respectively, from a store. How far apart are the towns?
Answered by Chris Fisher.
Linear feet on a paper roll 2006-03-20
From Vishal:
I WOULD WISH TO KNOW OF A STANDARD FORMULA BY WHICH I CAN CALCULATE THE LINEAR FEET OF A PAPER ROLL AS PER THE FOLLOWING DETAILS:-

THICKNESS: 150 GSM (GRAMS PER SQUARE METER)
REEL WIDTH: 85 CM
REEL DIAMETER: 140 CM
REEL INNER CORE: 10 CM
REEL WEIGHT: 946 KG's


Answered by Stephen La Rocque.
Back to the nines 2006-03-15
From Victoria:

Can you answer this problem, does an answer exist?

  • Get a set of numbers 1-9 !
  • Using the whole set of nine number tiles (digits 1-9), try to arrange them to make three 3-digit numbers so that the sum of the first two is the third.
Can this be done without carrying over? If not can it be done without carrying over into the hundreds column?

Answered by Claude Tardif.
2x+3y=0 and 3x-y=0 2006-03-14
From Lisa:
my name is lisa I am doing math for work and i have a math problem to solve and i need help with it here is the question 2x+3y=0 and 3x-y=0 this is one question can you help me please
Answered by Penny Nom.
8,_,4,9,1,_,10,3,_,0 2006-03-08
From Rachel:
I cant figure out this sequence if someone could help me 8,_,4,9,1,_,10,3,_,0
Answered by Claude Tardif.
2,4,9,6,5,6,____,____,____,... 2006-03-06
From Mike:
I am having problems figuring out the following sequence:

2,4,9,6,5,6,____,____,____,...

We were able to guess that the pattern simply started to reverse itself, but I was wondering if there were other possibilities.


Answered by Claude Tardif.
The nth term 2006-03-05
From Umar:
my question is what is the nth term for a house of cards 4 stories high if you use the following numbers:
2 7 15 26

Answered by Penny Nom.
A cone with an oval as base 2006-03-01
From Richard:
I am trying to find the volume of a cone that is not round but oval.
Answered by Penn Nom.
The path of a submarine 2006-02-26
From Meadow:
Suppose that a submarine has been ordered to follow a path that keeps it equidistant from a circular island of radius r and a straight line shoreline that is 2 units from the edge of the island. Derive an equation of the submarine path, assuming that the shoreline has equation x = -p and that the center of the island is on the x-axis.
Answered by Penny Nom.
The graph of y = 2x + 1 2006-02-23
From Geneva:
I'm having trouble with graphing equations like y=2x+1 which is my homework I'm not sure how to graph it. I'm in the 8th grade taking a 9th grade course algebra I.
Answered by Penny Nom.
The nth term of a sequence 2006-02-21
From Mike:
I'm having trouble finding the nth term in an equation, can you help me?
Answered by Stephen La Rocque.
Broken-line graphs and histograms 2006-02-16
From George:

1. What is the main difference between a broken-line graph and a histogram? Both represent continuous variables.

2. What is the correct way to read a multiplication array: x-axis first and then y-axis, other way around or it doesn't matter?


Answered by Penny Nom.
A lampshade shape 2006-02-11
From Rose:
Is there a solid shaped like a lampshade? Or do you just call it the bottom portion of a cone?
Answered by Penny Nom.
sinh(i/2) 2006-02-09
From Louis:
How can you set up an equation to find sinh(i/2)
Answered by Penny Nom.
preuve par neuf 2006-02-08
From Thonet:

Je suis enseignante en primaire en Belgique et j'aii actuellement une stagiaire étudiante qui va devoir apprendre à mes élèves comment fonctionne la preuve par 9. Elle sait expliciter concrètement celle-ci et a trouvé l'explication du pourquoi de sa fonction mais nous ne savons pas l'origine de cette application mathématique...

Qui l'a inventée ? Pourquoi ? ...
Pourriez-vous nous éclairer ?


Answered by Claude Tardif.
Find the ground speed and the planes true heading. 2006-02-05
From Jimmie:
An aircraft going from city A to city B on a bearing of S69E (degrees) is traveling at a speed of 430 mph. The wind is blowing out of the north to south at a speed of 25 mph. Find the ground speed and the planes true heading.
Answered by Penny Nom.
How do you find the angles in a triangle? 2006-01-27
From Keith:
How do you find the angles in a triangle if you know the lengths of the sides?
Answered by Chris Fisher and Penny Nom.
Two parallel lines 2006-01-22
From Marbara:
On my math homework I have a problem where it says find the value of x so that L is parallel to M

on one of the there are three lines. the top one is horizontal which is L. the bottom one is also horizontal directly under L which is M. there is another line that is diagonal going through the middle sections of L and M.

To the right of the diagonal line and right under line L is an equation 5x-10

to the right of the diagonal line and right above line M is an equation 8X-5

I tried to find what X is but I can't because it never matches both equations no matter what way I try to do it.

Answered by Penny Nom.
One boundary of a pond is parabolic in shape. 2006-01-20
From Glenn:
One boundary of a pond is parabolic in shape. The boundary passes through the points A(-20,45), B(40,40) and E(30,35). The equation of the parabola is of the form y=ax2+bx+c. Find the equation of the parabola and the coordinates of the vertex of the parabola. Any assistance you could provide would be greatly appreciated.
Answered by Penny Nom.
Intercepts and slope 2006-01-15
From Anthony:
Given -1.7x + 0.8y=7.1:

(a) state the intercepts and slope for the graph

(b) Draw the graph

Answered by Penny Nom.
Who is lying? 2006-01-14
From Diane:
A business man was working in his home office when he realized he had left a five-dollar bill in the book he had been reading. He called his butler to bring him the book from the library. When he got the book, the bill was no longer there. He then questioned the maid and the butler. The maid remembered seeing the bill between pages 99 and 100 in a book to the left of a business book. The butler did not recall seeing the bill, but was sure the book was to the right of the business book, because to the left of it there was a statistics book. Who is lying?
Answered by Penny Nom.
y - y1 = m(x - x1) 2006-01-08
From Greg:
How do you change 2x + 3y = 7 into y - y1 = m(x-x1)?
Answered by Penny Nom.
Extraneous solutions 2006-01-01
From Liz:

Question: solve and check for extraneous solution

3(w + 1)1/2 = 6


Answered by Penny Nom.
Simultaneous Equations 2005-12-21
From Matt:
I have these two equations,
336 = 60a + 10b
and
432 = 84a + 6b
Am I right in saying both a and b are 4.8?

Answered by Penny Nom.
-3^4 and (-3)^4 2005-12-17
From Brad:
What is the answer to

-34 = ?
-(3)4 = ?

(-3)4 = ?

Answered by Penny Nom.
5(x-3)/4 =x+1 2005-12-14
From Jennifer:

I have two problems i am stumped on I don't know how to help my daughter.

5(x-3)/4 =x+1
3(x+12)/5 =x +2


Answered by Penny Nom.
2/n +4 = 10- 4/3n 2005-12-11
From Jennifer:

2/n +4 = 10- 4/3n

5(x-3/4) = x+1


Answered by Penny Nom.
A sheet metal cone 2005-12-05
From Laura:
I am an art student and in the process of making a cone out of sheet metal. I am unable to work out the template I need to produce my final cone. The dimensions I have are that the final cone will be 58mm high and will have a diameter of 102mm.
Answered by Penny Nom.
A long distance call 2005-12-05
From A student:
The Small World long distance phone company charges 55cents for the first minute of a long distance call and 23cents for each additional minute.

1. How much would a 10- minute long-distance call cost?

2. If a call costs $4.55. how long did the call last?

3. Write an equation for the total cost, C, of an m-minute long-distance call.

Answered by Penny Nom.
How do you simplify a fraction if one of the numbers is negative? 2005-12-05
From Stephanie:

Question: How do you simplify a fraction if one of the numbers is negative?

Ex. -32/40


Answered by Penny Nom.
Solve for x 2005-12-04
From Lisa:
5(x-4)=3(2-3x)
Answered by Penny Nom.
A pattern for a truncated cone 2005-12-04
From Nick:
I need to make a large cone segment. The large end has ID of 57 inches and the small end has ID of 23 inches. The cone is essentially a 45 degree cone (90 degrees at the tip). The sides of the segment are 2 feet long. How do I lay out a flat pattern that will fold into this segment? I need to know radius 1 and radius 2 and the angle the piece must be.
Answered by Penny Nom.
A truncated cone 2005-12-03
From Sonny:
I need to create a template for a cone that has a 2" opening at the top, a 4" base and stands 6" tall. Can you tell me how to achieve this?
Answered by Penny Nom.
An integer equation 2005-12-01
From Henry:
3x = 2y + 7

I know the answer: x=9 and y=10, but what are the algebraic steps to solve the equation?

Answered by Penny Nom.
Express t as a function of d 2005-11-30
From Matthew:

I have a problem on my homework I cant seem to figure out. Basically it gives me this Linear Regression Equation:

ln t=1.5 ln d-0.8954

and it wants be to transform the equation expressing t as a function of d. For some reason I keep getting it wrong. Anyone got an idea of how to do this?


Answered by Penny Nom.
Graphing two lines 2005-11-20
From A student:
About y = 5x + 24 and y = -3x - 8
1. Graph both equations on the same set of axes. When you made your graph, how did you choose the range of x and y values?


2.Find the point of intersection for the graphs.


3. Test the point of intersection you found by substituting its coordinates into the equations. Do the coordinates fit the equations exactly?

Answered by Penny Nom.
Solving a pair of equations 2005-11-10
From Steven:
what is answer to
2 x + y = 1
y = -1/3 x - 4

Answered by Penny Nom.
-120 divided by -15 2005-11-09
From Joshua:
how do you evaluate -120 divided by (-15)
Answered by Penny Nom.
How many passengers boarded the plane in Phoenix? 2005-11-03
From Vish:
Passengers boarded a plane in Phoenix. In Dallas 1/2 of the passengers got off and 4/5 of the original number got on. In Memphis 3/4 of the passengers got off and twice the number of remaining passengers got on. In Chicago 2/3 of the passengers got off the plane leaving 39 passengers. How many passengers originally boarded the plane in Phoenix?
Answered by Penny Nom.
2, 6, 12, 20, 30, ... 2005-11-01
From Rebecca:
I have a sequence of I have a sequence of 2, 6, 12, 20, 30, and have no idea how to figure out the nth term please help!
Answered by Claude Tardif.
Volumes 2005-10-31
From Diane:
My name is Diane and I am a returning student to a vocational technical school.

As a reference point to see what I did/didn't remember from my HS math days, I was sent home with a 75 problem math packet. I was fine until I got to computing:


1. the volume of a cylinder-is it pi r2 h?

2. the volume of a cone- is it pi/3 r2 h?

3. the volume of a sphere- I can't even hazard a guess.

Help- my last classroom was 23 years ago, and I've forgotten far too much!

Answered by Penny Nom.
(-2^6) = ? 2005-10-29
From Amy:
My teacher gave me the problem (-26) on our homework. She marked it wrong when I wrote down -64 for an answer. She said the correct answer was 64. Can you explain why it is 64?
Answered by Penny Nom and Leeanne Boehm.
A slump cone 2005-10-27
From Wendy:
we are trying to make a slump cone (used to measure the slump in concrete). It has dimensions of 8" diameter on the bottom, 4" diameter hole on the top and a height of 12". Please help, it is getting frustrating.
Answered by Penny Nom.
A conical hat 2005-10-22
From Manish:
I need to make a conical hat for my daughter's upcoming fancy dress, the circumference of the base(hollow) is 50 cms,the height of the cone is 30 cms,what should be the dimensions of the paper which will make a cone of the beforementioned dimensions?
Answered by Penny Nom.
sin(kx) = x 2005-10-20
From David:
What is an integer value for k so that sin(kx) = x has exactly 2005 solutions? How does one arrive at the answer?
Answered by Harley Weston.
What is the angle between 2 non-collinear parallel lines? 2005-10-17
From Ben:
What is the angle (if any) between 2 non-collinear parallel lines?
Answered by Chris Fisher and Walter Whiteley.
A random sample 2005-10-16
From Stu:
If I have a set of data points (14 to be exact)of unknown pedigree from a large population, what tests can I apply to see if they constitute a random sample from the large population?
Answered by Andrei Volodin and Penny Nom.
2x - 5x + 6.3 = -14.4 2005-10-16
From Leslie:
How do we solve 2x - 5x + 6.3 = -14.4
Answered by Penny Nom.
Simultaneous equations 2005-10-13
From Daniel:
5x + 3y = 22 4x - 7y = -20
Answered by Penny Nom.
Open dots and closed dots 2005-09-29
From Cynthia:
When graphing the solutions of an inequality, what is the difference between an open dot and a closed dot?
Answered by Penny.
A cone with vertex (1,1,2) 2005-09-26
From Brandon:
Find the equation of a double cone with vertex (1,1,2) and which intersects the xy plane in a circle of radius 4.
Answered by Penny Nom.
Find the next 5 terms 2005-09-24
From Grant:
n+7, 2n-0.5, 4n. Find the next 5 terms of each one and explain the sequence in words
Answered by Penny Nom.
A piecewise function 2005-09-15
From Duncan:
A child is assigned to your care and she has a headache. The parent has authorized the administration of Children's Tylenol. Read a box of Children's Tylenol (or any other similar drug) and find the directions for administration. Note the child's weight and the corresponding dosage. (Be sure the drug you choose has a minimum of 4 weight intervals.)
Answered by Penny Nom.
Finding the nth term 2005-09-12
From Paul:

Hi , Im having a problem helping my daughter to find the Nth Term in the following sequences.

1/3 1/2 3/5 2/3

and

1 3 6 10


Answered by Penny Nom.
Adding positive and negative numbers 2005-09-03
From Billy:
46 + -24 =
-38 + -22 =
19 - 20 =

Answered by Penny Nom.
12-42, 8-26, 10-34, 9-30, 16-58 2005-08-31
From Elizabeth:
here is a math question my niece gave me, grade 7

12-42, 8-26, 10-34, 9-30, 16-58

first day of school and this is what they give her no explanation

Answered by Harley Weston.
1,4,9,1,6,2,5,3,6,4,9,6,4,8,1 2005-08-30
From Liz:
Find the next four numbers to the sequence 1,4,9,1,6,2,5,3,6,4,9,6,4,8,1,___,___,___,___.
Answered by Penny Nom.
Graphing a linear inequality 2005-08-26
From Gina:
When graphing a linear inequality, how do you know if the inequality represents the area above the line?
Answered by Penny Nom.
Labour efficiency 2005-08-23
From Rob:

The problem, on the surface, seems very simple and yet has created some controversy among a group of accountants. The problem itself has to do with labour efficiency rates and only involves two variables; standard working hours, and actual working hours. The difficulty lies in deriving an efficiency % from these two numbers.

Standard working hours or the targeted number of labour hours required to produce one widget, which I will represent as "s". Actual working hour or the actual number of labour hours require to produce one widget, which I will represent as "a". Labour efficiency I will represent with "E". The prevailing calculation with which I have a problem with is this:

s/a=E or if s=3000, and a=4000 then 3000/4000=75%

What bothers me about the calculation is that the standard hours get represented as a percentage of the actual hours and in my opinion changes the focus of the calculation from standard or target, where it should be, to the actual hours. I cannot define why, but this just seems inherently wrong to me.
The calculation that I use:

(1+((s-a)/s))=E or if s=3000, and a=4000 then (1+((3000-4000)/3000))=66.67%

My calculation is like a %change from standard calculation. However, there is something that also concerns me about my calculation.

If you substitute 100 for a and 50 for s, then you come to a quandary, because if you plug those numbers into the second equation the result is of course zero % efficient which doesn't sit right with me either. If you plug them into the first calculation you get 50% efficiency which doesn't really seem to work either, because you require 100% more hours to do the same work in this case. ???

Is the first calculation correct? Am I missing something altogether? Are both calculations off base?


Answered by Harley Weston.
Which number is greater? 2005-08-18
From Dante:
Which number is greater: 888....88 X 333...33 or 444.......44 x 666........67 (each of the numbers has 1989 digits)?
Answered by Claude Tardif.
What is the radius of this planet 2005-08-05
From Kelly:
Assuming that a North-South line has been established, you set up two camps that are 67 miles apart. You now set up poles at each camp perpendicular to the ground. On a certain day at "noon" the pole at the South camp casts no shadow, while at the North camp a shadow is cast. The shadow makes an angle of 89 degrees with the horizontal. What is the radius, diameter, and circumference of this planet?
Answered by Chris Fisher.
The period of sin(x) + cos(x) 2005-07-21
From A student:
WHAT IS THE PERIOD OF SIN(X)+COS(X)?
Answered by Penny Nom.
The volume of a pool 2005-07-09
From Douglas:
I have a pond that I need to treat with an anti algae solution which needs to applied based on the volume of water in the pond (in gallons).

The rough diameter of the surface of the pond = 25 ft
The depth of the pond = 8 ft
The pond shape is conical

How many gallons of water are in my pond?

Answered by Penny Nom.
The substitution method 2005-06-28
From Addie:
how do i know where to began the substitution method? how do i solve y = x + 1 2x - y = 1
Answered by Penny Nom.
Which is larger, 31^11 or 17^14? 2005-05-29
From Linda:
Which number is greater 3111 or 1714 (without using calculator).
Answered by Chris Fisher and Paul Betts.
50^99 and 99! 2005-05-23
From Romel:
Which number is greater, 5099 or 99!
Answered by Claude Tardif.
Angle of incline 2005-05-15
From Kyle:
What is the degree of incline of a 12 foot plank that goes from 10.5 inches on one end to zero inches on the other?
Answered by Penny Nom.
Surface areas 2005-05-11
From Jessica:
How can I demonstrate to my high school students the reason for the formulas for the surface area of a prism, right cylinder, and regular pyramid, and right cone?
Answered by Penny Nom.
Graphing an inequality 2005-05-02
From Janice:
I need to know how to graph inequalities. I need to know just abot everything. From graphing an equation to shading.
Answered by Leeanne Boehm.
How much sand is in a pile? 2005-04-16
From Larry:
I was wondering if there is a formula for determining how much sand would be in a pile. I am a student in medical school, and this is a bonus question for a test. I hope you can help me.
Answered by Harley Weston.
Ax + By = C 2005-04-03
From Farzana:
I was wondering if you could please explain to me how I would write the equation of line through the given points [(-3,7), (0,5)] in Ax+By=C form.
Answered by Penny Nom.
Some liquid in a cone 2005-04-03
From Vasuki:
There are 2 right cones, height is X. one of them is filled from the bottom up (round side) = x/2, when you invert the cone and add the same amount of liquid inside the second cone, what is the HEIGHT of the liquid?
Answered by Penny Nom.
An inequality 2005-03-31
From Ambross:
demostrate the following inequality:
2.Sqrt(K+1)-2.Sqrt(k)<1/Sqrst(K)<2.Sqrt(K)-2.Sqrt(k-1)

Answered by Penny Nom.
The shortest distance from a point to a line 2005-03-30
From carly:
Find the shortest distance from the point (-15, 2.5) to the line 5x+6y=30.
Answered by Penny Nom.
Gasoline in a cylindrical tank 2005-03-23
From Jennifer:

Gasoline is stored in a tank which is a cylinder on its side. Height of fuel is "h" meters and the diameter is "d". The length is "l".

I need to find the amount of gas in the tank when the height is h and also to calculate the fraction of how full it is.

Also, the part I am really confused on is this one,
E(h/d) is the error of the function of h/d, when h/d is used to measure how full the tank is. For what value of h/d is the error maximal?


Answered by Penny Nom.
1 pied linéaire 2005-02-18
From Christiane:
Pouvez-vous m'indiquer 1 pied linéaire égale quoi? 3 pied....?
Answered by Claude Tardif.
Graphing inequalities 2005-02-15
From Melanie:
Solve and Graph the following variable inequalities:

y<2x+4

y>-2x+6

3x+4y<-24

4x-6y>-24

Answered by Penny.
Extraneous solutions 2005-02-04
From Heather:
My teacher wants to know why there are extraneous solutions in logarithms?
Answered by Penny Nom.
Strategies for teaching operations with positive and negative decimal numbers 2005-02-02
From Erin:
I am wondering if you can suggest any strategies for teaching operations with positive and negative decimal numbers that do not rely on rules that must be memorized?
Answered by Diane Hanson.
Solve for x 2005-02-02
From Christie:
Solve for x

.387 = (.40 - .265x)/(sqrt(1-x2))

Answered by Penny Nom.
A line segment connecting two vertices of a polygon 2005-02-02
From James:
What is a line segment (not a side) connecting two vertices of a polygon called?
Answered by Penny Nom.
The length of a chord 2005-01-13
From A parent:
Does anyone have a formula for calculating the chord length for a segment of a circle when you know the radius and the enclosed angle or radian ?
Answered by Penny Nom.
The tide at a boat dock 2005-01-11
From Abraham:
The tide at a boat dock can be modeled by the equation

y = -2cos(pi/6 t) + 8,

where t is the number of hours past noon and y is the height of the tide, in feet. For how many hours between t=0 and t=12 is the tide at least 7 feet?

Answered by Penny Nom.
Modelling monthly temperature with a cosine 2004-12-25
From Regis:
The average monthly temperature for a location in Ontario as a function of month number can be modelled using the equation y = a cos[k(t + b)] + d.
Answered by Harley Weston.
Two intersecting lines 2004-12-18
From James:
Imagine a vertical line 1.107' tall. Leaving the top of this line, sloping down to the right at a 4:1, at what horizontal distance will the line strike a second line, which leaves the the bottom of the vertical line sloping down to the right at a rate of .02 ft/ft (or 2%)? There is a fairly easy solution to this, but I have lost my notes!
Answered by Harley Weston.
2^10 = -1/2^x - 1 2004-12-13
From Randy:
Could you explain to me how the following equation is solved please.

2^10 = -1/2^x - 1

Answered by Harley Weston.
A sequence 2004-12-11
From Amanda:
Find the nth term of the following sequence

4, 10, 28, 82

Answered by Penny Nom.
e = ln(b/a) solve for a 2004-12-01
From Daniel:
I am trying to make "a" the subject of this equation "e = ln(b/a)" but am not sure if i'm doing it right. Any help would be appriciated,
Answered by Claude Tardif.
The tangent line at an inflection point 2004-11-28
From Louise:
the equation of the tangent line to the curve y = x3 - 6x2 at its point of inflection is...
Answered by Penny Nom.
Construction of a cone 2004-11-20
From John:
I am a builder working on a project where I need to make a cone. It's a right circular cone with 15" base radius and slant angle of 30 degrees. I want to cut it out of flat sheet metal then bring the edges together to form the cone. Is this enough information?
Answered by Penny Nom.
A sequence 2004-11-19
From Liucy:
Find the Nth term: 10 40 90 160 250
Answered by Penny Nom.
An ODE 2004-11-10
From David:
I have a question that i really cant do, it is as follows:

The ODE dy/dx + 0.5y = 0.5e^(-1.5x) ; y(5) = 2

Solve the ODE subject to the given condition using exact methods and evaluate the solution y for x = 5 x=5.2 x=5.4 x=5.6 x=5.8 x=6

Answered by Harley Weston.
The combined force of two vectors 2004-10-30
From Brian:
Two teams are playing push ball with a large 8 foot diameter ball. One team exerts a force represented by the vector a = 2i + -5j, and the other team exerts a force represented by the vector b = -8i-3j.

1.Determine the direction of movement of the ball if the i axis is due east.

2.Determine the combined force magnitude.

Answered by Harley Weston.
Solving triangles 2004-10-30
From Allen:
Solve the following triangles.

Given

1. B = 20 Degrees, a = 25, b = 16
2. A = 35 Degrees, b = 2, c = 3
3. A = 32 Degrees, C = 44, c = 20

Answered by Harley Weston.
The force of a 5000 tonne lorry 2004-10-27
From Aaron:
A 50 tonne lorry is parked on a slope with a gradient of 4%. Due to the self weight of the lorry, what is the force

* acting down the slope
* acting normal (at right angles) to the slope

Answered by Penny Nom.
The point slope form of a line 2004-10-26
From Jack:
Given a set of ordered pairs, ie (1,1) (2,4) (3,7), how does one determine the rule f(n) other than by trial and error
Answered by Penny Nom.
Square roots and inequalities 2004-10-25
From Waheed:
Q1. What is the simplest way of finding a square root of any number using just a pen and paper? (I am asking this question because I browsed a few sites a didn't find any method that is simpler than the one I use. so I am just curious.)

Q2. Is it possible that you take an equation and turn it into an inequality by performing same mathematical operations on both sides?

Answered by Claude Tardif and Penny Nom.
An aircraft is flying directly from airport A to airport B 2004-10-22
From A student:
In this question, i is a unit vector due east and j is a unit vector due north. An aircraft is flying directly from airport A to airport B, which is 2000 km from A. The velocity in still air of the aircraft is (150i+50j)km/h and the ground speed is (200i-30j)km/h. Calculate

(i) the time of flight, to the nearest min
(ii) the direction of the wind.

Answered by Penny Nom.
The last digit of a large exponent 2004-10-20
From Landon:
How can one determine the last digit of a large exponent by hand. I have seen several examples of how to do this by breaking it up into smaller exponents, but is there a formula or some common things to look for?
Answered by Penny Nom.
sin(3A) 2004-10-20
From A student:
Express sin3A in terms of sinA and cosA.
Answered by Penny Nom.
Manipulating exponents 2004-10-17
From A student:
I was wondering if you can describe the steps in getting the solution to the problem below. I have the solution of 5/3 in the back of a book, but I have no idea how that answer was reached.

2raised to the 100th power + 2raised to the 98th power divided by 2raised to the 100th power - 2raised to the 98th power

Answered by Penny Nom.
A theorem involving a trapezoid 2004-09-29
From Abraham:
Given:Trapezoid ROSE with diagonals RS and EO intersecting at point M
Prove:Diagonals RS and EO do not bisect each other.

Answered by Harley Weston.
The volume of a pile of mulch 2004-09-22
From Sam:
Is there a formula to determine the cubic feet of something in a pile. IE- I need to determine the cubic feet of a pile of mulch. The pile comes to a peak, so the length and width decrease as the pile increases.
Answered by Penny Nom.
1,3,6,11,18,29,__ 2004-09-21
From Guillermo:
How do you get to the answer of this sequence 1,3,6,11,18,29,__
Answered by Claude Tardif.
Zero to the zero 2004-09-18
From Greta:
My name is Greta, and I am a 7th grader and my test is coming up. The question I would like to ask you is why 0 elevated to 0 is unidentified.
Answered by Penny.
A challenge 2004-09-17
From Lasse:
xy
xx
xyx
xxx
xyxx


"x" and "y" each represents a number. Find out the system to make the next line

Answered by Penny Nom.
Pizza for Jack? 2004-09-16
From Grace:
Jack is playing pool with Jim for $1 a game. He has only $2 and decides to play until he goes broke or has $5, at which point he will quit and go out for a pizza with Jim(Dutch treat). Jack knows from past experience that he beats Jim 60% of the time. What is the probability that Jack will get to eat pizza? Hints: Let A be the 6x6 matrix defined by A=[aij], where aij is the probability that Jack will have $(i-1)after one game is he starts with $(j-1). For example, a23 - .40 since there is a 40% probability that Jack will end up with $1 after a game is he starts the game with $2 (If Jack wins 605 of the time, he must lose 40% of the time). Also, for example, a52 = 0 since there is no way jack can have $4 after one game if he had $1 at the beginning of the game. Since Jack will stop if he goes broke or accumulates $5, a11 and a66 are both 1.

Let x0 = [0 0 1 0 0 0 ] transposed, which we interpret as saying that initially Jack has $2 with a probability 1. Then Ax0 will represent the porbability of each amount of money, $0-$5, after one game. What is the probability that Jack will be able to eat pizza by computing Akx0 for large k and finding a limiting value.

Answered by Penny Nom.
Constructing a cone 2004-09-07
From Steve:
I am trying to build crayfish traps; one of the components is a cone shaped entry section.

The cone I want to make would be 12" in diameter at the base and 12" in height, from base to peak.

I need a formula to calculate the dimensions and a method of transferring the shape onto a flat piece of material.
Answered by Harley Weston.

The intersection of planes 2004-09-07
From Joshua:
I was wondering about the intersection of planes. Can planes intersect?
Answered by Penny Nom.
Nine minutes 2004-09-02
From A student:
You have two hour glasses-one measures 7 minutes and one measures 4 minutes.How can you time 9 minutes?
Answered by Penny Nom.
Combined operations 2004-08-26
From Louise:
Question 2 1/4 x 1/8 / 1 3/4 x 12 4/9 x 3 =
Answered by Penny Nom.
When is the limit of f(x) undefined? 2004-08-03
From Nicolasa:
When is the limit of f(x) undefined?
Answered by Penny Nom.
The law of sines 2004-08-01
From Joy:
How do you solve this? Do you solve this triangle using the law of sines of the law of cosines? (ASA)

A=120DEG. B=40DEG c=35 cm I keep getting different answers.

Answered by Penny Nom.
Extraneous solutions 2004-07-28
From Nicole:
When I have a problem like (2x + 3)/5 = (x + 1)/6 and the question asks to check for extraneous solutions, how do you solve that?
Answered by Penny Nom.
Water in a cone 2004-07-28
From A student:
A vertically inverted cone( i.e. vertex down) has a radius 7 inches and height 24 inches. Water is filled to one third of its height .Find the ht of water when cone is turned upside down
Answered by Penny Nom.
The number of blocks in a mile 2004-07-18
From Valerie:
I don't know if you can help me but I was wondering how many blocks are in a mile in Ocean City, New Jersey?
Answered by Penny Nom.
x = 2^x 2004-07-12
From A student:
If the graphs of y=x and y=2x are drawn on the same set of axes, they will intersect when x is equal to what?

(1)1
(2)2
(3)0
(4)They won't intersect.

I know the answer is number 4 by graphing which is permitted; however, I would like to know if there is any algebraic way to do this type of problem.If yes, please show me how.Thanks.

Answered by Penny Nom.
An ineqlality with absolute values 2004-07-05
From Uneeza:
abs(x2 - 4x) > 3
Answered by Claude Tardif.
Pick a number greater than 1 2004-06-25
From A student:
I understand that when you pick a number greater than 1 and less than 10; multiply it by 7 and add 23, then add the digits of that number until you get a one digit number. Then multiply that number by 9, add the digits of that number until you get a one digit number, subtract 3 from that number and divide the difference by 3; that this process will always give you the result of 2. Does this have a name or theory for it as to why the answer will always be 2?
Answered by Penny Nom.
Solving an equation 2004-06-14
From Anthony:
I was hoping that you could help me with this problem. I believe that this is a high school level problem but as an adult who has forgotten the "rules", I am stuck.

the equation is this: x=t2/(t1-t2)

we know that x=5.73
t1=303

we must solve for t2

Answered by Penny Nom.
Finding bearings 2004-05-24
From James:
This question is about finding bearings. A boat race starts from point A, goes North to Point B, a distance of 1000 meters. The course is triangular. The bearing from point B to point C is South 70degrees West. The distance from Point B to point C is 1500 meters. Find the course bearing from C to A.
Answered by Penny Nom.
Solving inequalities 2004-05-03
From Joe:
Let x be a real number find and graph the solution to the following sentances:

2x-10+9 4X+2<15 5X+3>(x-1)

Answered by Peny Nom.
The equation of a line 2004-04-30
From Amy:
Write the equation of the line that contains the point (-4,6) and is perpendicular to the line y=5x+100.
Answered by Penny Nom.
What's it called? 2004-04-22
From Gerry:

I'm a father and a grandfather and have come up with a game for my offspring to play while we're on the road. When we see a license plate, the object is to be the first one to add all the numbers on it, and come up with THE one digit number that sums them up.

For example: ABC-787 = 7+8+7 = 22 = 2+2 = 4

Another example is 2932 = 2+9+3+2 = 16 = 1+6 = 7

Up 'til now, I've called it just plain "Numerology", but I'm sure that there's a math term for what we're doing, and I'd sure appreciate it if you could tell me what it is!


Answered by Chris Fisher and Penny Nom.
The line through (4,2) and (4,-8) 2004-04-17
From Kim:
Write an equation for the line.
Given points (4, -8) and (4,2) are on the line.
I believe m=undefined.

How do I solve for b when m is undefined, and then plug in the values for m, b, and y?

Answered by Penny Nom.
What is the nth term 2004-04-07
From Jason:
1 4 10 20 35 56 84
what is the nth term

Answered by Claude Tardif.
Algebraic simplification 2004-03-31
From Chris:
Simplifying Exponents

Question

b^5d^2/b^3d^8

Answered by Penny Nom.
A stained glass window 2004-03-29
From Kay:
I'm doing a stain glass project and it's on a 4 foot across octagonal window...and I'm trying to set up the pattern and I don't know how long the sides are!
Answered by Penny Nom.
Large exponents 2004-03-26
From John:
I have encountered problem with the lack of memory every calculator seems to have. No calculator, on or off the computer, I've found has the amount of memory or writing space to calculate the sums I want to solve. The sums are in great importance for my continued progress. The sums are following:

16777216^1310270, 16777215^995328, 16777215^786432 and 16777215^480000

Answered by Claude Tardif.
What's the next term? 2004-03-22
From Garrett:

A friend hit me with these questions and won't tell me the answer nor how... please help? I've racked my brain and did all sorts of formulas, but I can't get it to work out... please...

What's the next number in this series... 5, 12, 17, 29, 46, ?

And this series... 12, 19, 28, 39, 52, 67, ?

and these... ocoa, boco, oboc, dobo, odob, ?


Answered by Claude Tardif.
Equivalent sets 2004-03-06
From A student:
If A=(1,2,3,4,...) and B=(5,10,15,20,...), is A equivalent to B. Why or Why not ?
Answered by Penny Nom.
Cosine of 35 degrees 2004-03-03
From Jason:
How do you find the exact solution to cosine 35 degrees.
Answered by Chris Fisher.
A dress-maker's pattern 2004-02-27
From A dress-maker:
Does a dress-maker's pattern for a dress, such as a skirt made up of several pieces of isosceles trapezoid, qualify as a net? How about patterns for other types of clothing or accessories e.g. jackets, handbags?Î
Answered by Diane Hanson.
An elliptic cone 2004-02-24
From Ben:

I am building a model for my architecture class. I need to build a elliptic cone out of chipboard and i have no idea how to do this.

The cone needs to be 20in tall and the ellipse has a max radius of 10in and a min radius of 8in.

So my question is how do i lay this out on a piece of paper so that i can form the cone after i cut it out.


Answered by Penny Nom.
Graphing lines 2004-02-08
From A student:
how do you solve y=1\3x+3 and y=1\3x-3 by graphing?
Answered by Penny Nom.
The incline of an exercise machine 2004-02-02
From An exerciser:
This is a question about incline percentage. On an exercise machine if the elevation is raised 3 and one half inches from a level position then what would be the incline percentage?
Answered by Penny Nom.
The vertex of a cone 2004-01-29
From Richard:
Please help me explain to my fourth graders as to why a cone has a vertex even though it does not have any straight edges.
Answered by Chris Fisher.
Intersecting a line and a curve 2004-01-29
From Senthil:
between line and curve how can i find intersection point? could you write me the formula and explanation also sir.
Answered by Penny Nom.
Sin(3x), cos(3x) and tan(3x) 2004-01-28
From Jon:
What is the identity for cos3x, sin3x, and tan3x? In class, we learned double angel identities and were asked to find out the identity to these three trig functions. If you can help, please do. Also, i know that the cos4x- sin4x is the same as cos2x. Is cos8x-sin8x = cos2x also true? Thank you.s
Answered by Chris Fisher.
x^x^x^x^... 2004-01-23
From Ryan:
you have a number say x and it is to the power of x which is to the power of x and so on infinite times like x^x^x^x^x^x^x^... i have to figure out what x is so that the answer is always 2
Answered by Penny Nom.
Two consecutive negative integers 2004-01-17
From Amanda:
Find two consecutive negative intergers whose product is 182.
Answered by Penny Nom.
Inequalities 2004-01-08
From Michael:
How do you use inequalities in your job and in your everyday life?
Answered by Penny Nom.
Solving an equation 2004-01-03
From A student:
the question is to solve and check equations and the question is
2n + 3n + 7 = -41

Answered by Penny Nom.
Making a windmill 2004-01-02
From Matthew:
I am a farmer in Ontario. It has been almost 20 years since high school. I am toying with making a windmill. The output chart for the the old generator I have is shown below. Before I tear it appart I would like to develop a formula from the chart that can predict the output at various speeds.
Answered by Penny Nom.
A new way to measure randomness 2003-12-31
From Stephanie:

Last year, I did a science project in which I asked, "Which shuffles better, an automatic card shuffler or shuffling by hand?" To measure this I decided the "best" shuffler was the one to become random first. Last year, to measure randomness, I numbered cards 1-52 and had the subjects shuffle them until they broke up the rising sequences or reached 10 shuffles. (Usually 10 shuffles came first...) Anyway, I did the same thing with the automatic card shuffler, and, as hypothesized, the automatic card shuffler randomized the deck first.

This year, I have decided to continue the project. The problem is, I need a new way to measure randomness without the use of fancy computers or something. I have searched the Internet, I have posted my query on websites based on math, and I have searched the local library.

I have found many useful things on the Internet, but none of them can tell me a new way to measure randomness. I cannot do a perfect shuffle, and I am not terribly gifted in the art of using computers. If you have any information (anything will help) or advice, I would be greatly obliged.


Answered by Andrei Volodin.
Making a cone 2003-12-22
From Tracie:

I am working on a craft project at home and I have been given the following information:
16" tall and 13" diameter at base, with 1 and 1/2 " opening at top.

Is there a basic formula for creating a cone with this info?


Answered by Claude Tardif and Penny Nom.
Business trip 2003-12-19
From Ameer:
A businnessman drives from Washington, D.C., to Boston, a distance of 442 miles, and then makes the return trip. On the way to Boston, he drives 65 miles per hour, taking an 1-hour rest stop during the drive. After finishing his business in Boston, he make the return trip driving at 60 miles per hour and takes a 45-minute rest stop halfway through the trip. Which leg of the journey, Washington, D.C. to Boston, or Boston to Washington, D.C., takes the longer time?
Answered by Penny Nom.
How far can you see? 2003-12-15
From Judy:

How far apart, assuming no obstacles, can two people stand and still see each other?

i know this deals with the curvature of earth, but i can't figure out the formulas involved.


Answered by Chris Fisher.
Non-euclidean geometry 2003-12-08
From Geoffrey:
How can you use non-euclidean geometry to navigate on a sphere? What geometers did work in this area?
Answered by Chris Fisher.
Non-Euclidean geometry 2003-12-03
From Geoffrey:
What are the applications of Non-Euclidean geometry (especially hyperbolic and spherical)?
Answered by Walter Whiteley.
Mr.Carter is very cautious 2003-11-26
From Bob:
Mr.Carter is very cautious. He decides to invest in only three stocks: one low stock, one high stock, one medium stock. Given that the expected annual yields are 6% for low stock, 7% for medium stock, and 8% for high stock, he wants his investment in medium stock to be half of his total investment in low and high stock. How much should I invest in each type fo stocks to expect a total annual return fo $650 form my investments?
Answered by Harley Weston.
An inequality whose solution is all points 2003-11-25
From Katie:

Hi my name is Katie and I am in the 9th grade and I would like help on this question. Is it possible to have a system of inequalities whose solution is all points?

1.I need to know why (explanation)

2.how to do it(example)


Answered by Penny Nom.
Laws of sines and cosines 2003-11-23
From A parent:
On the one side of a stream lines PA= 586.3 feet, PB = 751.6 feet are measures, angle APB being 167 degrees and 36 min. Q is a point on the opposite side of the stream. Angle PAQ=63 degress and 18 min and PBQ=49 degrees and 24 min. Find PQ.
Answered by Penny Nom.
A point and a line 2003-11-19
From Meenakshi:

Given a line segment L(x1,y1) to (x2,y2) and a Point P(x3,y3).
I need the formula to find a point on the line L that is closest to the point (x3,y3).


Answered by Penny Nom.
Lines 2003-11-14
From A student:
What is a name that a group of lines pass through?
Answered by Chris Fisher.
A least squares line 2003-11-09
From Michelle:
Hooke's Law asserts that the magnitude of the force required to hold a spring is a linear function of the extension e of the spring. That is, f = e0 + ke where k and e0 are constants depending only on the spring. The following data was collected for a spring;

e: 9 , 11 , 12 , 16 , 19
f : 33 , 38 , 43 , 54 , 61

FIND the least square line f= B0 + B1x approximating this data and use it to approximate k.
Answered by Penny Nom.

Intercepts 2003-11-04
From A student:
I am totally confused on how to determine intercepts of an equation and then to graph the line. For example, how to determine the intercepts of 2x-3y-12=0.
Answered by Penny Nom.
Maine and Nevada 2003-10-24
From Jimmy:

From 1989 to 1990, the population of Nevada increased b y 157,000, and that of Maine increased by 30,000. In 1990, the population of Nevada was 1,206,152, and that of Maine was 1,233,223. If the populations of the two states continue to increase at the same rates, when will the populations of Nevada and Maine be the same?

Write a verbal model for this problem
Write an equation for the model
Solve the equation and answer the question.


Answered by Penny Nom.
Squares in a rectangle 2003-10-21
From Raj:

Draw a rectangle with sides of 3 and 4. Divide the sides into 3 and 4 equal parts respectively. Draw squares joining the points on the sides of the rectangle. You will have 12 small squares inside the 3 x 4 rectangle.

If you draw a diagonal of the rectangle, it will intersect 6 of the the 12 smaller squares.

Similarly, if you have a 4 x 10 rectangle, the diagonal would intersect 12 of the 40 squares inside the rectangle.

Is there an algebric equation that determines the number of squares that will be intersected by the diagonal of a rectangle?


Answered by Chris Fisher.
The sketch of a graph 2003-10-07
From A student:
I was wondering how do you figure out if a graph has a horizontal tangent line. One of my homework problem was to sketch the graph of the following function; (4/3)x3-2x2+x. I set f''(x) ( the second derivative) of the function equal to zero and got the inflection point:(1/2,1/6). Also i am having trouble finding the concavity for x>1/2 and x<1/2, i am getting a different answer from the back of the book, the graph i draw looks completely different from the correct answer.
Answered by Penny Nom.
The Sieve of Eratosthenes 2003-09-18
From Lynn:
My daughter has been asked to find all the prime numbers by using the Sieve of Eratosthenes. I have no understanding what this means.
Answered by Penny Nom and Claude Tardif.
Notation 2003-09-15
From Kim:

Question:
i have a couple of questions

1. write in standard form
a)4x2x8 b) 5 to the power of 4

.
.
.

Answered by Penny Nom.
Two line segments and a plane 2003-09-11
From Laura:
Do all figures made up of two segments lie in a plane?
Answered by Penny Nom.
The general equation for a sphere 2003-09-11
From Jaidev:
Is there any general equation for a sphere?
Answered by Penny Nom.
Can 2 vertical planes intersect? 2003-09-06
From Erin:
My question is can 2 vertical planes intersect?
Answered by Penny Nom.
One million seconds 2003-09-04
From Vince:
If you can write a number each second for one million seconds, how many hours would it take?
Answered by Penny Nom.
One percent of a billion 2003-09-04
From Charles:
What is one percent of one billion?
Answered by Penny Nom.
Positive, negative and zero 2003-09-04
From Brian:
Can the Value of zero be positive or can it be negative or can it be both.
Answered by Penny Nom.
Newton's binomial theorem 2003-08-30
From William:
According to page 126 of Murtha & Willard's "Statistics and Calculus" (Prentice-Hall, 1973), Newton's binomial theorem can proved inductively. I suppose that was his method, which I would like to see.
Answered by Penny Nom.
How many cubic yards must I purchase? 2003-08-29
From Walt:

I am trying to determine how much top soil I must purchase (by the yard).

The area is 80' x 15' and I want it 3" thick. How many cubic yards must I purchase?


Answered by Penny Nom.
Why is 5 to the 0 power always 1? 2003-08-24
From Crystal:
Why is 5 to the 0 power always 1?
Answered by Penny Nom.
X.9999... and X+1 2003-08-23
From David:
I have read your answers to the questions on rational numbers, esp. 6.9999... = ? and still have a question: The simple algebraic stunt of converting repeating decimals to rational numbers seems to work for all numbers except X.999999.... where X is any integer. The fact that the method yields the integer X+1 in each case seems to violate the completeness axiom of the real numbers, namely that there is no space on the number line which does not have an number and conversely that every geometric point on the number line is associated with a unique real number. In the case of 3.999... for example, it seems that both the number 4 and the number 3.9999.... occupy the same point on the number line. How is this possible???
Answered by Penny Nom.
Water in a cone 2003-08-12
From Adrienne:

Water is poured into a tank in the shape of an inverted right circular cone.ð The height of the tank is 8 m and its radius at the top is 4 m.

a. Draw and label a picture to represent this situation.ð (I know how to do this)

b. Identify all variable quantities. (h = 8m, r = 4m)

c. Find an equation that relates the variable quantities, and reduce the number of variable quantities to two.

I was thinking about the equation V = 1/3 pi r2 h, which is the Volume of a cone, but I am stumped as to how I am supposed to "reduce the number of variable quantities to two." Can you point me in the right direction?


Answered by Penny Nom.
A sphere inscribed in a cone 2003-08-10
From A student:
A sphere with radius 5cm is inscribed in a right circular cone 20 cm in height.find

(a) the base radius ,volume of the cone
(b)volume of the shaded space( to 3 sig fig)

Answered by Penny Nom.
The incline of a road 2003-08-03
From A driver:
incline- % grade as to degree. ie 6% hwy grade= what degree of incline?
Answered by Penny Nom.
nine digit numbers 2003-07-23
From John:
With a nine digit number, with each of the nine digits having a possibility of 10 different numbers then what is the total number of possible mathematical variations in the nine digit number. i.e. Social Security numbers have nine digits and if each of the nine digits have a possibility of being any one of ten numbers, i.e. 0,1,2,3,4,5,6,7,8,9. Then what is the formula to calculate the maximum possible number of variations in this nine digit number and what is the mathematical maximum possible number of variations of this nine digit number?
Answered by Penny Nom.
Cathy's Clothing 2003-07-15
From Cathy:
I am supposed to write a practical and useful word problem that would use the following algebriac inequality to solve it: 12x + 25 > 15x + 10. State the quanity that represents x and solve the inequality.
Answered by Penny Nom.
Sunshine? 2003-07-11
From lori:
It is noon, your lunch hour, but you cannot go out because there is a terrific hailstorm. Turning on your radio you hear the weathercaster predict that the hail will change to rain and that it will pour all day today. How can you determine whether the sun will be shining in 36 hours?
Answered by Claude Tardif.
A triangle in the complex plane 2003-07-10
From Scott:
The vertices O and A of an EQUILATERAL triangle OAB in the complex plane are located at the origin and 3 + 3i. Find all possible values for the complex number representing the vertex B. Give the location of B in both polar and cartesian form (to 2.d.p)
Answered by Penny Nom.
Solve x = y^z for z 2003-06-28
From Nathan:
If x = y^z is there a way to solve for z, if x and y are given, without guessing or already knowing the answer?

ie: 64 = 2^z
6 is the obvious value for z. Is there a way I could solve for this variable using a formula, which would also apply when more complex values were substituted for x and y?

Answered by Penny Nom.
Odd powers of sine and cosine 2003-06-25
From Antonio:
Can you please tell me how to integrate a trig function involving sine and cosine? I know if the powers of both the sine and cosine are even and nonnegative, then I can make repeated use of the power-reducing formulas. But for the question I have on my hand, the powers of both sine and cosine are odd: ( sin3x + cos7x ) dx.
Answered by Harley Weston.
The sum of two numbers is 5 and their difference is 2003-06-16
From Akhil:
The sum of two numbers is 5 and their difference is 11. What is the product of the two numbers?
Answered by Penny Nom.
A project about crosses 2003-06-10
From Joel:

I have this project to do about crosses and I can't think of what the answer is for the following questions: What is the area rule of the crosses (the table below will help you)?
Cross NumberArea sq cm
15
213
325
441
561

I also need to know what the formula is for it?


Answered by Penny Nom.
Two trig problems 2003-06-10
From Bett:

I have this ongoing trouble with trig and solving triangles with laws of cosines and sines!! For example if it asks to solve triangle FGH, given angle G=102.7 , side f=14.2, and h=18.6. Now do I use law of cosines because I don't have the measure of an angle and length of the opposite side??I don't know where to go from here,I am totally confused!!!

I also have a problem with this word problem I have been doing. It asks: An airplane flies 847.5 km at a bearing of 237.3 degrees. How far south and west fo its original position is it? Huh? Please help!


Answered by Penny Nom.
5 to the power n 2003-05-28
From Sam:
What is the integer n for which

5 to the n power + 5 to the n power + 5 to the n power + 5 to the n power + 5 to the n power = 5 to the 25th power?

Answered by Penny Nom.
sin theta = 7/8 2003-05-07
From Patty:
If sin0 = 7/8 and 0 is in quadrant 2, find the other five trigonometric functions of 0. (report your answers in radical form)
Answered by Penny Nom.
Rules of exponents 2003-05-05
From Carl:
Hi, I am a student who would like to recall how to multiply exponents. Here is such an equation:
6.02569 X 1025 X 5.254 =?

Also, adding exponents. Don't I just add subtract the exponents separately?
Such as 523 +15-12 =??

Answered by Penny Nom.
Write sin(3x) in terms of sin(x) 2003-05-05
From A student:
Write sin 2x in terms of sin x
Answered by Penny Nom.
A circle, tangent to two circles and a line 2003-04-30
From Keith:
I have a horizontal line (that is treated as a datum line or the X axis), with two circles having their center points at different heights from that line (X1,Y1 & X2,Y2). The two circles are also at different diameters (R1 & R2). Both circles and the line (X-Axis) do not intersect nor are they tangent. My goal is to determine the maximum diameter of an inscribed circle that will fit between all three.
Answered by Chris Fisher and Harley Weston.
One one 2003-04-22
From Brad:
find the next two rows of numbers?
     1                        
    1 1                        
    2 1                      
  1 1 1 2                     
  3 1 1 2                    
1 1 1 2 1 3 

Answered by Leeanne Boehm.
Manipulating exponents 2003-04-21
From Denita:
Rewrite using only positive exponents:

(4a^4b^2)^4 * (4a^ -3 bc^2)^ -3

Answered by Harley Weston.
The substitution method 2003-04-14
From Patty:

I need help with the following:

5x - 4y = 13

2y + 3x = -1


Answered by Penny Nom.
A fractional inequality 2003-04-08
From Jessica:
Solve for x. Write in interval notation.

[(x2)-9]/[x-5] >= 0

Answered by Penny Nom.
The vertex of a cone 2003-03-27
From Holly:
I read your response to Callie about whether a cone has a vertex or not. Is it ONLY a vertex if both halves of the cone are together or can one half of the illustration have a vertex?
Answered by Walter Whiteley.
Friends and enemies 2003-03-24
From Becky:

Consider a room that contains six people. Any two people are either friends of each other, or they are enemies.

A. Argue why there are three people, all who are friends, or there are at least three people, all who are enemies

B. Rephrase the situation using graph terminology, using all of these terms correctly: vertex, edge, graph, complement, clique, independent set, and bipartite.


Answered by Penny Nom.
A sequence that converges to e 2003-03-16
From Dane:
Something I noticed fooling around with a calculator about 30 years ago.
Considering e = 2.718281828459045....
Using Window's Calculator you will find


1.111 = 2.8531167...
1.01101 = 2.731861...
1.0011001 = 2.71964085...
1.000110001 = 2.71841774...
1.00001100001 = 2.7182954...
1.00000110000011 = 2.178231875...
1.000000110000001 = 2.178219643...


There apears to be a pattern. My conjecture is:


1.'infinite number of zeros'11'infinite number of zeros'1 = e.


Answered by Penny Nom.
y < 2x + 1 2003-03-01
From Erika:
how can I solve and graph y<2x+1 and which side in the graph should i shade after i finish solving my inequality?
Answered by Penny Nom.
A number line 2003-02-27
From Shery:

My seventh grader problem of the month

0__________1______________5____>

This is a arrow, the number should be below

A.Mrs Decker created an arrow representing a number line shown above. She wanted to find points and label them with a heart (G) for Valentine's Day so that the fraction 5/g is less than 1. (be sure to mark the G and not the fraction 5/g). She pondered, "Are there any other locations for G?" Is so help her description the location of all these points. If not why not?


Answered by Penny Nom.
Storyteller figurines 2003-02-10
From A student:
It takes 3/4 of an hour to bake a storyteller figurine. If only one figurine can be baked at a time, how many can be baked in 6 hours?
Answered by Penny Nom.
Finite differences 2003-02-10
From Jenny:

I need to find a formula that will work with any number.
I am finding the volume of a 3d cross- shape. Here are my results so far:

Term Number      0   1    2    3     4      5
nth term         1    7   25   63   129    231
1rst diff           6   18   38   66     102
2nd diff              12    20   28   36
3rd diff                  8     8    8

I can't seem to find a formula that will work with any number. Any help would be much appreciated.

Answered by Penny Nom.

Two investments 2003-02-07
From A student:
you have $7200 in principal and invest different sums @ 10% and 14%. You receive 920 at end of term. (one year)What amounts are invested at what rate?
Answered by Penny Nom.
Two equations in two unknowns 2003-02-06
From Patrick:
What are all ordered pairs of real numbers (x, y) for which: yx2 - 7x + 12 = 1 and x + y = 6?
Answered by Penny Nom.
Two airplanes leave Dallas 2003-02-06
From A student:
TWO AIRPLANES LEAVE DALLAS AT THE SAME TIME AND FLY IN OPPOSITE DIRECTIONS. ONE AIRPLANE TRAVELS 80 MILES PER HOUR FASTER THAN THE OTHER. AFTER THREE HOURS, THEY ARE 2940 MILES APART. WHAT IS THE RATE OF EACH AIRPLANE?
Answered by Penny Nom.
Arithmitic sequence 2003-02-01
From A student:
I am having problems solving this arithmetic sequence... 1, 5, 10, ___, 50, 1.00, ___, 10.00, ... I believe the answers to be 25 and 5.00 but I can't figure why.
Answered by Claude Tardif.
Extraneous solutions 2003-01-24
From Paul:

What is an extraneous solution and in what cases do you get one?

How do you know it is extraneous?


Answered by Penny Nom.
Collinear Points 2003-01-13
From Gary:

Which of the 4 points are collinear when you construct the following concurrent lines or rays of a triangle?

  1. P(1), the point where the angle bisectors intersect.
  2. P(2), the point where the altitudes (or extensions) intersect (inside or outside of the triangle).
  3. P(3), the point where the medians intersect.
  4. P(4), the point where the perpendicular bisectors (or extensions) of the three sides of a triangle intersect.


This is for my 9-12 high school class in geometry.
My name is Gary


Thanks for your help.
Gary


Answered by Harley Weston and Chris Fisher.
The intersection of conics 2002-12-19
From Glenda:
We are studying systems of equations where two conic sections are the two equations that we are solving simultaneously. We were studying the number of solutions that are possible if you have an ellipse and a parabola. We all agree that there can be none, one, two, three or four solutions. The question that the students had for me was whether or not a portion of an ellipse and a parabola can overlap and thereby allow an infinite number of solutions. What should I tell them?
Answered by Chris Fisher and Harley Weston.
y = 1 - sin(x + 60) 2002-12-10
From Eman:
Sketch the graph of y = 1 - sin(x+60). for 0 <= x<= 360, giving the coordinates of the maximum and minimum points and the pints where the curves crosses the y axis.
Answered by Penny Nom.
A function that is onto but not one-to-one where f:N-->N 2002-12-06
From Lisa:
A function that is onto but not one-to-one where f:N-->N
Answered by Penny Nom.
x + y=80 and x + z=100 2002-11-28
From PJ:
x + y=80
x + z=100

can you solve for x, y or z?


Answered by Penny Nom.
A two stage rocket 2002-11-26
From Hoda:
a two stage rocket accelerates in free space by ejecting fuel at a constant relative speed , v(ex). the full fuel load makes up 80% of the initial mass of the entire two stage rocket . the rocket accelerates from rest until at the end of the first stage when 75% of its fuel has been burnt. find an expression for the speed of the rocket at the end of the first stage in terms of v(ex).
Answered by Claude Tardif.
A lampshade from a cone 2002-11-26
From Ellsie:
I need to make a pattern to cover an old lampshade. This is actually the bottom portion of a cone. Please help me figure out how to draw this pattern, so that we can complete our project.
Answered by Penny Nom.
Performance reviews 2002-11-21
From Lara:
Perhaps you can help.

He's doing performance reviews at work.

The average performance review is 3.92 on a 5.0 scale. This person would receive a 3.5% raise for next year.

The highest performance review is 4.9 and this person would recieve a 6.0% raise for next year.

How do I solve for the other performance review numbers.


Answered by Penny Nom.
The substitution method 2002-11-15
From A student:
Solve by using the substitution method.

x = 3y - 1
x + 2y = 9


Answered by Penny Nom.
Determining a parabola 2002-11-06
From Shelley:
let (0,3) and (1,9)and (-1,1) be given points in a parablola. Determine a b and c
Answered by Penny Nom.
Compass points 2002-11-05
From Mhairi:
I was asked this in maths but I am not sure if it is geography. Name all the compass points? We have been given 8 but he told us there is more and we have to name them.
Answered by Chris Fisher.
The percentage grade of that hill 2002-11-05
From Cathy:
If there is an 80ft climb over a kilometer(about 3280ft) what is the percentage grade of that hill?
Answered by Penny Nom.
Rounding 27.27 2002-11-03
From A parent:
If you have a problem 27.27 and you need to round to the first 7 the answer would be 27

Would 27.0 be acceptable as well or is it completely wrong.


Answered by Penny Nom.
Exponential notation 2002-10-19
From Ivy:
I understand how to figure out for example 4 to the power of 3,what I don't understand is how to figure it out using both negative numbers and negative exponents,as an exaple -4 to the power of 6 and what makes this different that (-5) to the power of 7,please help me if you can.I am an adult taking correspondence and finding it quite frustrating.
Answered by Penny Nom.
Three algebra problems 2002-10-13
From Veronica:
Solve the following inequality:

5(xsqured-4)/(xto the 5th(2x-5)to the 3rd) < or equal 0

Solve the following equations for all roots

square root of x+2=-1+square root of 2x+3

what's ZERO FACTOR PROPERTY???

The hypotenuse of an isosceles right triangle is 7cm. long. Determine the lenghts of the other sides.


Answered by Leeanne Boehm.
Exponent 0 2002-10-02
From Debbi:
can you please tell me what the exponent 0 is (i.e. 70)
Answered by Penny Nom.
The interrelations of the propositions in Euclid I 2002-10-01
From Rob:
I'm a student at St. John's College in Santa Fe New Mexico, are school is devoted to the great books and we're now reading Euclid's Elements in english and Attic Greek. We have an assignment to show a frequency or flow chart of how all the propositions of book one are interrelated.
Answered by Chris Fisher.
Two rate problems 2002-09-30
From Rebecca:
  1. There are two small holes in the bottom of a tub filled with water. If opened, one hole will empty the tub in three hours; the other will empty it in six hours. If both holes are opened at the same time, how long will it take to empty the tub?

  2. An airplane flies 400 miles per hour in calm air. It can cover 900 miles flying with the wind in the same time that it can cover 700 miles against the wind. What is the speed of the wind?

Answered by Penny Nom.
A cone that is cut off at the top 2002-09-23
From Stuart:
I have to work out the dimensions and arcs of a cone that is cut off at the top.

I.e Top diameter is 33mm to bottom diameter is 43mm and the depth is 80mm

Are you able to work what the flat of this cone would be as I need to design within the flat area and then have it cut out.

I really need to know what the flat of it is before it is cut and curled to form the above cone.


Answered by Walter Whiteley.
Fractional exponents 2002-09-20
From Jill:
The problem is with fractional exponants:
10 1/3 mult. by 10,000 The 1/3 is an exponant of 10.

Answered by Penny Nom.
A linear function 2002-09-09
From Chad:
If y = 9x - 13 does it represent a linear function? And if it does what is its slope and is it a direct variation?
Answered by Peny Nom.
8,27,64 . . . 2002-09-05
From Mike:
We have been trying with no luck all night to complete the following math pattern (next 3 entries) 8,27,64 . . . Any ideas??
Answered by Penny Nom.
1, 2, 4, 8, 16, 32, etc. 2002-09-04
From Ginger:
I've been working with finding the nth term for a few weeks, and i can't seem to figure this one out!

1, 2, 4, 8, 16, 32, etc.


Answered by Leeanne Boehm.
A group of 9 2002-08-31
From William:
what is the mathmatical name for a group of nine like a group of 3 is called 'triplets'
Answered by Penny Nom.
Mixing gas and oil 2002-08-27
From Sherry:
how do I calculate the correct fuel ratio for a boat motor? 50:1 the oil containers only say how many ounces to add to 5 gallons of fuel to make a 50:1 ratio.(13 ounces to 5 gallons of fuel) I have a 20 gallon tank so what if I add 11.6 gallons of fuel, what formula would I apply to get the correct measurement of oil to add?
Answered by Penny Nom.
When does an inequality represent the area above the line? 2002-08-25
From Tamara:
How do you determine that an inequality represents the area above the line and when do you include the line in the solution?
Answered by Penny Nom.
Parallelograms 2002-08-21
From Moorthy:
11 parallel lines are intersected by another 9 parallel lines.Then how many parallelograms are formed?
Answered by Walter Whiteley.
A paper model of a cone 2002-08-14
From Bruce:
I have made a paper model of a cone, cut a sloping section, and removed the top. I have drawn the major and minor axis on the paper surface of the section. The major axis is not symmetrical about the minor axis. To me, this is not an ellipse. To me, an ellipse is a tubular section, because this gives a symmetrical major axis. What is your opinion?
Answered by Walter Whiteley and Chris Fisher.
Two equations 2002-07-26
From Derek:
1. 3x + 2y = 4
2. -7x + 2y = 24

finding x and y.


Answered by Penny Nom.
Some inequalities 2002-07-25
From A student:

1)-5 <= -3X+1 < 1

2)4X-2 < 6 OR X+2 > 9


Answered by Penny Nom.
Future value 2002-06-27
From Susan:
If one were to invest $115 a month for 20 years and expect a 4% annual return, what is the value of this money in 20 years?
Answered by Leeanne Boehm.
What is the next term? 2002-06-21
From A student:
What is the next term in the sequence:

1, 5, 14, 30.....


Answered by Penny Nom.
Ratios and weights on Neptune 2002-06-03
From Janice:
The ratio of an object's weight on earth to it's weight on Neptune is 5:7. How much would a person who weighs 150 pounds on earth weigh on Neptune.
Answered by Penny Nom.
Conics 2002-05-29
From Brooke:
Which conic cannot be generated by an intersection of a plane and a double napped cone?
Answered by Chris Fisher.
Linear programming 2002-05-27
From Jes:
A machine shop makes two parts, I and II, each requiring the use of three machines, A, B, C. Each Part I requires 4 minutes on Machine A, four minutes on Machine B and five minutes on machine C. Each Part II requires five minutes on Machine A, one minutes on Machine B and six minutes on Machine C. The shop makes a profit of $8 on each Part I and $5 on each Part II. However, the number of units of Part II produced must not be less than half the number of Part I. Also each day the shop has only 120 minutes of machine A, 72 minutes of Machine B, and 180 minutes of Machine C available for the production of the two parts. What should be the daily production of each part to maximize the shop's profit?
Answered by Claude Tardif.
Telephone banking 2002-05-22
From Fiza:
To use telephone banking to pay bills, the customer has to enter the last three digits of each bill. the numbers 0 to 9 can be used. if the number happened to be the same on more tan one bill, the customer has to enter the first three digits as well.

(a) what is the probability that a person has 2 bills to register with the same last three digits?
ANSWER:P(2 bills to register)=1/10P3 1/(10!/7!), 1/(10*9*8)=.001389

(b) what is the probability that a person with 10 bills to register has at least 2 bills with the same last three digits?
ANSWER:P=1/(10*10*10) =0.001


Answered by Andrei Volodin.
How far apart are the transmitters? 2002-05-18
From Jeff:
A ship at sea is 70 miles from one transmitter and 130 miles from another. The measurement of the angle between the signals is 130 degrees. How far apart are the transmitters?
Answered by Penny Nom.
Sequences that agree on their first four terms 2002-05-16
From Mike:
Are there two formulas that define sequences that agree on their first four terms, but differ on the fifth term and all succeeding terms?
Answered by Chris Fisher.
The law of cosines and obtuse angles 2002-05-09
From Bryant:
The question that I am pondering is that I need to derive the law of cosines for a case in which angle C is an obtuse angle.
Answered by Penny Nom.
oneths 2002-04-30
From Logan:
why are there tens-tenths hundreds-hudreths, but not oneths

if that made no sense then how about this why does everything have two ways of saying things like tens and tenths but not a oneths


Answered by Claude Tardif.
What's the next term? 2002-04-29
From A student:
Find the nth term for the following:

1) 12, 30, 56, 90, 132
2) 6, 30, 84, 180, 330
3) 8, 15, 24, 35, 48
4) 10, 17, 26, 37, 50
5) 24, 40, 60, 84, 112
6) 24, 60, 120, 336, 504

Please show the FULL working out.


Answered by Penny Nom.
25m = 100 2002-04-22
From Megan:
If 25m = 100, Then m equals what number?
Answered by Penny Nom.
Lines of symmetry 2002-04-22
From Cindie:
How many lines of symmetry do the following figures contain?

trapezoid:

rhombus:

hexagon:

pentagon:


Answered by Walter Whiteley.
An augmented matrix 2002-04-20
From A student:
Hi my math teacher asked us to solve an augmented matrix. I am in twelfth grade and need help. The book we are working on is college algebra. Here it is

a+2b+c=0
2a+5b+4c=-1
a-b-9c=-5


Answered by Penny Nom.
The vertex of a cone 2002-04-13
From Callie:
Does a cone have a vertex?
Answered by Penny Nom.
3/4n-6=12 2002-04-07
From Diane:
I am in 6th grade advanced math and I just started Algebra. My teacher tries to explain it to me but I just can't get it. I am having difficulty understanding it. Please HELP!!! I will type a problem, if you could answer it and show your work so i can understand it that

would be great. Please explain in simple terms.

here is a couple problems I am having trouble with,

PROBLEM #1

3/4n-6=12


Answered by Penny Nom.
Pairs of equations 2002-04-04
From A student:

high school level
student is asking

y=4x x=-4y 
x+y=5 3x+2y=20   

y=x-1 3x-y=4 
x+y=3 2x-3y=-9   

x+5y=4 
3x+15y=-1 

. . . 

Answered by Penny Nom.
100 from four 9's 2002-03-27
From A student:
My teacher gave us a math problem to try and figure out and I am stumped. The problem was this.... by only using four nines, how can you get an answer of 100? You can add, subtract, multiply or divide...anything you need to do ,but you can only use four nines.
Answered by Leeanne Boehm.
If the matrix A is inverible and AB =AC, then B = C 2002-03-27
From Vikki:
i hope you can help i am soooo stuck here goes:

a)

 A= 0 1 B= 1 1 c= 2 5    0 2    3 4    3 4 
A,B and c are matrices

Evaluate AB and Ac (which I can do)

then

b)
I need to prove that if the matrix A is inverible and AB =AC, then B = C. Why does this not contradict what happened in part a)?
Answered by Leeanne Boehm.

A probability problem 2002-03-23
From Kate:
If I have a spinner and then spinner has the numbers 1,2,3,4 on it and it is spun 3 times, what is the probability that the spinner will stop on 4,3,and 1 ? In that order?
Answered by Leeanne Boehm.
A trigonometric identity 2002-03-22
From Debby:
I am stuck on a problem and wondering if you can help?? It is: Prove the following:
sec2(X)+csc2(X) = sec2(X)csc2(X)

Answered by Harley Weston.
A cone in 3 space 2002-03-20
From Matthew:
Let C in R3 be the cone defined by x2 + y2 - z2 = 0

(A) Let P be the plane described by x + 2z = 1

(i) Find a description of P in terms of two parameters s and t

.
.
.

Answered by Walter Whiteley.
F = -3 + 3 2002-03-19
From Jessica:
F= -3 + 3 WHAT IS F?????
Answered by Penny Nom.
The slope of a tangent line 2002-03-04
From Ridley:
Suppose a function f(x) has the line 3x+4y=2 as its tangent line at x=5. Find f'(5).
Answered by Harley Weston.
sin 2x = cos 3x 2002-02-25
From Allan:
solve:

sin 2x = cos 3x

Primary question: how do you handle the cos 3x?


Answered by Paul Betts and Chris Fisher.
The substitution method 2002-02-24
From Joe:
whats the answer to this question? 3x+y=11
x+2y=-3

its substitution method i am having alot of trouble figuring it out. send the answer as soon as possible. thank you


Answered by Penny Nom.
1,4,27,____,_____46656 2002-02-21
From Brenda:
1,4,27,____,_____46656..... please find the fourth and fifth numbers in this sequence.
Answered by Penny Nom.
Nets for pyramids 2002-02-14
From Michelle:
I want to have my students create nets for pyramids and I need to know how to find the correct range of degrees for the interior congruent angles of the isosceles triangular faces. For example, I know for a square-based pyramid that 77 degrees will work; however, I know other angle measures will also work. I'm just not sure how to find the minimum degree measure to have the net actually "work". I'm assuming the maximum would be 89 degrees, although that would make for a very tall pyramid.
Answered by Penny Nom.
Successive differences 2002-01-20
From Linsey:
what is the rule for working out the formula for a sequence with three lines of difference? eg.
                      
                     1, 3, 7, 15, 29 
1st line differences:  2, 4, 8, 14 
2nd line differences:    2, 4, 6 
3rd line differences:     2, 2 

Answered by Penny Nom.
Rolling 5 sevens before rolling a six or an eight 2002-01-20
From Tony:
When rolling 2 dice, what is the probability of rolling 5 sevens before rolling a six or an eight?
Answered by Andrei Volodin and Penny Nom.
My salary is doubled everyday for 30 days 2002-01-17
From Kanishk:
I recieve 1 penny the 1st day, 2 pennies the 2nd day, and my salary is doubled everyday for 30 days. How much money will I have by the end of the 30 day time period? (Is there a way of solving this problem without a chart?)
Answered by Penny Nom.
A sequence 2002-01-16
From Chris:
I have spent two days trying to determine the pattern to the following set of numbers: 1,4,9,1,6,2,5,3,6,4,9,6,4,8,1,____. I need the next four numbers to the sequence.
Answered by Claude Tardif.
Negative exponents 2002-01-16
From Julie:
If z to the power of -4= 1/z4 then how would you solve 1/t-2?
Answered by Penny Nom.
Linear regression 2002-01-16
From Murray:
If you have a set of coordinates (x[1],y[1]),(x[2],y[2]),...,(x[n],y[n]),find the value of m and b for which SIGMA[from 1 to m=n]AbsoluteValue(y[m]-m*x[m]-b) is at its absolute minimum.
Answered by Harley Weston.
Algebra review questions 2002-01-14
From A student:
  1. A wildebeast was observed racing a distance of 86.4m in 4.8 seconds. Find the speed of the wildebeast in m/s.

  2. 8x - 7 - 5x = 23

  3. 7 z - 3 (z + 2) = 26

Answered by Penny Nom.
Adding vectors 2002-01-12
From Lena:
how do you add vectors together?
If you are given the length and angles of both vectors and are asked to add/subtract them, how do you do it? I know you are supposed to do the head to tail method, but whenever i try it i get the wrong answer. I need help setting it up.

example: A is 2.7cm, and 60 degrees, B is 1.6cm and 135 degrees, find the magnitude and amplitude


Answered by Penny Nom.
The tangent function 2002-01-12
From Justine:
if you know that sin45degress = cos45degrees, how do you know that tan45degrees = 1?
Answered by Penny Nom.
A sequence of number pairs 2002-01-10
From Terry:
Three is the first number of a pair, and 8 is the second.

a. If 50 is the first number, what is the second number?
b. If 200 is the first number, what is the second number?
c. If 89 is the second number, what is the first number?
d. If a number n is the first number, what is the second number?

3-8
4-11
5-14
6-17
____
____
____
10-29
____
____
____

Answered by Peny Nom.
Glasses of wine 2002-01-10
From A student:

6 glasses of regular wine (12% alcohol) would equal aprox.? glasses of wine (8% alcohol)

a)9.33
b)8.5
c)7.75
d)9.0


Answered by Peny Nom.
Maple trees and pine trees 2002-01-06
From A student:
A team of scientists found that in a forest there was four maple trees for every nine pine trees.How many maple were there if they counted 45 more pine than maples?
Answered by Penny Nom.
Simultaneous equations 2001-12-17
From Matthew:
4x + y = 4
2x - 3y = 5

what is x and y


Answered by Penny Nom.
A 1 m by 1 m square box in the xy-plane 2001-12-15
From Murray:
Inside a 1 m by 1 m square box in the xy-plane, there are finitely many line segments, whose lengths sum to exactly 10 m. Show that there exists a straight line in the plane which crosses at least six of these line segments. (Hint: first, show that there exists a straight line in the plane which crosses at least five of these line segments.)
Answered by Claude Tardif.
Exponential form 2001-12-11
From Crystal:
Hi, I am asking a question on behalf of a homeschooling grade 11 student. She is having trouble with changing something like (3-4)2 into a exponential form.

First of all she needs to know what the term means "exponential form" and then the process of it.


Answered by Penny Nom.
y = zx - x - c 2001-12-11
From Andrew:
Could you please help me work out the value of x where: y = zx - x - c
Answered by Penny Nom.
Normal lines 2001-12-11
From Kristie:
Why are perpendicular lines called normal lines?
Answered by Chris Fisher.
Two equations in two unknowns 2001-12-04
From Courtney:
y = 3x + 2
y = 4x - 5

solve for x

Answered by Penny Nom.
Solving x^2-3x+5=0 2001-11-30
From Katy:
How do you find the solution to x2-3x+5=0?
Answered by Penny Nom.
Slope intercept form 2001-11-27
From A student:
what is slope intercept form?
Answered by Penny Nom.
Undetermined coefficients 2001-11-22
From Hoda:
The equation is:

y" - 2y' + y = t et + 4

We need to use The method of Undetermined coefficients. I have tried assuming that the solution is Atet+Bet+C, but all I get is C=4 and I tried (At2+Bt+C)et+D, but again I get 0=0 when I calculate the first and second derivatives, so i get no information on the constants. Any suggestions?


Answered by Harley Weston.
Matrix 2001-11-21
From Hoda:
I have a question about matrix multiplication; well, actually, matrix division. I am assuming that matrices are divided the same as they are multiplied; that is, row by column. But what happens if you have to divide by zero? How does this affect the resulting matrix?
Answered by Patrick Maidorn and Penny Nom.
A tangent line 2001-11-21
From A student:
write an equation of the line tangent to the graph of

ey + ln(xy) = 1 + e at (e,1)

Answered by Harley Weston.
Number sequences 2001-11-17
From Stephanie:
My name is Stephanie. My grade 7 math class is studying number sequences and, to me, they make almost no sense at all. Here I wrote down 2 of the many problems I have for homework and was wondering if you could explain them to me (I have to find the next 3 numbers in each sequence.) It would be a big help!

10, 14, 18, 22, _, _, _.

12, 21, 39, 75, _, _, _.


Answered by Penny Nom.
How do you get to 100 by using 6 nines? 2001-11-17
From A student:
How do you get to 100 by using 6 nines?
Answered by Claude Tardif.
Negative numbers 2001-11-15
From Jewel:
If zero means nothing or an empty set, then how can a number be less than zero, as in negative number? My understanding of negative numbers is distance from a set point in a given direction. Thus having a negative and positive side of a line is arbritary and is related to displacement rather than value. Am I in error?
Answered by Penny Nom.
3x-1/2(x-4)+5= -3 2001-11-13
From Tausa:
3x-1/2(x-4)+5= -3
Answered by Penny Nom.
Inequalities 2001-11-13
From Pat:
When writing the solution to an inequality, why is the solution written in set notation? {x/ }
Answered by Penny Nom.
Squares of negative numbers 2001-11-03
From Susana:
I wanted to know if I can square a negative number..?
Answered by Leeanne Boehm.
(-3)to the power 2, divided by 3 2001-10-31
From Raz:
what is (-3)to the power 2, divided by 3=
Answered by Penny Nom.
Is the tree a pine tree? 2001-10-27
From Kate:
Suppose Stock Brokers tell the truth 2/5 of the time, and 1/3 of the trees in a given forest are Pine. If 4 Stock Brokers say that a certain tree in that forest is Pine, what is the probability that the tree is indeed a Pine tree?
Answered by Penny Nom.
Negative times negative is positive 2001-10-26
From Mary:
I have a question about adding and multiplying positive and negative numbers. When we add two negative numbers the answer is negative BUT when we multiply two negative numbers the answer is positive. I don't understand. Why?
Answered by Penny Nom.
Why do you flip the inequality sign? 2001-10-26
From Sean:
I have a question about an inequality problem. Here it is: Solve and graph 5 - 3x => 17. (=> is greater than or equal). Please tell me why you flip the inequality sign when divinding by a negative number.
Answered by Penny Nom.
Lines in the plane 2001-10-21
From Greg:

*If the line with equation 3x-4ky= -16 has y-intercept -2, find k.

*Write the equation of the line passing through (2,5) and perpendicular to the line passing through (2,-4) and (6,1)


Answered by Penny Nom.
Water in a conical tank 2001-10-20
From Sarah:
The problem: Water flows into a conical funnel at a continuous rate of one gallon per minute (One gallon = 231 Cu.In.). The height of the funnel is 5" and the diameter is 8".

The 1st formula: I need to develop a formula that will give the volume, in cubic inches, of the water in the funnel at any time t (in seconds). V = f(t).

The 2nd formula: I need to develop a formula that will give the height of the water in the funnel at any time t (in seconds). h = f(t).


Answered by Penny Nom.
A function or not a function? 2001-10-18
From Christian:
Which are the following are not functions?
  1. y = x2

  2. y = x3

  3. x = y2

  4. y = 3

Answered by Leeanne Boehm.
A line in 3 dimensions 2001-10-17
From Murray:
I'm working on a complicated proof and i need the equation for a line in 3 dimensions.
Answered by Claude Tardif.
Rules of exponents 2001-10-14
From Carissa:
how do you work this out? Investigate the relationship between a,b,c and d if 2a*2b=4c/4d?
Answered by Leeanne Boehm.
Six nines 2001-10-09
From A mom:
My middle schooler (sixth) has to calculate the integers 0-20 using only 6 nines. We have done all but the integer 14. He can not use decimals or double the nine like 99 or 19. the fraction 9/9 is okay. Keep in mind of course the order of operations.
Answered by Claude Tardif.
Zero as an exponent 2001-10-04
From Wendy:
How do I explain to kids that i.e. 5 exponent 0 = 1?

Does 0 exponent 0 = 1?


Answered by Leeanne Boehm and Penny Nom.
(x^2-5x-6)/(x-6) 2001-10-02
From Bill:
given f(x) = (x2-5x-6)/(x-6) find f'(6).
Answered by Harley Weston.
1:5 2001-09-27
From Sandy:
A colleague and I have a disagreement about dilutions. I seem to remember that I was taught (although it was a long time ago) that a RATIO was written thusly: 1:5 , and read " one part to five parts", for a total of 6 parts. On the other hand, a fraction is written: 1/5 and is read" one part in five" for a total of 5 parts. My colleague says that the ratio reads "one part in five" and is essentially the same as a fraction. Which is right? Have I been doing dilutions wrong all this time? I teach Animal Nursing and need to know where I can find a consise answer to such questions for making up medicated fluids, etc.
Answered by Penny Nom.
The triangle inequality 2001-09-27
From A student:
    
           __   __   __ 
Prove that AX + XB = AB for any X on AB.  

Answered by Chris fisher.
Four sequences 2001-09-24
From Nicole:
I have a few questions I couldn't figure it out. So any help will be great to find the next three numbers or letters. Here's the patters...
  1. DEGHNORY, EHILMU, HIILMTU, BEILLMRUY,...

  2. 50, 33, 25, 20, 16, 14, 12,...

  3. 1777, 1795, 1818, 1819, 1820, 1822, 1836, 1837..

  4. A,H,I,M,O,T,...

Answered by Chris Fisher and Penny Nom.
An acute scalene triangle 2001-09-24
From Kim:
Hi- I'm on a mission--I'm suppose to have a group of children write up a hypotheses about the altitudes and angle biscetors of the follwing 3 types of triangle--equilateral, isosceles and acute scalene triangle.

Is there such a thing as a acute scalene triangle and why or why not.


Answered by Chris Fisher.
A linear inequality 2001-09-17
From Eric:
How can I solve |x + y| + |x - y|< =6?
Answered by Penny Nom.
Integers 2001-09-09
From A student:
I am a Grade 9 student from Ontario Canada who has never understood how to do integers very well. I was wondering if you would be able to either send me some sites that will tell me how to calculate them in a simple way or if you would be able to write me back and give me some pointers.
Answered by Leeanne Boehm.
Earthquake warning 2001-09-03
From Dillon:
In the city of Skangai there are 6 million people.Each person has to tell 2 more people that theres going to be an earthquake. After they do they leave the city. There will be no duplications.It takes 3 minutes to tell each person. How long will it take for the whole city to know about the earthquake?
Answered by Penny Nom.
Linear regression 2001-08-01
From A student and a mother:
My mom is taking a correspondence course in Simple Linear Regression and Correlation Analysis and we've been arguing about the relationship between the correlation -rxy and standard estimate of error-Sy.x. I took statistics last year in high school and I remember something about the Sy.x being proportionate to the r. Are they inversely related, directly related, not related, or can only range from 0 to 1.00? Her book doesn't say very much and I believe they are inversely related. She says they're directly related.
Answered by Chris Fisher and Penny Nom.
The radius of a planet 2001-07-30
From Jessica:
A satellite is orbiting the earth at an altitude of 100 miles. If the angle of depression from the satellite to the horizon is 50 degrees, what is the radius (to the nearest mile) of the planet?
Answered by Harley Weston.
The best linear model 2001-07-29
From Rebeca:
I am a student (senior) summer hire working for Dept of the Army Resource Management. I've been assisting the Analyst in developing a model to predict training costs. The technique we used was simple linear regression. Now I've been asked for my opinion in choosing the best model I feel is better. I've chosen #1. Garrison Training but I want a second opinion before I give my choice. I've taken basic statistics last year in high school and will probably be taking more this year. Could you look over the information and give me a reason my choice would not be best? The dep variable-y, is Training costs in dolars per day.

ModelInd Variable- Xabrsy.x
1.Garrison Training126.310.9820.9055117.2
2.Local Training212.161.370.7937142.7
3.Man Rig Area116.382.470.6481175.6
4.Major Training346.124.670.7280167.2

Answered by Andrei Volodin.
A trig identity 2001-07-27
From Jeff:
prove this identity and show steps
tan(x/2+pi/4)=secx+tanx

Answered by Harley Weston.
Matrix reconstruction 2001-07-19
From Guy:
Is there a way to get the sums of rows, columns and diagonals of an n x n matrix to reconstruct the original matrix?
Answered by Walter Whiteley and Patrick Maidorn.
The speed of the boat 2001-07-12
From Sharon:
A motor boat is travelling in a southeasterly direction in water that is flowing from the south at 2km per hour. Show that the speed of the boat is (6 times the square root of 2) km per hour, given that it can travel at 10km per hour in still water.
Answered by Penny Nom.
A phone bill 2001-06-18
From Janet:
What is the formuala to calculate cost per minute?

Here is the data below

# of calls - 238
# of minutes - 443
cost - $70.06


Answered by Penny Nom.
An inequality involving triangles 2001-06-12
From Sandra:
The triangle inequality guarantees that the sum of the lengths of two sides of a triangle is greater than the length of the third. As a consequence, if x and y are legs of a right triangle, with x less than or equal to y, and z the hypotenuse, then x + y is greater than z, so x is greater than z - y. Under what circumstances will x is greater than 2(z - y) be true?
Answered by Chris Fisher and Penny Nom.
Exponential form 2001-06-06
From A student:
Hello can u please answer my question what is the exponent form of 8x8x8x8
Answered by Penny Nom.
The shortest distance from a point to a line 2001-06-05
From Kat:
Find the shortest distance from the given line. Round to the nearest tenth. (-2,-1) and x+5y+20=0
Answered by Penny Nom.
Conservation, consumption and population growth 2001-06-04
From Steve:
I'm trying to quantify the relation between conservation/consumption and population growth. For instance let's consider California:

The 2000 census states that California's population grew from 29,760* in 4-1990 to 33,871 in 4-2000. I want to find r or rate of growth per year. Based on the exponential growth formula for population growth:
.
.
.


Answered by Penny Nom.
Common solution 2001-06-02
From Samantha:
  1. Solve for common solution: x+y=6 2x-3y=2

  2. Solve for y in terms of x: 3x-y=4

Answered by Penny Nom.
New Quebec reform 2001-05-24
From Marie-Andrée:
I am a middle school teacher beginning to adapt to the new Quebec reform. I am regularely doing projects in écologie and géographie in French immersion based on the international schools way of thinking (What form does it have? How does it work? What is its history? How does it change? How is it linking to our lives? What is our responsibility about that fact or that thing?) and I have real difficulty to make it work for math. So now, I am looking for math concepts (8) which would link to the same way of thinking. What are your suggestions?
Answered by Claude Tardif.
25 y^2 + 20 y z - 4 z^2 2001-05-15
From Kimmy:
i don't know how to factor the trinomial 25y2 + 20yz - 4z2. I can get it, but i get messed up on getting the minus 4 at the end. all i get is a plus 4. Can it even be factored.
Answered by Penny Nom.
A pile of sand 2001-05-14
From Gul:
  1. Sand for use on icy roads is stored in a conical pile 14.2 m high and with a base diameter of 34.4 m
    • calculate the volume of the pile

    • if one sander can take 6.9 m of sand, how many sanders can be filled from the pile?

Answered by Penny Nom.
The angles in a triangle 2001-05-11
From Nikki:
Find the measure, to the nearest degree, of each angle of a triangle with sides of the given lengths.

26, 35, 40


Answered by Penny Nom.
Isoscles and scalene 2001-04-17
From Autumn:
explain where the term isoscles and scalene came from?
Answered by Chris Fisher.
Solving some equations 2001-04-16
From Justin:
Solve the following equations for x
  1. x+7=10
    x=

  2. x-7=10
    x=

  3. 3x=7
    x=

  4. x/3=10
    x=

  5. .

  6. .

  7. .

  8. .

Answered by Penny Nom.
Where do the lines y=2x-4 and y=x-1 intesect? 2001-04-06
From Bryce:
solve the following problem by setting them equal to each other. Solve for x and y. Where do the lines y=2x-4 and y=x-1 intesect?
Answered by Penny Nom.
The unit circle and trigonometry 2001-04-05
From Ashley:
"My teacher wants us to find out what a unit circle is, which I found out, a circle with the radius of 1, but the problem is he wants us to show the relationship between the unit circle and the sine(30,45,60 degrees), cosine(30,45,60 degrees),and tangent ratios(30,45,60 degrees). I need help with this and my teacher will not help us out. Thanks very much ...
Answered by Penny Nom.
A sugar cane crop 2001-03-27
From A student:
a sugar cane farmer harvested only 1/4 of his crop. he sold 2/3 of his harvest to the mill. what fraction of the total crop did he sell.

darens recipe for florida fudge calls for 5/8 cup of sugar. if he wants to make only 1/2 of the recipe how much sugar should he use.


Answered by Penny Nom.
Matrices 2001-03-26
From Peg:
What are some applications of matrices, basic trigonometry, and linear systems in the real world? I'm writing and Algebra report about where these topics are used outside of the classroom.
Answered by Judi McDonald and Walter Whiteley.
An elliptic tunnel 2001-03-24
From Janna:
A tunnel is built under a river for a road 12m wide with a 2m sidewalk on either side. The top of the tunnel is semi-elliptical. A local bylaw stipulates that there must be a clearance of at least 3.6m at all points on the road. If the smallest possible ellipse is used, find the clearance at the center of the road.
Answered by Harley Weston.
A famous landmark 2001-03-23
From Corinne:
A family is traveling due west on a road that passes a famous landmark. At a given time the bearing to the landmark is N 62 degrees W, and after the family travels 5 miles farther the bearing is N 38 degrees W. What is the closest the family will come to the landmark while on the road?
Answered by Harley Weston.
Genealogy 2001-03-22
From Brian:
In considering tracing genealogy the problem of a large and growing number of direct (grandparent) ancestors arose. Since the number of grandparents in any given generation doubles, for a person born in 2000 and assuming 25 year generations, in the year 1000 that person would have over one trillion direct ancestors in that year alone. Since estimates I have read place the total world population in 1000 at 250-300 million, the one trillion figure cannot be correct. I must be missing something in my calculations. Can you help provide the answer? While I am unlikely to be able to trace back to 1000, there is reason to believe 1250 is possible. Even in that year there would exist over one billion direct ancestors. That is certainly rather daunting and unlikely to be achieved if correct.
Answered by Penny Nom.
Systems of equations 2001-03-16
From joy:
How do u solve problems using systems of equations?
~ finding x and y~

ex:

26 = 3x - 2y
42 = 4x + y


Answered by Penny Nom.
The substitution method 2001-03-05
From A student:
Solve each system of equations by the substitution method. Show your work.
  1. y = 8
    7x = 1 - y

  2. y = x - 1
    4x - y = 19

Answered by Penny Nom.
Powers 2001-03-04
From A student:
Hey, can you show me how you do ..

(2xy)to the 3rd power (x) to the 2nd power?


Answered by Penny Nom.
Solving Equations 2001-02-23
From Stephanie:
Do you know who came up with solving equations?
Do you have any web-sites that can give me good information on solving equations?
Do you know any history on solving equations?
Do you know what solving equations is used for?
And finally............Sorry about all of the questions :)
Has the form of solving equations changed from the time it came out to now?

Answered by Juci McDonald.
Faces 2001-02-21
From Sandy:
How many faces are there on a sphere?

What are the faces of a cone?

What is the definition of a "face" of a 3D object?


Answered by Walter Whiteley.
The law of cosines in the real world 2001-02-21
From Hope:
Do you have any examles and/or labs that show how the law of cosines is used in the real world?
Answered by Harley Weston.
Law of cosines 2001-02-20
From Emily:
I missed a few days of class and I can't figure out how to solve Law of Cosines problems. I have the notes and formulas but I can't figure out how to do the math involved to answer the problems. I am also getting confused about how to use degrees and seconds in the Law of Sine and Cosine. I can't seem to get the correct answers and I don't really know how to solve them and enter them into my TI 86.
Answered by Harley Weston.
Linear equations 2001-02-13
From Jamie:
I have a question how do you do linear equation?
Answered by Penny Nom.
Two inequalities 2001-02-12
From A student:
If 3
Answered by Leeanne Boehm.
Multiplying one and two digit numbers 2001-02-11
From Marty:
When multiplying do you put the number you are multiplying by on the top or the bottom. Example if you are multiplying by 6's would you write the problem 6
x 2
or 2
x 6
Maybe it just doesn't matter. But when you get to double digits, the double digit always goes on the top?

Answered by Penny Nom.
Kinds of lines 2001-02-09
From Robyn:
What are the different kinds/types of lines?
Answered by Leeanne Boehm.
Partitioning of an arbitrary line segment 2001-02-08
From David:
Did Euclid's Geometry include a construction for the regular partitioning of an arbitrary line segment?
Answered by Chris Fisher.
Bicycles and phone calls 2001-02-08
From Sarah:
  1. A bicycle has a diameter of 66 centimeters. How many times must the tire rotate to travel 1 kilometer?explain answer.

  2. Becky want to make a long distance call to her friend Sarah from a pay telephone.She has $5.00 in change.The call costs $0.90 for the first three minutes and $0.24 for each additional minute.How long can Becky talk to Sarah?

Answered by Leeanne Boehm.
(-2b+3)(-b-1) 2001-02-06
From Melissa:
Alright, lets say you are multiplying (-2b+3)(-b-1) and you are using "FOIL" first you would get : 2b2+2b-3b-3 then you add like-terms. when multiplying,if the bases are the same, add the exponents...does the same thing apply when you are adding the results of the multiplication even though it's addition?

would the answer be 2b3-3b-3 or would it be 2b2-b-3?


Answered by Penny Nom.
Which fraction is greater? 2001-02-01
From Carol:
Hello My name is Carol I am a student teacher! I am asked to aproach these students on their problems. I am unsure of how to explain to them that these are wrong. Can you help? Thanks.

Iris claims that if we have two positive rational numbers, the one with the greatest numerator is the greatest.

Shirly claim that if we have two positive rational numbers, the one with the greatest denominator is the least. Thanks!


Answered by Claude Tardif.
1 + 1 = 1 2001-01-23
From Stephanie:
My friend has this as a bonus question the other day and I want to figure it out. I don't know how 1+1 in any form could equal 1. Please let me know how you come about geting that.
Answered by Claude Tardif.
A problem with distance 2001-01-16
From A student:
For what values of a is the distance between P(a, 3) and Q(5, 2a) greater than the square root of 26.
Answered by Penny Nom.
Polynomials and exponents 2001-01-15
From A student:
I am duing a project in math on polynomials and exponents. I need a real life usage of polynomials and exponents for my project.
Answered by Penny Nom.
Some terms in a sequence 2001-01-15
From jamie:
Please help as soon as possible! i am stuck with some terms in a sequence. the terms are 1st term=1, 2nd term=10, 3rd term=35, 4th term=84, 5th term=165, 6th term=286. can anyone tell me the formula for the nth term? n is term number
Answered by Sukanta Pati.
Exponents 2001-01-14
From A student:
I am wondering if a number raised to the second power is "squared" and a number raised to the third power is "cubed" is there a name for any number raised to any other power.
Answered by Harley Weston.
Vitamins A and B 2001-01-14
From Sara:
A diet is to include at least 140 mg of Vitamin A and at least 145 mg of vitamin B. these requirements are to be obtained from two types of food. type X contains 10 mg of vitamin A and 20 mg of vitamin B per pound. Type Y contains 30 mg of vitamin A and 15 mg of vitamin B per pound. if type X food costs $12 and Type Y $8 per pound, how many pounds of each type of food should be purchased to satisfy the requirements at the minimum cost?
Answered by Claude Tardif and Harley Weston.
Blood flow and age 2001-01-11
From Gayle:
I don't understand the second part of this problem Here is the first part that was correct. A 35yr old has 100% blood flow. A 65yr old has 64% blood flow. Find the rate of change per yr in the percent of blood flow. (This part is correct) m=(64-100)/(35-65)= -36/30= -1.2 per year of blood flow.

Now, here is the second part I DON'T UNDERSTAND! .Let b represent the percent of blood flow and a represent a person's age. Write an equation that you could use to predict the percent of blood flow for a person of any age over 35.


Answered by Penny Nom.
x^5 + y^5 + z^5 = w^5 2001-01-11
From Samuel Brown:
x5 + y5 + z5 = w5 I have no idea whether or not this is possible.
Answered by Claude Tardif.
cot(arcsin 3/5) 2001-01-07
From Jason:
Find value. Assume that all angles are in Quadrant 1.

cot(arcsin 3/5)


Answered by Harley Weston.
The laws of sines and cosines 2001-01-02
From Faydene:
Can the sine /cosine rule be applied to a right -angle triangle to find a particular solution or are these 'rules' applied only when the triangle is not right angled?
Answered by Penny Nom.
Solving x - sin(x) = constant 2000-12-29
From Keith Roble:
If x is in radians, how do you solve for x, where: x-sin(x) = constant?
Answered by Harley Weston.
Calculating the tax 2000-12-22
From A grocer:
I have a small grocery store. I have a bag of money containing $325. This represents the mornings receipts and a tax of 5.75% is included in the total. How can I separate the gross take from the tax take so that I'll know how much is mine and how much is the state's?
Answered by Penny Nom.
Networks of satellites and linear spaces 2000-12-08
From David:
Let us suppose some companies have collaborated to place several satellites in orbit. Let us call the set of all satellites that a given company helped place in orbit a network. Finally let us assume the following 4 rules.
  1. There are at least two distinct satellites.


  2. For each pair of satellites there is exactly one network containing them.


  3. Each network contains at least two distinct satellites.


  4. For each network, there is a satellite not in it.

What is the least number of satellites.

what is the least number of networks?


Answered by Penny Nom.
< and > Which one is which? 2000-12-06
From Alice:
This grandmother forgot and wants to know the correct for greater and the one for lesser.....

the two are < and > Which one is which?


Answered by Penny Nom.
Trig identity crisis 2000-11-29
From Rhiannon:
I have tried many times to find the answer to these problems but I can't I am in grade 12
  1. tan(x)=csc2(x)-cot2(x)
  2. cos(x)/csc(x)-2sin(x)=tan(x)/1-tan(x)
  3. cos(x)[ tan2(x)1-1]/cos2(x)+sin2(x)=sec(x)

Answered by Harley Weston.
Vector Problem 2000-11-27
From Ben:
An aircraft can fly 260km/h in still air and the wind is blowing at 70km/h towards the West. In what direction should the aircraft head so that its actual velocity is on a bearing of 030 degrees?
Answered by Harley Weston.
A piecewise linear equation 2000-11-24
From Jacky:
There is a light bulb and it is given that the light bulb cost $0.75 and the cost of operating it is $0.0081 per hour. From the information give, I came up with the linear equation: Let c be total cost and Let h be hours used. Therefore: c = 0.0081h + 0.75 represents the total cost of the light bulb and the electricity. However, the second part of the question added the fact that the light bulb will only last for 800 hours. If the light bulb is replaced as soon as it burns out exactly after every 800 hours, how can I write an equation that represents that? Is it possible? What would it look like on the graph.
Answered by Harley Weston.
Net, gross and vat 2000-11-20
From Chris:
I have my gross but want to find out the net so I know how much the VAT is?
Answered by Penny Nom.
Solving an equation 2000-11-14
From Heather:
4x-3 = 3x+4 these kinds of problems i dont get but everything else in my math class i have got. i just dont understand it.
Answered by Penny Nom.
Graphing F(x) = 3x 2000-11-06
From Jose:
graph the exponential problem F(x)=3x
Answered by Harley Weston.
Divisibility by 9 2000-10-24
From Kelera:
If the sum of the digits of a number is divisible by 9, then the number itself it divisible by 9. Why is that? How do you explain this?
Answered by Penny Nom.
A sequence question 2000-10-23
From Chell:
I have the following sequence:

4 16 37 58 89 145 42 20

I have no clue how to arrive at an answer. I've checked to see if it can be defined as an arithmetic or geometric series, and it can't. I've looked for primes and noticed that 37 and 89 are the only primes ... but from there I've lost. Nothing I've tried seems to work, so please help! Thanks a million!!!!


Answered by Chris Fisher.
Connecting to a water line 2000-10-20
From Vanja:
My question is...A house is to be connected to a new water main that runs along the line y=2/3x-1. The connection point at the house has coordinates (2,9), where the units represent metres. What lenght of plastic pipe is needed to connect to the water main at the closest point?
Answered by Penny Nom.
Larger and smaller 2000-10-10
From Nicole:
which one of these arrows < , > points to the greater number? which arrows points to the smaller number.
Answered by Penny Nom.
Pillows and Cushions 2000-09-27
From Fiona:

The following problem was given to grade eleven algebra students as a homework assignment. To manufacture cushions and pillows, a firm uses two machines A and B. The time required on each machine is shown. Machine A is available for one full shift of 9.6 hours. Machine B is available for parts of two shifts for a total of 10.5 hours each day.
Answered by Harley Weston.

Isolating an exponent 2000-09-24
From C. Scott:
Suppose you invest $500.00 in an account that pays 10% interest compounded annually. How long will it take for this value to triple?

A=p(1 + i)a A=1500 i=0.1
1500=500(1.1)a p=500 a=unknown

How do you solve this problem algebraicly? How do you isolate the variable (a) when it is an exponent?


Answered by Harley Weston.
Buying art supplies 2000-09-18
From Travis:
An art teacher buys exactly 100 items at this sale, and the total cost before tax is $100. She buys at least one of each item. How many of each item does she buy?

Markers $0.50
Sketch Pads $3.00
Paint Sets $10.00


Answered by Denis Hanson.
Two linear equations 2000-09-14
From David Dean:
2a + 1b = 3.39 3a + 3b = 6.59
What formula do I use to find what a = ?

Answered by Harley Weston.
Manipulating powers 2000-09-13
From Phil:
21/5 x 42/5 = ?
Answered by Penny Nom.
Inequalities 2000-09-02
From Jennifer:
I would like some help with equations and inequalities. My first inequality is 3p-6>21 and 3q+11+8q>99.
Answered by Paul Betts.
How many planes contain a line and a point? 2000-08-30
From Harold:
How many planes contain each line and point?
Answered by Harley Weston.
Three points on a line 2000-08-25
From Casey:
I am trying to find the slope and y-intercept of an equation. I have THREE x values and THREE y values. How do you do it? Help!
Answered by Penny Nom.
Slope 2000-08-19
From John K. DePonte:
My question is to why the variable "m" was chosen to represent the slope of a linear equation. I always told my class to think of "m" for "m"ountain slope. But what is the real reason for this?
Answered by Penny Nom.
Find the next term 2000-08-12
From Ashley:
8,27,64
I need the next three numbers & I can't figure it out. I have worked on this all day.

Answered by Penny Nom.
Some trigonometry 2000-08-11
From Angela:
I have some PreCal questions. I am a student at the secondary level. I would be very grateful for your help.

Solve the equation for theta (0 <= theta < 2pi).

tan2(theta) = 3 I know sec2(theta) -1 = tan2(theta)

.
.
.
Answered by Harley Weston.

A problem with a quadratic 2000-08-09
From David Xiao:
Find the value of a such that 4x2 + 4(a-2)x - 8a2 + 14a + 31 = 0 has real roots whose sum of squares is minimum.
Answered by Harley Weston.
Graphing an inequality 2000-08-02
From Lori:
How do I find x and y and graph this problem

3x + y < 5


Answered by Penny Nom.
Two linear systems 2000-08-01
From A student:
please help me with this homework question. 5/x + 6/y= 19/6 3/x + 4/y =2
hint: let a = 1/x and b = 1/y substitute these expressions into the system to find a and b. Then find x and y.

This is a question I answered wrong on a test: solve the given system for x, y, and z. Express your solutions in terms of a, b, and c.


Answered by Penny Nom.
Making a paper cone 2000-07-30
From John:
The question of how to lay out & cut out of paper, cones came up. I would like the cone have :
A base of 4 inches
and height of
4 inches,
6 inches,
8 inches.

Answered by Harley Weston.
Calculator and telephones keypads phones 2000-06-27
From Thomas Smith and Veronica Yates-Riley:
On a keyboard, why is the number "7" on the top of the keypad as opposed to the number "1" at the top a telephone keypad?
Answered by Penny Nom.
Lines on an oscilloscope screen 2000-06-20
From Janis Watson:
What are the proper names for the vertical and horizontal lines on a graph? I am trying to explain an oscilloscope screen, and I don't know what to call these, except "vertical lines" and "horizontal lines."
Answered by Penny Nom.
A sequence 2000-06-16
From Claire Hall:
Can you find the formula for the pattern 4,12,24,40,60
Answered by AClaude Tardif.
Using the inverse sine function 2000-05-31
From Nelson Rothermel:
This has me completely baffled. I have to use the laws of sine or cosine to find the angles of a triangle when I have 3 sides, so I can't go 180-x-y when I have 2 angles. Now, I have a triangle with values of 3, 7, and 9. Here are the steps I used (A,B,C are angles; a,b,c are opposite sides):

angle A (16.1951 degrees): cos-1*((b2+c2-a2)/(2*b*c))
angle B (40.6011 degrees): sin-1*(b*sin(A)/a)
angle C (56.7962 degrees): sin-1*(c*sin(A)/a)

If you notice, A+B+C does not equal 180. According to the book, A and B are correct, but C is supposed to be 123.2038 degrees. Why doesn't it work???


Answered by Harley Weston.
The equation of a parabola 2000-05-22
From Ian Forsyth:
Given the points A(0,0) B(60,10) C(24,d) find the equation of the parabola. leave the equation in terms of x, y and d if the general form of a quadratic is y = ax2 + bx + c.
Answered by Penny Nom.
Volume of a sphere 2000-05-21
From Kevin Partridge:
Does anyone have a way to physically demonstrate how to explain the volume formula for a sphere? Or perhaps how to derive the formula without calculus?
Answered by Harley Weston.
Radioactive decay 2000-05-18
From Catherine Sullivan:
Please help me with the following: The radioactive isotope carbon-14 is present in small quantities in all life forms, and it is constantly replenished until the organism dies, after which it decays to carbon-12 at a rate proportional to the amount of C-14 present, with a half life of 5730 years. Suppose C(t) is the amount of C-14 at time t.
  1. Find the value of the constant k in the differential equation: C'=-kC
  2. In 1988 3 teams of scientists found that the Shroud of Turin, which was reputed to be the burial cloth of Jesus, contained 91% of the amount of C-14 contained in freshly made cloth of the same material. How old is the Shroud according to the data?

Answered by Harley Weston.
Graphing a linear function 2000-05-17
From Chelsea:
I need help with grahing linear functions.If you could e-mail me back the basics and how tos I would be much appriciative.
Answered by Penny Nom.
A matrix equation 2000-05-14
From A student:
Right now, we are dealing with matrices and we are supposed to solve the following problem on our graphing-calculators: 2a+3b-4c+d=20
a-2b+3c-5d=-14
3a+4b-2c+3d=19
5a-b+6c+4d=-5

Answered by Penny Nom.
Solve 2sin 3x-1=0 2000-05-11
From Cynthia:
How would you solve 2sin 3x-1=0? I don't know what to do with the 3.
Answered by Penny Nom.
Parallel and perpendicular lines 2000-05-08
From Jimmy Lumpkins:
Need a method for solving the following problem: Find the equation of a line through point P that is parallel or perpendicular to another line.
Answered by Paul Betts.
sin(7pi/12) 2000-05-04
From Kristel:
What is the exact value of sin 7pi/12?
Answered by Chris Fisher and Paul Betts.
A colony of bacteria 2000-04-22
From Nicci Demmin:
Suppose that the bacteria in a colony can grow unchecked, by the law of exponential change. The colony starts with one bacterium and doubles every half hour. How many bacteria will the colony contain at the end of 50 hours?
Answered by Penny Nom.
Uniting algebra and geometry 2000-04-16
From Beth:
Who is the mathmatician that united algebra and geometry???
Answered by Claude Tardif.
Lining a cone 2000-04-06
From Jim Campbell:
I am not a student, I am trying to solve a business problem. The question. If I want to put a lining in a chute that is cone shaped, how do I calculate the size steel plate I need to do that. The cone is 10' in diameter at the top and has a 20" hole at the bottom. The total height of the chute is 8'.
Answered by Harley Weston.
y = x^x^x^x... 2000-04-05
From Michael Hackman:
Find the derivative of: y = x^x^x^x... on to infinity.
Answered by Claude Tardif.
Running Through a Train Tunnel 2000-04-01
From Eugene Chan:
A man is running through a train tunnel. When he is 2/3 of the way through, he hears a train that is approaching the tunnel from behind him at a speed of 60 mph. Whether he runs ahead or back, he will reach an end of the tunnel at the same time the train reaches that end. At what rate, in miles per hour, is he running? (Assume he runs at a constant rate.)

I think the answer (12 mph) is wrong. Also, I believe it should read 1/3 of the way through, but don't know how to prove it. Could you come up with some way to prove it, please? I would really appreciate it.


Answered by Penny Nom.
Taxis in Chicago 2000-03-27
From A high school student high school student:
Suppose that taxis pick up and deliver passengers in Chicago, which is divided into three zones. Records kept by the drivers show that of the passengers picked up in Zone 1, 50% are taken to a destination in Zone 1, 40% to Zone 2, and 10% to Zone 3. OF the passengers picked up in Zone 2, 40% go to Zone 1, 30% to Zone 2, and 30% to Zone 3. Of the passengers picked up in Zone 3, 20% go to Zone 1, 60% to Zone 2 and 20% to Zone 3. Suppose that at the beginning of the day, 600 of the taxis are in Zone 1, 100 in Zone 2, and 300 in Zone 3. What is the distribution of taxis in the various zones after all have had two riders?
Answered by Harley Weston.
Divisibility by 3 2000-03-24
From Pat Walsh:
W hy does it work when you add the digits of a number then divid by three to see if the number is divisible by three
Answered by Penny Nom.
Compounding continuously 2000-03-21
From Gina:
You deposit $1500 in an account that pays 6.5% annual interest, compounded continuously. Find the balance after 10 years.

I'm not sure what to do with the "compounded continuously" part.


Answered by Penny Nom.
A system of equations in five unknowns 2000-03-20
From Will:
I have been having some problem with the following question for some time. I would appreciate any help on solving the problem or a solution.

Q: Assume that a system of equations in the unknowns x1, x2, x3, x4 and x5 when converted to row echelon form gives

.
.
.

Answered by Penny Nom.
Completing a sequence 2000-03-17
From David Grimmer:
Complete the following sequence so that, after the first two terms, each successive term is the average of all the preceding terms.

20__ __ 28 __ __


Answered by Penny Nom.
Functions that satisfy f' = f 2000-03-16
From Kevin Palmer :
Recently my calculus teacher asked his students to try and find any functions whose derivatives where the exact same as the original function.

The only function then I have determined that statement to be accurate in is all the natural exponential functions. Ex. f(x) = ex, f'(x) = ex

If possible could you please email me all the functions that you can find in which the original function and its derivative is identical.


Answered by Claude Tardif.
Simultaneous equations 2000-03-11
From Laura Molck:
My name is Laura Molck and I am in Year 11 in Australia. Please help me with the following. I know that they are all simultaneous equations which I can do but I have trouble with the formulae to work the equations. Can you please help!!

1. A tent manufacturer produces 2 models, Outback and Bushwalker. From earlier sales records it is known that 20% more of the Outback model is sold than the Bushwalker. A profit of $200 is made on each Outback sold, but $350 is made on each Bushwalker. If during the next year a profit of $177,000 is planned how many of each model must be sold?


Answered by Penny Nom.
Systems of linear equations 2000-03-10
From Ann Marie Devereux:
hi there!!, I guess I have a problem!!!
  • 3x+4y=10 (over)
    4x+y=9

  • 2x=5y+3 (over)
    x=3y+1


Answered by Penny Nom.
Angle of Intersection of Two Lines 2000-03-02
From Veronica Patterson:
I am having a real hard time trying to figure out this problem. Could you please help me! The homework question says to find the acute angle of intersection between the two lines y=3x+1 and y=(1/2)x-1. (It also says to use the results of a problem I had already figured out.) That problem was to use information from a picture shown that tan(theta sub1-theta sub2)= ((m sub2- m sub1)/(1+(m sub1 * m sub2))). I used the difference identity of tangent to figure out the answer. Any help on this problem would be greatly appreciated.
Answered by Harley Weston.
Saving for college 2000-03-01
From Andrew Kunz:
SAVING FOR COLLEGE In this project, you will forecast a friend's finances. Jane has received $75 from her grandparents on every birthday since she was one year old. She has been saving the money in an account that pays 5% interest. She is saving her money to help you pay for her college education, which she will start this fall after her 18th birthday. She also has been receiving birthday checks from her other relatives, but these didn't start until she was 12 years old. The amounts of these checks from her 12th birthday until her 18th birthday are $45, $45, $55, $50, $55, $60, $65.

How much money will she have saved just from her birthdays by the time she starts college? IS this a reasonable amount to pay for a used car during her junior year in college? If she had invested her money in a different accoutn that had earned 7% interest, how much more money would she have saved?


Answered by Penny Nom.
What's the next term? 2000-02-29
From Meredith O'Neill:
At this point, the whole family wants to know the answer. if possible an explanation. thanks
a sequence ?

16,37,58,89,145 ??


Answered by Chris Fisher.
A negative times a negative 2000-02-29
From Michael J. Butler :
I have reviewed the answer in relation to the question of why (-3)x(-2)=6; however, I am still not able to properly explain the reasoning to my son, Jonathan, who is in grade 7. I want him to understand the reason for the rule that the multiplication or division of two negative numbers equals a positive number. Can you help?
Answered by Chris Fisher.
Slant height of a cone 2000-02-24
From Jocelyn Wozney :
I need help with this problem for my high school calculus class. Any help you can give me will be greatly appreciated-I am pretty stumped. "Express the volume of a cone in terms of the slant height 'e' and the semi-vertical angle 'x' and find the value of 'x' for which the volume is a maximum if 'e' is constant.
Answered by Harley Weston.
More on googols 2000-02-23
From Kevin Brennan:
Could you please give me a couple of examples of when you MATH guys would use a google. Do youu measure, like, galaxies with it ? Or was it created to keep track of Mr Gate's bank account.
Answered by Penny Nom.
Triple angle formula 2000-02-23
From Sara:
Can one derive a triple angle formula for sine and cosine? If so, how?
Answered by Chris Fisher.
A valentines project 2000-02-21
From Courtney Dohoney:
I was assigned a valentines project. Our job is to make a valentines card that has a math theme. On my valentines card I would like to include the set of equations that when graphed look like a heart. If you can figure out a set of equations and get it back to me I would be very happy.
Answered by Harley Weston.
A trig identity 2000-02-17
From Eric:

Question:
How do I solve this problem?

 sin3x   cos3x _____ - _____ = 2 sinx cosx 

Answered by Chris Fisher.
Midpoints and endpoints 2000-02-15
From Jessica Sipes:
I need to know how to find and endpoint using the midpoint and the other endpoint.
Answered by Penny Nom.
Ax + By = C 2000-01-18
From Fatiha Khanboubi:
I was wondering if you could please explain to me how I would write the equation of line through the given points [(4,1), (6,3)] in Ax+By=C form.
Answered by Harley Weston.
An unbelievable story 2000-01-17
From Melissa:
good morning-i am taking a course "math for elementary teachers" and i have a take home problem that i really need help on. here is the question
-the million dollar problem
a recent item in the newspaper described a robbery in which the robber had held up a local bank for 1 million dollars in small bills, had the tellers put the money in a briefcase which he gave them, and fled the scene on foot. is this story believable? explain your answer!

Answered by Harley Weston.
Parallel lines 2000-01-12
From Lori:
Find an equation of the line parallel to the given line containing the given point. x + y = 2; (1,2) We know parallel line have the same slope and we know the answer y = -x + 3.
Answered by Penny Nom.
Parallel planes 2000-01-10
From Hugo Alvarez:
When two parallel planes are cut by a third plane, the lines of the intersection are?
Answered by Claude Tardif.
Phone Number Possibilities 2000-01-09
From Hamilton Weston:
How do phone companies arrive at the possible number of phone numbers that can be generated for customers?
Answered by Harley Weston.
Derive the quadratic equation 2000-01-01
From Thuy:
My teacher asked us this question in which i can't understand when she asked to derive the quadratic equation. What does this mean?
Answered by Penny Nom.
Graph of a sine function 1999-12-23
From Pierre:
Given;
amplitute:1
period: 540
Phase shift: 60 degree,right

I am ask to right the equation: sin 2/3 (value -60degree)

When I am asked to graph the equation, the period is mixing me up.
Answered by Harley Weston.

Logarithms 1999-12-16
From Pierre:
Can you please answer those three question

  1. inverse or the exponential function.

    logx 81 = 4,
    x expo4 = 81,
    x = +or- 4square root 81,
    x = +or- 3
    s.s = {3}
    Why is -3 not acceptable.

  2. .
    .
    .

Answered by Harley Weston.
Sines & cosine laws 1999-12-10
From Pierre Boivin:
Triangle LMN, angle L=71 degree , LM= 7.2 , MN=8.3 , ln= 5.9

The questiion was to find angle M. Using the cosine law I found the answer to be 44 degree. It is also the book answer. Using the sines law I found the answer to be 42.2 degree. why can't I use the sines law.
Answered by Chris Fisher.

Inequalities and absolute values 1999-12-07
From Austin Cline:
Solve for x: The absolute value of x-1 is less than or equal to the absolute value of x-2
Answered by Harley Weston.
Bases other than 10 1999-12-06
From Garret Magin:
We are doing a lesson on numbers of other bases than 10. We are working with binary, octal, and Hexadecimal. I was wondering what is used to represent number of different bases other then 16? Does it just continue on with the alphabet and if so what happens when you get to Z. It would be a help if you could answer this because it is really bugging me. And none of the math teachers at my school could let me know.
Answered by Claude Tardif and Patrick Maidorn.
Systems of equations 1999-12-06
From Roger Hornbaker:
I am having problem figuring out x and y solutions.
  1. 5x + y = 4
    3x - y = 4

  2. 3x + 2y = 6
    - 3x + y = 0

Answered by Penny Nom.
Log Cancel 1999-12-03
From Justin Murray:
Ok I feel like a goof because I know this and I cant think of an answer But I want to isolate the x in the function y=2^x how do I do this?
Answered by Harley Weston.
The elimination method 1999-12-02
From Jennifer:
Could I get an answer to this one:

2x+5y=36
3x+2y=32

I have to use the Elimination method, as I already know how to do Substitution. How do I begin and show my work? I'm attempting to eliminate the values for y.
Answered by Penny Nom.

Zero to the zero 1999-12-02
From A middle school teacher:
I am a middle school teacher who is looking for a precise explanation of why zero raised to the zero power is undefined. I am hoping to get an explanation using something other than the fact that diividing by zero is undefined.
Answered by Penny Nom.
Get rich quick 1999-12-02
From Holly Hutson:
If one cent were placed in an investment that doubles in value every day, what would be the value of the investment at the end of 30 days?
Answered by Penny Nom.
Cos x = -1/2 1999-12-01
From Pierre Boivin:
When I factor[ 2cos (square) - 5cos -3], I get (2cos + 1)(cos - 3). 2cos + 1 = 0, 2cos = -1, cos = -0.5,. Using inv cos on calculator, I get 120 degree related angle.

When I graph I get two values, between 90 and 180 degree and between 180 and 270 degrees. How do I find those two values. How do use 120 degree in relation with the x axis.
Answered by Penny Nom.

A matching quiz 1999-11-20
From William J. Ricciardi:
I gave a matching quiz the other day and one of my students got all 9 incorrect. That got me thinking......What is the probability that someone could get all 9 questions incorrect?
Answered by Peny Nom.
Critical numbers 1999-11-17
From Sarah:
My question is this:

If f(x)=2x3+4x2-9x+8

and the derivative is f'(x)=6x2+8x-9,

how do I find the critical points if f'(x) is not factorable??
Answered by Penny Nom.

Two derivatives 1999-11-16
From Gina Renicker:
The derivative of:

y=e(xlnx) and y=x2arctan(x1/2)
Answered by Harley Weston.

Number sequences 1999-11-11
From Justin Schessler:
I CAN NOT figure out this sequence or how we continue this sequence...

3,2,6,5,15,14,___,___,___
Answered by Chris Fisher and Harley Weston.

Linear models 1999-11-09
From Helen Moreno:
What is a linear model?
Answered by Harley Weston.
Nines and ones 1999-11-05
From Greg Miller:
Using the numbers 1, 1, 9, 9, only once each, how can I create an expression that equals 10 using only the basic functions of addition, subtraction, multiplication, division, and/or parentheses?
Answered by Claude Tardif.
-log(a) 1999-10-22
From Brenda Miskimmin:
I need to know the mercury concentration in mg/L or ng/L for the following:

-log M (Hg) = 8.5

where mw of Hg=200.59

(it's the negative sign in front of log that confuses me).
Answered by Harley Weston.

|6-2x|<4 1999-10-20
From Tchilco:
Solve |6-2x|<4
Answered by Harley Weston.
Broken line graphs 1999-10-19
From Lori Samara, Kristi Cameron and the 4th grade students:
Students in Ms. Cameron's grade 4 class were wondering if a broken line can be used anywhere in the range on the y - axis when creating a broken line graph? They understand that the broken line can be used at the 0 on the broken line but what if there is a large gap in the data from 100 - 200? What do we do? The text book and mathematics dictionary do not answer these questions.
Answered by Penny Nom.
(-5)^2, -5^2 and -(5)^2 1999-10-13
From Jennifer Brown:
What is the difference between the following problems:

(-5)2, -52 and -(5)2

Our text book (Beginning Algebra, fourth edition, published by McGraw Hill, by Streeter, Huthison and Hoetzle) says the second and third problem are exactly the same. I don't see how that can be. Is there a mathematical rule that explains this?
Answered by Penny Nom.

Length of a line 1999-10-10
From Dagmara Sarudi:
My question has to do with the length of a diagonal. This problem came up when I thought about the shortest distance between two points, for example walking from one point to another in my neighborhood. I can choose a zig zag route and assuming the blocks I walk are exactly the same length, it shouldn't matter what route I took, the distance I travel should still be the same when I reached my goal. If, on the other hand I could travel in a diagonal line, the distance would be shorter. But what if, in my zig zag motion, the sections get so small the route approaches a diagonal. Shouldn't it be that each separate section added together equals the value of the two original sides? Or would it suddenly equal the value of the diagonal (which, of course was shorter than the two sides added together)?
What gives?


Answered by Chris Fisher and Harley Weston.

sin x = x/10 1999-10-07
From Amandeep Grover:
Solve the equation sin x = x/10
Answered by Harley Weston.
A googol 1999-10-06
From Kevin Brennan:
describe to me a guegoll is that how u spell it?

it sounds like that
Answered by Penny Nom.

Surface area of a cone 1999-09-18
From Frothy:
I don't understand how to find the surface area of a cone. The height is 12cm and the radius is 5cm.
Answered by Walter Whiteley.
2 to the x and x squared 1999-09-17
From John:
For what values of x is 2 to the exponent x greater than x squared?
Answered by Harely Weston.
Is zero positive? 1999-09-03
From Shah:
Is zero a positive integer?
Answered by Penny Nom.
A double negative 1999-09-01
From Dennis:
If b = -2 what does -b = ? As in (a + 8.5) - [(-b) + |c|] a = 1.5, c = -1.7
Answered by Penny Nom.
A Trigonometry Question 1999-08-28
From Diane Simms:
My question is can the following be factored. I am a teacher who needs the factors to this right away. 2 Sin2X + 2 SinX CosX - 1= 0
Answered by Harley Weston.
Division by a negative 1999-08-19
From Sangeeta B.:
56/-8
Answered by Penny Nom.
Why is slope designated m? 1999-08-18
From Peter Komlos:
Why is the slope of a line is designated by the letter "m"?
Answered by Penny Nom.
Parametric Equations 1999-08-06
From Nicholas Lawton:
Show that an equation of the normal to the curve with parametric equations x=ct y=c/t t not equal to 0, at the point (cp, c/p) is :

y-c/p=xp^2-cp^3
Answered by Harley Weston.

From an airport control tower 1999-08-04
From Pammy:
Hi I am a 30 yo mature age student doing my HSC but am having difficulty understanding this, if you can help me.

From an airport control tower, a Cessna bears 023 degrees T and is 27km away. At the same time, a Boeing 767 bears 051 degrees T and is 61km from the tower. Both planes are at the same height.

i) What is the size of angle ATB?

ii) Using the cosine rule to calculate the distance the planes are apart, to nearest kilometre.

I figured out and drew the triangular diagram but can't figure out the rest and which formula to use. sorry about this, thankyou kindly
Answered by Harley Weston.

Standard Deviation 1999-07-11
From Anthony Fama:
I have seen several answers to this question: If one standard deviation represents 68% of the population, what does two, three, four and five sigma [std deviation] represent? As stated, I have seen several different answers and thus, the impetus for my question.
Answered by Harley Weston.
Sin 4A 1999-06-22
From Ryan Cochrane:
If sinA = 4/5, and A is a first quadrant angle, find sin4A
Answered by Harley Weston.
Slopes of curved lines 1999-06-09
From Stephen Ehrler:
When one plots the graphs of y=2x, y=3x, y=xx When each of these graphs pass through point (0,1) do they have the same slope? I know they are different lines but is it possable that they have the same slope at point (0,1).
Answered by Harley Weston.
Graphing a linear equation 1999-05-27
From Dylan Bradley:
Im in Grade ten and am doing linear equations, I cann't figure out how to make a chart for questions like y = -2x + 3
Answered by Penny Nom.
Parallel and perpendicular lines 1999-04-23
From Crystal Pilling:
My name is Crystal Pilling and I am in 9th grade algebra. We are currently studying parallel and perpendicular lines. I am having trouble with this problem: 3/4x - 5y= 16, (5,-6) I have to find a line that is perpindicular to this line on a graph.
HELP ME PLEASE!!!!!

Answered by Penny Nom.
Shopping at Wegman's 1999-04-22
From Stan:
Joanne, Steve, Pat, Alice, Joan, and Bill go to Wegman's at the same time. Joanne buys 2 gal of milk, 1 dozen oranges, 8 apples, and 2 lb. of ground beef, paying a total of $13.24. Steve buys 3 qts. of milk, 5 lbs. of ground beef, 10 lb. of potatoes, and 2 bags of mixed vegetables, paying a total of $16.95. Pat buys 3 gal. of milk, 2 dozen oranges, 1 dozen apples, 5 lb. of potatoes, and 5 bags of mixed vegetables, paying a total of $25.09.
....
Find the cost of: 1 qt. of milk, one orange, one apple one lb. of ground beef, one lb. of potatoes, and one bag of mixed vegetables.

Answered by Penny Nom.
y=mx+b 1999-04-15
From Selena Royle:
When solving a problem on slopes,and in the formula y=mx+b what do the M and B stand for? Why M and B?
Answered by Penny Nom.
The Binomial Theorem for rational exponents 1999-04-15
From Angela Evans:
The full question is this: Isaac Newton generalized the Binomial Theorem to rational exponents. That is, he derived series of expansions for such expressions as (x+y)-3 (x+y)2/3 (x+y)5/6 What did Newton find? What are the first four terms of the series expansions of binomials above? How can this extended Binomial Thrm. be used to aid in calculations?
Answered by Penny Nom.
Dividing a Circle 1999-04-12
From Mike Kenedy:
I am having trouble with a homework question for bonus marks. A Circle is continually divided by lines that do not intersect the center so that they produce the most pieces of circle. For example
  • 1 line divides the circle into 2.
  • 2 into 4.
  • 3, however into 7.
  • 4 into11
  • 5 into 16
  • 6 into 22
  • 7 into 29
  • 8 into 37
  • etc...
I am stumped and cannot figure out the equation, though I'm sure it involves squares. Can you help?

Answered by Penny Nom.
Linear programming and optimization 1999-04-09
From Shams:
What is Linear programming and optimization?
Answered by Jack LeSage and Penny Nom.
Large Numbers 1999-04-05
From CK:
Hello. I need a lesson plan for grades 4 to 7 dealing with the concept of large numbers. Specifically, how to teach scientific notation. The lesson plan has to deal with the following: the size of the universe is so huge that is is almost beyond the ability of the mind to comprehend. One way is to measure...
Answered by Chris Fisher.
0/11 1999-03-09
From Jacob:
What is the term applied to 0 divided by a number.

In calculating slope problem my answer was 0 divided by 11. I don't know what to do with this answer or how to interpret it.
Answered by Jack LeSage and Penny Nom.

An airplane problem 1999-03-08
From B.M.R.:
A plane left New York and headed East to its destination 3600 miles away across the Atlantic. On the way back its speed was boosted by a 50 mph tail wind and it arrived an hour early. What was its normal speed?
Answered by Jack LeSage.
Reversing the direction of an inequality. 1999-03-06
From Mallory White:
If the Problem was -4a plus -5 is less than or equal to 14, why would you change the sign to greater than or equal to?
Answered by Jack LeSage and Harley Weston.
Divisibility by 9 1999-02-21
From Razzi:
I've been having a hard time trying to solve the following problem and I was wondering if you could help me.

For any positive integer a let S(a) be the sum of its digits. Prove that a is divisible by 9 if and only if there exist a positive integer b such that S(a)=S(b)=S(a+b).
Answered by Chris Fisher and Harley Weston.

Four Corners Maths Problem 1999-02-16
From Helen Williams:
I am currently a student teacher in the UK and I have to write a 1000 word report on the following maths problem which I am completely stuck on! PLease HELP!! Choose and 3 by 3 section of the hundred square. Add the total of the four corners. How many different groups of four numbers can you find that add up to that number? eg,
123
111213
212223
Total of 4 corners add up to 48.

Adding 2, 13, 22, 11 also make 48 etc..

How many different groups of 4 numbers would add up to 48?

How would these results compare with thoses obtained from a 3 by 3 square in which the numbers are consective? eg,
123
456
789

PLEASE HELP AS I AM COMPLETELY STUCK? WHY DO ALL THESE DIFFERENT WAYS ADD UP TO THE SAME NUMBER??
Answered by Harley Weston.

Non-Euclidean Geometry 1999-02-10
From Robert Smith:
Is non-euclidean geometry necessary for the college bound student? I have students that are inerested in teaching math one day. My school is restricted to Euclidean Geometry.
Answered by Walter Whiteley and Jack LeSage.
Primary Mathematics 1999-02-08
From Syairul:
Mr Wu earned $2365 in January. He earned $375 more in February than in January. If he spend $4250 in the two months and saved the rest, how much did he save?
Answered by Jack LeSage.
Lunes 1999-02-04
From Kai G. Gauer:
A prof once told me that a certain type of lune is quadrable given that the diameter is an integer. She used the construction of a right isosceles triangle within a semicircle and later constructed another semicircle on the base of the first semicircle and used area subtraction to show equality to a smaller triangle with quadrable area. What happens when the original inscribed triangle is no longer isosceles? She mentioned something about other lunes also being quadrable; but not all. What are the dimensions of other such lunes? Note: I'm not certain if I still have my hercules account; please simply post on Q&Q.
Thanks!

Answered by Chris Fisher.
Money and Counting 1999-02-03
From Deborah Goodman:
I am looking for ways to tutor my daughter in money counting and handwriting skills.
Answered by Jack LeSage.
Fitting a Curve 1999-01-19
From Kirk:
Hello my name is Kirk from Scarborough, Ontario.

I have been out of a formal education system for thirty years. I program microcontrollers in my spare time. I have built a temperature sensing device ready to go but, thermistors are very non-linear. I do know that there is a way to calculate the input condition of the thermistor and display the correct temperature in degrees C. I am sending a file to show my progression so far.
Answered by Harley Weston.

Linear Equations 1999-01-16
From Casie Tomney:
My dad and I have been trying to figure, how to solve for x on any of these problems my teacher gives us. The problem is:

2h/3 + 1/2 = 5h/6 - 3/4

Thanks!!!!!!!!
Answered by Jack LeSage.

Intersection of Planes 1998-12-03
From Lindsay Fear:
My name is Lindsay Fear. I am an OAC student (which is the Ontario equivalent to Grade 12 in most other states and provinces). I am in an Algebra and Geometry course and am currently studying a unit on equations of planes. Our teacher has given us this question that my friend and I have attempted several times, but we are still unable to solve it. My teacher has also suggested using the internet as a resource. The question is:

Prove that a necessary condition that the three planes

 -x + ay + bz = 0 ax -  y + cz = 0 bx + cy -  z = 0 
have a line in common is that
a^2 + b^2 + c^2 + 2abc = 1

Answered by Walter Whiteley.
-(4)^0 = ? 1998-12-02
From Derek Yau:
I would like to know why any number to the exponent 0 equals to 1 also, I would like you to please answer this question: -(4)^0 = ? I know the answer is 1 but I am confused as to if it is a negative one or a positive one.
Answered by Jack LeSage and Penny Nom.
Algebra 1998-11-25
From Casey:
yeah my son brought home a math worksheet and he was wondering and me too if u could help him and me out . he has problems like this 4n+9=7n+2= ??? and he has to show work so if u could help us out that would be great
Answered by Jack LeSage.
Intersection of planes 1998-11-22
From Dave Rasmussen:
I am a teacher of secondary mathematics with a question about the uses of Three Dimensional Co-ordinate Geometry. I have been teaching my students to write equations of planes and lines, - to find the intersection of these and the distance between them. What I am having difficulty finding are good applications of these techniques to "real world" situations. Can anybody help me?
Answered by Walter Whiteley and Harley Weston.
Linear systems of equations 1998-11-16
From Crystal Girvan:
My name is Crystal Girvan.Im in grade 11. I have a question.We are solving linear systems of equations: comparison method. My questions is. it says

a)use the method of comparison to solve this system a=b-1, 3a+ b=3

b) verify your solutin in a

I dont understand it. please help
Answered by Jack LeSage.

What's the next term? 1998-11-12
From Ilia:
What the formula and explonation for formula for next patterns:

1) 1, 4, 10, 20, 35, 56, 84,...

2) 0, 6, 30, 90, 210, 420,...

Thanks!
Answered by Penny Nom.

A Linear System 1998-10-08
From Jason Bussey:
The linear systems with the following pattern all have the same solution. Is their some kind of reasoning as to why this is so?

2x+3y=4
3x+4y=5

4x+5y=6
5x+6y=7

The solution is always x=-1, y=2

Thank you
Jason
Answered by Penny Nom.

Operations Research 1998-10-08
From Lisa Barrett:
What is the history of operations research and the study of linear programming?
Answered by Judi McDonald.
French Francs 1998-10-01
From Richie Franklin:
1.what did one dollar equal in France francs ten years ago, twenty years ago, and fifty years ago?
Answered by Chris Fisher.
Dividing a Class 1998-10-01
From Tom Barker:
My eighth grade niece called with the following homework problem:

A teacher wanted to divide her class into equally numbered groups. She tried to divide the class into groups of two, but was one student short. She tried to divide the class into groups of five, but was one student short. She tried to divide the class into groups of seven and was successful. What is the least number of students that were in her class?

I know the answer is 49, but don't know how to prove it. I must be getting old if I can't solve eighth grade math problems. Your assistance would be appreciated.
Answered by Penny Nom.

A Horizontal Line 1998-09-23
From Tess Warer:
Find the equations of a line through (-1,-5) and parallel to the X-axis.
Answered by Harley Weston.
Concurrent Lines in a Triangle 1998-08-10
From Chris Woolf:
The question is Name four types of concurrent lines, rays, or segments that are associated with triangles.
Answered by Chris Fisher.
Solving Quadratics 1998-07-16
From Fred Goodrich:
I need a step by step lesson on solving quadratic equations.

Thank you,
Answered by Penny Nom.

15 to the zero 1998-06-24
From Mike Husken:
My daughter has the problem: 15 with an exponent of zero=1. Could you help explain why? I would appreciate an answer soon as she has class tomorrow.
thanks,

Answered by Penny Nom and Patrick Maidorn.
A Logic Problem 1998-06-07
From Anthony Bacigalupo:
My name is Anthony Bacigalupo and I take Sequential ][ Math and am taking a practice regents. When doing a logic problem, I encountered the following statements, where I am trying to prove P ( I left out steps unrelated to the question)....
Answered by Chris Fisher.
A Place Value Curiosity 1998-05-25
From Ed:
I was visiting with an elderly gentleman this afternoon. He showed me this curiosity and then asked if I could explain it to him. Can you provide an explanation of why the 9 or multiple of 9 keeps occurring in this procedure? Choose any number, say 125 and add the digits to get 8. subtract the 8 from the 125 and the result is 117. Add the digits in 117 to get 9. Subtract the 9 from the 117 to get 108. Add the digits in 108 to get 9. If this procedure continues a 9 or a multiple of 9 reoccurs. What is the mathematical explanation behind this happening?
Answered by Denis Hanson.
Percentage Question 1998-05-18
From Eddie Knox:
Example: If I have a recipe calling for 16 ounces of 4% vinegar (vinegar which has been diluted with water to a strength of 4%), and all I have on hand is 16 ounces of 5% vinegar, how much water should I add to decrease the strength to 4%, before measuring the 16 ounces to be used.

I'll appreciate any insight and help.

Thank you so much!
Answered by Harley Weston.

Graph question 1998-05-12
From Rose Seminary:
Why is the point of intersection of two lines the solution to the corresponding system of equations?
Answered by Penny Nom.
Multiplying by Nine - Chismbop Style 1998-04-27
From Noria Jones:
About a year ago a grade 5 teacher at my son's school taught the children how to multiply the 9 times table on their fingers quickly.

It was part of a kind of finger math kind of thing...
Answered by Patrick Maidorn.

Seed Problem 1998-03-26
From Jody A. Johnson:
Seed Problem: The height of an amazing plant grown from a seed varies linearly with the time it has been in the ground. After six days it is 6cm tall and after 60 days it is 15.8m (1580 cm) tall.
  1. Write the particular equation giving the height as a function of time.
  2. Find the intercepts and explain them.
  3. What would be the height in one year?
  4. How long would it take for the plant to grow to a height of 1km?
I'm having a problem with this equation. Could you please help. Thank you.

Answered by Penny Nom.
Y-intercept 1998-02-27
From Don Trumpet:
What is the Y-intercept for the problem:Y = -2(x+2) + 9
Answered by Harley Weston.
Pay Phone Problem 1998-02-26
From Shameq:
Hi, I've been given a problem that I'm having some trouble with. I'd really appreciate any help. Here's the question (it's called the Pay Phone Problem)

A pay phone will take only 10p, 20p, 50p, and £1 coins"(It's British).

A woman has plenty of 10p and 20p coins. She has no other coins. She can put the coins into the pay phone in any order.

INVESTIGATE the number of different ways, she could put the 10p and 20p coins into the pay phone.
Answered by Penny Nom.

Tangrams 1998-02-24
From Allison:
Do you know the story behind the tangram? I know that it is a Chinese puzzle about 200 years old. If there is a story about its creation, I would love to share it with my 7th graders. Thanks.
Answered by Diane Hanson and Penny Nom.
(50^20)(20^50) 1998-02-24
From fion:
50 power of 20 X 20 power of 50?

How many zero can be found in the answer and why?
Answered by Penny Nom.

Intersecting Lines. 1998-02-10
From Erik Heppler:
My typewriter frequently makes mistakes. Suppose I gave you the following system of equations to solve:

35.26X + 14.95Y = 28.35
187.3X + 79.43Y = 83.29

When I looked back, I realized that I meant to use 14.96 as the "Y" coefficient in the first equation instead of 14.95. Solving the system using 14.95 and then solving another system using 14.96 with all other values the same both times results in the intersection points (1776, -4186) and (-770, 1816) respectively.
How can that be?
Answered by Penny nom.

Calculating the Intersection Point. 1997-12-10
From Milton Parsons:
Given 2 lines through (x1,y1),(x2,y2),and (x3,y3),(x4,y4). Can you give me a good formula for calculating the intersection point. I would greatly appreciate it.Thanks.
Answered by Chris Fisher and Penny Nom.
Negative Primes. 1997-11-10
From Leah Zucker and Paul Michael:
I have received the following question via e-mail from my granddaughter: "Can negative numbers (like -7) be prime? If not, why not?"
Answered by Chris Fisher.
Some Calculus Problems. 1997-10-30
From Roger Hung:
  1. What real number exceeds its square by the greatest possible amount?

  2. The sum of two numbers is k. show that the sum of their squares is at least 1/2 k^2.

  3. .
    .
    .

Answered by Penny Nom.
Diagramming Powers 1997-10-15
From David Fill:
I am a teacher in Massachusettes. We have been diagramming numbers such as two to the third (a 3-D cube). One of my students asked me how you would diagram two to the fourth. I have searched through all of my teachers books and cannot seem to find the answer to this question. Is there a way to diagram this? If there is, how would you do this?

Your help would be greatly appreciated.
Answered by Penny Nom.

How many intersections? 1997-10-08
From James:
(a) A collection of eight points, no three collinear. If lines are drawn between each pair of these points, how many points of intersection would there be?

(b) what would your answer have been in part (a) if there had been n points to start with?
Answered by Chris Fisher.

Cos(x) Cos(2x) Cos(4x)=1/8 1997-09-24
From Tan Wang:
How many distinct acute angles x are there for which cosx cos2x cos4x=1/8?
Answered by Chris Fisher Harley Weston and Haragauri Gupta.
A Trigonometric Limit 1997-09-18
From Brian Ray:
What is the limit, as x approaches 0, or tan^23x/x^2? (read, tan squared 3x over...)?
Answered by Harley Weston.
Two Questions 1997-08-28
From Faye Tan:
I am a trainee teacher. I would appreciate it very much if you could help me solve the following two problems which I think are meant for students at middle grade.

1. If a hen and a half lays an egg and a half in a day and a half, how many hens are needed to lay a dozen eggs in one day?

2. There are fewer than 200 passengers on a train. If they get off in pairs, one passenger will be without a partner. If they get off in groups of 3 or 4, there will still be one passenger left by himself. However, if they get off in groups of 5, no one will be left by themselves. How many passengers are there on the train?
Answered by Penny Nom.

Finding the Mine 1997-06-23
From Billy Law:
Tom is gold prospector. On his last trip out from town, he headed 35 degree South of West to a lake where he had lunch. The lake was 24 km out of Town. He then headed due East for 35 km before Doubling back on bearing of 15 degree South of west for 20 km to reach his mine. By converting to Cartesian coordinate before doing vector additions do the following:

a) Calculate the position of the mine from town in term of a distance and a direction.
...
Answered by Harley Weston.

A problem with arccos. 1997-06-09
From Vanessa Chan:
Prove: arc cos4/5 + arc cos (-5/13) = arc cos (-56/65)
Answered by Harley Weston.
The General Equation of a Parabola 1997-05-28
From Michelle:
My name is Michelle and I am a 10th grade student in algebra 2 w/ analysis. I am doing a report on parabolas and I need to know what the general equation is. I've looked in books and keep finding different ones! I also need to know how they can be used in nature.

Thank you so much for your time. I really appreciate it!

- Michelle
Answered by Harley Weston.

Formulae for Surface Area. 1997-04-28
From Gary Millward:
I'm trying to help my son with his Math homework (Grade 10) and he has to find the surface area of a cone and rectangluar pyramid. We have the formulas for the volume of these solids, but can't seem to locate the formulas for surface area.
Answered by Walter Whiteley.
Natural Logarithm Functions 1997-04-23
From Rickson:
The following two questions are some of my son's homework that he is having trouble with......any advice or assistance would be appreciated.

(eX)5=1000.............the X and 5 are exponents

lnx + ln(x+3) = ln10

In each question the problem is to find x.
Answered by Harley Weston.

Equation of a line 1997-04-08
From Susan Gregson:
I am a secondary school teacher. My students and I would like to know why the letters m and b are traditionally used to stand for slope and Y-intercept in the standard form of an equation. Was this an arbitrary choice? Who made it? Are the letters from Greek ot Latin words?
Answered by Harley Weston.
Graphing Inequalities of Conic Sections 1997-03-24
From James Sheldon:
I'm trying to graph Systems of Conic Sections with inequalities, but I'm running into problems on which area to shade:
x^2+y^2 is greater than or equal to 16
xy > 4

So I graph these two equations, and then my teacher said to substitute a point into it but I'm still not sure how to do it...
Answered by Penny Nom.

Ajax, Beverley, Canton and Dilltown 1997-03-14
From S. Johnson:
The following towns are placed on a coordinate system. Ajax at (-x,z), Dilltown at (-10,0), Canton at (0,0) and Beverly at (0,10). The roads from Beverly to Canton and from Canton to Dilltown are perpendiculat to each other and are each 10 miles in length. A car traveling at all times at a constant rate, would take 30 minutes to travel straight from Ajax to Canton, 35 minutes to travel from Ajax to Canton via Beverly, and 40 minutes to travel from Ajax to Canton via Dilltown. What is the constant rate of the car, to the nearest tenth of a mile per hour.
Answered by Chris Fisher and Harley Weston.
A trig problem 1996-12-13
From S. Johnson:
sin t + cos t = 1/5. Find ALL exact values of cot t, given the original equation.
Answered by Harley Weston.
Smith, Rodriguez and Jones 1996-11-07
From Rafayel Ambartsumyan:
On a train, Smith, Rodriguez, Jones are the fireman, brakeman, and engineer, but not in that order. Also aboard the train are three businessman who have the same names, a Mr. Smith, a Mr. Rodriguez, and a Mr. Jones. ..... Who is the engineer?
Answered by Penny Nom.
Height of a Hotel 1996-11-07
From Irene:
"Irene" is to determine the # of floors in a hotel 500 feet up the street. Irene is on the 10th floor of an office building and can measure the angle of elevation to the top of the hotel, 57 degrees. Her view of the entire building is obstructed. If the street rises at an angle of 8 degrees from the office building to the hotel and the average distance between floors is 11 feet, how many floors are on the hotel?
Answered by Penny Nom.
Could you tell me the name for the bar in a division problem? 1996-10-21
From Linda:
Could you tell me the name for the bar in a division problem. Not the line with dots on either side but the line that divides the two numbers? My name is Linda. I am asking for my niece who is in 8th grade.
Answered by Chris Fisher.
How do you raise a number to an imaginary/complex power? 1996-07-03
From Andy Golden:
How do you raise a number to an imaginary/complex power? I know how you raise "e" to a complex power, like e^(pi*i): cos pi + i * sin pi But what about numbers other than "e"? What if I want to raise 5 to the 2i power? How is that done?
Answered by Chris Fisher.
a to the power zero 1996-06-03
From Beti:
Why is a number to the power 0 equal to 1?
Answered by Harley Weston.
Prove that these two lines are parallel. 1996-05-29
From Adam Piekarzewski:
Using parallel line theorems and side angle side etc. in triangle acd a line is drawn from point b on ac and point i on ad. another triangle fliped vertically and joined at d is named fdh, a line is drawn from point e on fd and point g on fh. ab = fg, ai = fe, ac || fh prove that bi || eg /- -/
Answered by Harley Weston.
How do you graph these inequalities? 1996-05-17
From Robert:
How do you graph the inequalities 5+gg, and g+h>6 on one coordinate plane?
Answered by Harley Weston and Maxine Stinka.
Cardano's method 1996-03-11
From Mike Gorski:
What is Cardano's method for finding the roots of a cubic equation?
Answered by Harley Weston.
A trig identity 1996-03-11
From Azmat Hussain:
Is there an easier/another way to prove the trig identity cos(a+b) = cos(a)cos(b)-sin(a)sin(b)?
Answered by Penny Nom.
Exponents 1996-02-25
From Eric Cote:
I want to know why the grade six math books tell you that the little number at the top right of another number (exponent) is called the power, but the grade seven and up books tell you it's called the exponent?
Answered by Harley Weston.
Neighbourhoods 1996-02-14
From Robert:
What are neighbourhoods?
Answered by Harley Weston.
(-3)x(-2) 1995-10-25
From Azmat:
Why is (-3)x(-2) = 6?
Answered by Herley Weston and Ed Giesbrecht.
Une sortie 2011-08-26
From SANGES:
un groupe d'amis souhaite organiser une sortie. si chacun donnait 14 euros, il manquerait 4 euro. Mais si chacun donnait 16 euros, il y aurait un excédent de 6 euro. combien y a-t-il d'amis dans le groupe ?
Answered by Claude Tardif.
une suite 2010-07-01
From djodilune:
Salut tout le monde ! J'ai un probleme de math dans un concours de Medecine , Le voilà :

n £ N* on a : S = 1/[n(n+1)] + 1/[(n+1)(n+2)] + ... + 1/([2n(2n+1)] .

reponse a : S= 1 - 1/n
Reponse b : S= 1 - 1/(2n)
Reponse C : S= 1 - 1/(n+1)
Reponse D : S= (1/n) - (1/2n )
Reponse E : S= (1/n) - [1/(2n-1)].
Si vous pouvez me donner le reponse avec justification pour comprendre bien. Merci d'avance

Answered by Claude Tardif.
Résolution d'une équation 2010-02-25
From sellin:
Il faut que j'exprime x en fonction de m dans l'équation suivante et je n'y arrive pas: -3x + 1 = (m-1) / 2 * x + (2m + 3) / 5
Answered by Claude Tardif.
Volume d'une sphere entre 4 autres spheres 2010-01-18
From Bonfils:
Bonjour,

Je souhaiterais en fait calculer le rayon de la sphère max qui pourrait s'insérer entre 4 sphères de rayon connu. Merci par avance pour votre réponse. Sincères salutations

Answered by Chris Fisher.
Une piscine 2009-11-04
From kemayou:
Dans un de ces problèmes,ma fille se trouve face à cette question,et je ne parviens pas à l'aider,pourriez-vous nous donner un petit coup de pouce? Voici l'énoncé:Un jardin a 22m de long et 8m de large.Au centre du jardin,on a fait construire une piscine dont le périmètre est de 44m.La surface restante est couverte de gazon. La question est :exprimer en fonction de x la longueur de cette piscine. Dans l'attente de votre réponse,recevez nos salutations distinguées. Merci.Mme kemayou
Answered by Claude Tardif.
Le volume d'une boule 2009-10-22
From ve.lu.ch:
Pour calculer le volume d'une boule, il faut utiliser la formule :

4/3 x Pi x R au cube

Pourquoi 4/3 ?

Merci d'avance

Answered by Pierre-Louis Gagnon et Claude Tardif.
Un bénéfice de 5% 2008-03-11
From mahiques:
Un propriétaire decide de vendre deux parcelles de terrains pour un montant total de 141750 F. Il fait un bénéfice de 15% sur la première et une perte de 10% sur la deuxième. L'ensemble de la transaction lui a rapporté un bénéfice de 5%.combien a-t-il vendu chacune des parcelles ?
Answered by Pierre-Louis Gagnon.
Combien y a t il d amis dans le groupe 2008-02-25
From vaucher:
un groupe d amis souhaite organiser une sortie si chacun donnait 14 € il manquerait 4 € mais si chacun donnait 16€ il y aurait un excedent de 6 € combien y a t il d amis dans le groupe
Answered by Maxime Fortier Bourque.
moyenne statistique 2008-02-16
From jossan:
Annexe
nombre d'habitants Nombre d'agglomération centre de classe
3 ; 3.5 11 3.25
3.5 ; 4 16 3.75
4 ; 4.5 7 4.25
4.5 ; 5 10 4.75
5 ; 6 12 5.5
6 ; 8 10 7
8 ; 10 9 9
10 ; 15 13 12.5
15 ; 20 6 17.5
20 ; 32 5 26
Total 99
question : compléter la deuxième et troisième colonne (c'est fait) puis en utilisant la valeur centrale des classes calculer le nombre moyen d'habitants . arrondir la valeur à l'unité. on peut utiliser uniquement les fonctions statistiques de la calculatrice et écrire directement la valeur de N

Answered by Pierre-Louis Gagnon et Antoine Letarte.
Je voulais savoir d'où provient le signe # ?! 2008-01-17
From Dimitri:
Je voulais savoir d'où provient le signe # ?! On l'appelle diez, sur les téléphones surtout, mais j'aimerai savoir plus précisément à quoi sert ce signe (autre que sur nos appareils) ! Et pourquoi, en anglais, s'il est placé avant un chiffre, on dit 'number' (par exemple #1, 'Number one'). Son origine m'interesse tout autant que son utilité !
Answered by Claude Tardif.
racine carre de 9 2008-01-01
From hichem:
racine carre de 9
Answered by Claude Tardif.
A=B+C+D+F+G+H+J 2007-11-27
From Boubendir:
voila ma question et comme savoir les nombres additioné en etulisant le resultat

A=B+C+D+F+G+H+J

au debut ons a les B,C,D,F,G,H,J on connaissent leur valeur comment retrouver valeur on etulisant leur resultat

Answered by Claude Tardif.
Multiplier et soustraire des nombres négatifs 2007-01-11
From Sylvie:
Voici trois nombres entiers: (-5), (-3) et (-4). On retranche le troisieme au produit des deux premiers, ecris la situation et donne le resultat.
Answered by Claude Tardif.
Origine du signe racine carré 2006-11-30
From Kévin:
Je suis simplement un élève de 3eme très curieux et j'aimerais savoir d'où vien le signe de la racine carré (l'espece de V) c'est une question toute bete mais ma prof de math ne peu me répondre le pouvez vous?
Answered by Claude Tardif.
volume d'un cône 2006-11-29
From Emmanuel:
On peut diviser un cube en 3 pyramides et ainsi en déduire le volume d'une pyramide. Mais comment peut-on déduire le volume d'un cône de celui d'un cylindre? Comment visualise-t-on la division d'un cylindre en trois?
Answered by Claude Tardif.
Taux à déterminer 2006-11-01
From Barrault:
Une certaine année,un article augmente d' un certain taux "t" au premier semestre puis d' un taux triple du premier au second semestre, sachant que l' augmenation globale sur l' année est 66.75%; uels sont les taux pour chacun des deux semestres de l' année?
Answered by Claude Tardif.
l'explication de 1 - 1,1 exposant -5 2006-08-12
From Karin:
Ma question est l'explication de 1 - 1,1 exposant -5
Answered by Claude Tardif.
Combien a t-il rapporté dans une journée ? 2006-06-21
From Madame Moro:
On lui donne un prix au km soit 1630 €/km, Marc a réalisé 211 m dans la journée, Combien a t-il rapporté dans une journée ?
Answered by Claude Tardif.
Volume d'un cône 2006-04-24
From Christelle:
Caroline décide de se servir de ses connaissances mathématiques pour "rouler" son petit frère: "Arthur, dit-elle, je te propose que nous mettions chaucun un euro dans l'achat d'un cône glacé. Je mangerai la première, jusqu'à mi-hauteur, et toi, tu auras la seconde moitié." Combien la part de Caroline représenterai-elle par rapport à celle de son petit frère s'il acceptait ?
Answered by Claude Tardif.
probleme sin cos 2006-02-19
From Thibault:
mon probleme commence par: f(x)=sinx (sinx+1)+ cos²x
donc en le dévellopant on trouve: f(x)= sin²x + sinx + cos²x
et apres ce que je ne comprend pas est que par la suite on trouve: f(x)= 1+sinx
qu'est-ce qui fait que l'on trouve ce resultat??

Answered by Claude Tardif.
exposant négatif 2006-01-23
From Jean:
je suis un grand-père qui veut aider sa petite fille :
niveau 4ème
3 exposant -2 =
-3 exposant -4 =
3 exposant -9 =
Je vous remercie de votre aide.
Corldialement.
Jean

Answered by Claude Tardif.
Partages inégaux 2005-09-29
From Un eleve:
Alex et julie ont ensemble 4728 .- euro Si Alex donne 538.- euro à Julie, ce dernier aurait alors le triple de Julie. Quelle est la somme possédée de chacun?
Answered by Claude Tardif.
Problème avec rapport point, prix... 2005-09-08
From Nico:

Le prix d'un diamant est proportionnel au carré de son poinds. Un diamant de 0.45 g vaut 7500euros.

1. Combie coute un diamant de 0.693 g ?

2. Quel est le poids d'une diamant valant 45000 euros ?

Je vous demande pas de le raisoudre à ma place mais j'aimeré comprendre comment je peut calculé le rapport entre le point et le prix.


Answered by Claude Tardif.
Le prix d'un diamant est proportionnel au carré de son poinds 2005-09-07
From Nico:

Le prix d'un diamant est proportionnel au carré de son poinds. Un diamant de 0.45 g vaut 7500euros.

1. Combie coute un diamant de 0.693 g ?

2. Quel est le poids d'une diamant valant 45000 euros ?

Je vous demande pas de le raisoudre à ma place mais j'aimeré comprendre comment je peut calculé le rapport entre le point et le prix.


Answered by Claude Tardif.
Exposant 2005-09-06
From Enzo:
Puis-je savoir combien donne 2 exposant 100 ? et 9 exposant 3012 ?
Answered by Claude Tardif.
le système RSA 2005-02-19
From Un eleve:
Je suis actuellement en classe préparatoire MP au lycée Henri Poincaré de Nancy et je souhaiterais obtenir des informations sur l'exponentiation modulaire car je réalise un TIPE sur la cryptographie et plus particulièrement le système RSA. pourriez-vous m'indiquez en quoi consiste l'exponentiation modulaire et comment cela fonctionne.
Answered by Claude Tardif.
Un problème de math 2004-09-10
From Sophie:
(V est en fait le signe de la racine carrée)

(3 / V27) x ( V75 ) = ???

Answered by Claude Tardif.
La longueur d'une corde 2003-01-22
From Un eleve:
Existe-t-il une formule permettant de calculer la longueur d'une corde quelquonque d'un cercle connaissant uniquement le rayon de celui-ci? Si oui quelle est-elle?
Answered by Claude Tardif.
Volume d'une calotte ellipsoidale 2002-12-11
From Fabien:
Quel est le volume d'une calotte d'ellipsoide de hauteur h (ellipse de longueurs d'axes a, b, et c)? Je souhaite une formule exacte, un développement limité, voire une intégrale à estimer numériquement. Quelle méthode pour le calculer ?
Answered by Claude Tardif.
Un exposant negatif 2002-11-25
From Vinny:
Moi je n'ai aucune idée de ce que vaut un exposant negatif com 3^(-5) ou encore 3^(-4/3)

de meme pour un exposant rationnel 3^(4/3)


Answered by Claude Tardif.
Les unités linéaires 2002-11-12
From Sandy:
je me demande qu'est ce qu'une unité linéaire, mesures de surface et des mesures de volume. je voudrais aussi savoir quelle est la règle de transformation pour les unités linéaires, les mesures de surface et les mesures de volume
Answered by Claude Tardif.
Comment fait on pour justifier que... 2002-09-30
From Lucie:
Comment fait on pour justifier que p au carré est égal à 2 fois q au carré?
Answered by Claude Tardif.
Espaces vectoriels 2002-08-18
From Bacar:
Comment montrer que trois (ou plus que trois) sous espaces vectoriels sont en somme directe.
Answered by Claude Tardif.
Ses points de discontinuités est les irrationnelles 2002-01-08
From Un eleve:
Je voudrait montrer qu'il n'existe pas de fonction de R--->R tels que ses points de discontinuités est les irrationnelles, en utilisant la methode de "baire".
Answered by Claude Tardif.
Fraction rationnelle 2001-12-11
From Audrey:
réduire en une seule fraction et simplifier.

(4-2x+x2)/(2+x) - (x+2)

Answered by Claude Tardif.
Minesweeper 2001-10-24
From Un eleve:
J'espere que vous pourrez m'aider. Est ce que vous connaissez un algorithme ou une strategie de resoulutiom pour le jeu du demineur (8x8 avec 10 mines)(Minesweeper).
Answered by Claude Tardif.
preuve par 9 2001-04-04
From Michel:
Pourriez-vous m'expliquer en détail comment fonctionne la preuve par 9 (pour une division et une multiplication). Je sais l'appliquer mais je ne sais pas pourquoi ça marche. Je ne retrouve pas la démonstration. Merci de m'aider. Exemple . 17x2=34 ; preuve par neuf : 1+7=8 ; 8x2= 16 ; 1+6=7 et 3+4=7, on peut donc supposer (sans affirmer) que cette multiplication a un résultat juste car la preuve par 9 est bonne, 7=7. Comment fonctionne cette preuve par 9???
Answered by Claude Tardif.
Recherhce sur pi et exponentielle 2001-03-17
From Cyril:
étudiant a la fac de sciences de montpellier je cherche des informations concernant la demonstration suivante: "comment prouver que pi et exponentielle sont irrationnels?"
Answered by Claude Tardif.
racine carrée 2000-11-24
From G. Bigorgne:
Quel est donc l'origine de l'expression "racine carrée" ? Pourquoi avoir choisi cette expression ?
Answered by Claude Tardif.
Exposant à la 0 2000-07-04
From Sylvie:
J'aimerais savoir combien fait 0 exposant 0.
Answered by Claude Tardif.
Combien y aura t il de 9 dans la réponse? 2000-06-17
From Marie:
J'ai lu une énigme mais je n'ai pas la réponse."si l'on divise 123 456 789 par 999 999 999, combien y aura t il de 9 dans la réponse?"
Answered by Claude Tardif.
logarithme neperien 1999-11-10
From Louise Kieffer:
D'où provient la valeur e ( 2,7....) des logarithmes népériens ?
Answered by Claude Tardif.
Le sang humain 1998-10-06
From Golden:
Le sang humain contient approximativement 2,5 x globules rouges. Chaque globule a un rayon de 0,004 mm. Si on le place ces globules bout a bout, quelle est la longeur de la chaine en millimetres? En kilometres? Compare toi avec la circonference de la terre (24 000 km) (Travail avec exposants)
Answered by Claude Tradif.
 
Page
1/1

 

 


Math Central is supported by the University of Regina and The Pacific Institute for the Mathematical Sciences.

CMS
.

 

Home Resource Room Home Resource Room Quandaries and Queries Mathematics with a Human Face About Math Central Problem of the Month Math Beyond School Outreach Activities Teacher's Bulletin Board Canadian Mathematical Society University of Regina PIMS